SlideShare uma empresa Scribd logo
1 de 84
Baixar para ler offline
Endocrine 05May2009
DO NOT DISTRIBUTE - 1 -
Endocrine #1 – Histology
1) Which of the following coordinates most of the endocrine functions of the body?
a) Anterior pituitary
b) Posterior pituitary
c) Hypothalamus
d) Cerebral cortex
e) Autocrine cells
2.1) Which of the following hormones does not dissolve in the blood and thus requires
plasma proteins or specialized carrier proteins?
a) Steroid hormones
b) Thyroid hormones
c) Small peptides, proteins, and glycoproteins
d) Catecholamines
e) A & B
f) C & D
2.2) Where are amino acid analogs and their derivatives (including catecholamines)
mainly produced?
a) Ovaries, testes, and adrenal cortex
b) Hypothalamus, pituitary, and pancreas
c) Thyroid and parathyroid
d) Neurons and adrenal medulla
e) Enteroendocrine cells of the GI tract and respiratory system
3.1) Most protein hormones and catecholamines use which of the following second
messenger systems for initiating the hormone-receptor interaction?
a) Adenylate cyclase/camp system
b) Tyrosine kinase system
c) Phosphatidylinositol system
d) Activation of ion channels
e) None of the above (intracellular receptors)
3.2) Which of the following is NOT a second messenger?
a) cAMP
b) 5-HT
c) DAG
d) IP3
e) Ca2+
3.3) Which of the following is NOT true of the intracellular hormone receptor system?
a) Usually localized in the cell nucleus
b) Is used by steroid and thyroid hormones (hydrophobic)
c) Receptors contain complexes with three binding sites
d) Binding causes allosteric receptor transformation that binds chromosomal
DNA and activates or inhibits RNA polymerase activity
e) Receptors influence gene expression but require second messengers
4.1) What portion of the pituitary gland (hypophysis) is most anterior?
a) Pars distalis
b) Pars intermedia
c) Pars tuberalis
Endocrine 05May2009
DO NOT DISTRIBUTE - 2 -
d) Pars nervosa
e) Infundibulum
4.2) What portion of the hypophysis contains neurosecretory axons forming the
hypothalamo-hypophyseal tracts?
a) Pars distalis
b) Pars intermedia
c) Pars tuberalis
d) Pars nervosa
e) Infundibulum
4.3) Which of the following describe the embryology of the hypophysis?
a) The anterior lobe grows down from the neuroectoderm forming the Rathke
pouch and the posterior lobe grows up from the oropharynx ectoderm
b) The anterior lobe grows down from the neuroectoderm and the posterior lobe
grows up from the oropharynx ectoderm forming the Rathke pouch
c) The anterior lobe grows up from the oropharynx ectoderm forming the Rathke
pouch and the posterior lobe grows down from the neuroectoderm
d) The anterior lobe grows up from the oropharynx ectoderm and the posterior
lobe grows down from the neuroectoderm forming the Rathke pouch
e) Both the anterior and posterior lobes extend from the floor of the third ventricle
4.4) The superior hypophyseal arteries arising from the internal carotid and posterior
communicating, supply the areas around and including the pituitary stalk. The inferior
hypophyseal arteries, arising from the internal carotid, supply the pars nervosa. The
hypothalamo-hypophyseal portal system serves to carry the neuroendocrine secretions
directly to what portion of the hypophysis, without entering systemic circulation?
a) Pars distalis
b) Pars intermedia
c) Pars tuberalis
d) Pars nervosa
e) Infundibulum
4.5) Nerves entering the anterior lobe of the pituitary gland have vasomotor function and
arise from:
a) Supraoptic nucleus
b) Paraventricular nucleus
c) Hypothalamo-hypophyseal tract
d) Autonomic postganglionic fibers
e) Autonomic preganglionic fibers
Match the pars distalis hormones with the histologic cell description:
4.6) FSH & LH a) Medium sized, oval cells, round central nuclei
4.7) ACTH b) Large polygonal cells with oval nuclei
4.8) TSH c) Medium sized, polygonal cells with round eccentric nuclei
4.9) PRL d) Small oval cells with round eccentric nuclei
4.10) GH e) Large polygonal cells with round eccentric nuclei
4.11) The pars tuberalis, an extension of the anterior lobe along the pituitary stalk, often
shows immunoreactivity for which of the following?
a) ACTH, FSH, LH
b) TSH, PRL, GH
Endocrine 05May2009
DO NOT DISTRIBUTE - 3 -
c) ACTH, GH, LH
d) FSH, PRL, TSH
e) LH, TSH, GH
4.12) Which of the following is a small polypeptide (4,000kDa) and stimulates secretion
of glucocorticoids and gonadocorticoids by the zona fasciculata and zona reticularis?
a) Growth hormone (somatotropin, GH)
b) Prolactin (PRL)
c) Adrenocorticotropin hormone (ACTH)
d) Follicle-stimulating hormone (FSH)
e) Luteinizing hormone (LH)
f) Thyrotropic hormone (TSH)
g) Oxytocin
h) Antidiuretic hormone (ADH, vasopressin)
4.13) Which of the following stimulates spermatogenesis in the testis?
a) Growth hormone (somatotropin, GH)
b) Prolactin (PRL)
c) Adrenocorticotropin hormone (ACTH)
d) Follicle-stimulating hormone (FSH)
e) Luteinizing hormone (LH)
f) Thyrotropic hormone (TSH)
g) Oxytocin
h) Antidiuretic hormone (ADH, vasopressin)
4.14) Which of the following maintains androgen secretion by the Leydig (interstitial)
cells of the testis and regulates final maturation of the ovarian follicle?
a) Growth hormone (somatotropin, GH)
b) Prolactin (PRL)
c) Adrenocorticotropin hormone (ACTH)
d) Follicle-stimulating hormone (FSH)
e) Luteinizing hormone (LH)
f) Thyrotropic hormone (TSH)
g) Oxytocin
h) Antidiuretic hormone (ADH, vasopressin)
4.15) Oxytocin stimulates ejection of milk from the glands and stimulates contraction of
the smooth muscle cells in the pregnant uterus. Which of the following, like oxytocin, is
produced then stored in the neurohypophysis?
a) Antidiuretic hormone (ADH, vasopressin)
b) Adrenocorticotropin hormone (ACTH)
c) Growth hormone (somatotropin, GH)
d) Follicle-stimulating hormone (FSH)
e) Thyrotropic hormone (TSH)
f) Luteinizing hormone (LH)
g) Prolactin (PRL)
5.1) Which of the following hypothalamic regulating hormones inhibits secretion of PRL
by lactotropes?
a) Growth hormone-releasing hormone (GHRH)
b) Dopamine
Endocrine 05May2009
DO NOT DISTRIBUTE - 4 -
c) Corticotropin-releasing hormone (CRH)
d) Somatostatin
e) Thyrotropin-releasing hormone (TRH)
f) Gonadotropin-releasing hormone (GnRH)
5.2) Which of the following hypothalamic regulating hormones inhibits secretion of GH?
a) Growth hormone-releasing hormone (GHRH)
b) Dopamine
c) Corticotropin-releasing hormone (CRH)
d) Somatostatin
e) Thyrotropin-releasing hormone (TRH)
f) Gonadotropin-releasing hormone (GnRH)
5.3) Which of the following stimulates secretion of LH and FSH?
a) Growth hormone-releasing hormone (GHRH)
b) Dopamine
c) Corticotropin-releasing hormone (CRH)
d) Somatostatin
e) Thyrotropin-releasing hormone (TRH)
f) Gonadotropin-releasing hormone (GnRH)
6.1) Which of the following is NOT true of the pineal gland?
a) Located on the posterior wall of the third ventricle
b) Shaped like a flattened pinecone
c) Develops from oropharynx ectoderm
d) Pinealocytes have a large deeply infolded nucleus with one or more nuclei
e) Functions in regulation of daily body rhythm (circadian)
6.2) Which of the following describes the release and action of melatonin?
a) Darkness inhibits the production of melatonin and melatonin stimulates the
neurons in the hypothalamus that produce GnRH
b) Darkness inhibits the production of melatonin and melatonin inhibits the
neurons in the hypothalamus that produce GnRH
c) Daylight inhibits the production of melatonin and melatonin stimulates the
neurons in the hypothalamus that produce GnRH
d) Daylight inhibits the production of melatonin and melatonin inhibits the
neurons in the hypothalamus that produce GnRH
7.1) Which of the following occurs at week 14 of development with regard to the thyroid
gland?
a) Colloid is produced and the gland starts working
b) Thyroid hormone is lost leading to cretinism
c) Follicles develop from endodermal cells
d) Thyroglossal duct descends from endoderm pharynx
e) Thyroglossal duct divides into two lobes at the trachea
7.2) Which of the following is NOT true?
a) Thyroxine (T4) and triiodothyronine (T3) regulate cell basal metabolism and
heat production
b) Calcitonin (thyrocalcitonin) increase blood calcium levels
c) Follicular cells produce T3 and T4
d) Parafollicular cells secrete calcitonin, the physiologic antagonist to PTH
Endocrine 05May2009
DO NOT DISTRIBUTE - 5 -
e) The thyroid follicle produces the colloid thyroglobin, which is the inactive
storage form for the thyroid hormones
7.3) Which of the following is true of tetraiodothyronine (T4)?
a) Formed by the addition of four iodine atoms to thyroglobulin
b) Formed by oxidative coupling of four monoiodotyrosine (MITs)
c) Formed by oxidative coupling of two diiodotyrosines (DITs)
d) Formed by oxidative coupling of two MITs and a DIT
e) Secreted in a T4:T3 ration of 1:20
8.1) Which of the following is true of the parathyroid glands?
a) Superior and inferior glands develop from the third brachial pouch
b) Superior and inferior glands develop from the fourth brachial pouch
c) Superior glands develop from third brachial pouch and inferior from fourth
d) Superior glands develop from fourth brachial pouch and inferior from third
e) All four glands will receive full blood supply without the inferior thyroid artery
8.2) Which of the following is NOT true of parathyroid hormone (PTH)?
a) Promotes calcium release from bone
b) Stimulates calcium reabsorption at distal kidney tubule
c) Increases formation of 1,25-dihydroxycholecalciferol (1,25-OH2)
d) Secreted by chief (principle) cells
e) Absorbed by oxyphil cells
f) Reduces serum phosphate
9.1) Which of the following describes the embryonic development of the adrenal gland?
a) Cortical cells arise from mesoderm; medulla cells originate from endoderm
b) Cortical cells arise from neural crest; medulla cells originate from ectoderm
c) Cortical cells arise from ectoderm; medulla cells originate from neural crest
d) Cortical cells arise from mesoderm; medulla cells originate from neural crest
e) Cortical cells arise from neural crest; medulla cells originate from endoderm
9.2) Which of the following layers of the adrenal gland does NOT receive a direct blood
supply, but receives blood via sinusoids?
a) Capsule
b) Cortex
c) Medulla
Match the adrenal gland secreted hormone with the location:
9.3) Catecholamines (epinephrine) a) Zona glomerulosa
9.4) Mineralcorticoids (aldosterone) b) Zona fasciculata
9.5) Gonadocorticoids (DHEA) c) Zona reticularis
9.6) Glucocorticoids (Cortisol) d) Medullary chromaffin cells
Match the function with the adrenal gland secreted hormone:
9.7) Induces weak masculinizing effect a) Mineralcorticoids
9.8) Aid in controlling electrolyte homeostasis b) Glucocorticoids
9.9) Promote normal metabolism c) Gonadocorticoids
10.1) Which of the following is formed from the ventral bud from the hepatic duct (not
dorsal bud from the foregut)?
a) Uncinate process
b) Body
c) Tail
Endocrine 05May2009
DO NOT DISTRIBUTE - 6 -
d) Anterior head
10.2) What is the function of somatostatin (from D cells)?
a) Activates insulin and glucagon secretions
b) Inhibits insulin and glucagon secretions
c) Activates insulin secretions and inhibits glucagon secretions
d) Activates glucagon secretions and inhibits insulin secretions
10.3) Which of the following is a function of insulin, secreted from beta cells?
a) Stimulates gluconeogenesis
b) Stimulates glycogenolysis
c) Mobilizes fat
d) Phosphorylation and use of glucose
e) Stimulates hepatic lipase
f) Increases blood glucose level
11) A patient presents to the Emergency Room after exposure to radiation from an atomic
bomb. This explosion has likely created radioactive iodine. Flooding which of the
following locations with potassium iodine (e.g. IOSAT pill) would help prevent side
effects?
a) Islet of Langerhans alpha cells
b) Islet of Langerhans beta cells
c) Adrenal cortex
d) Thyroid gland
e) Pineal gland
Endocrine #2 – Physiology: Hypothalamus & Pituitary
1) Which of the following would be involved in a negative feedback signaling loop
located on the same cell?
a) Autocrine signaling
b) Paracrine signaling
c) Endocrine signaling
d) Neurocrine signaling
2) The release of oxytocin to cause contractions during birth is an example of:
a) Autocrine signaling
b) Paracrine signaling
c) Endocrine signaling
d) Neurocrine signaling
3) Pancreatic beta cells secreting insulin that acts on the alpha cells is an example of:
a) Autocrine signaling
b) Paracrine signaling
c) Endocrine signaling
d) Neurocrine signaling
4) The circadian cycle stimulates hormone release (GH and cortisol) in the absence of
external stimulation. This is an example of:
a) Neurotransmitter stimulated hormone release
b) Hormone stimulated hormone release
c) Pulsatile stimulated hormone release
d) Stress stimulated hormone release
Endocrine 05May2009
DO NOT DISTRIBUTE - 7 -
5) Labs are taken from a patient every hour over a 24-hour period. The labs include
cortisol from the blood and GH via binding protein. When would these values most likely
be the highest?
a) Night (2am)
b) Morning (8am)
c) Afternoon (2pm)
d) Evening (8pm)
6) What GLUT transporter is inserted into target cells (muscle, adipose) when insulin is
released from beta cells, which drops blood glucose and causes a negative feedback loop?
a) GLUT1
b) GLUT2
c) GLUT3
d) GLUT4
e) GLUT5
7) The hypothalamus releases a thyroid-releasing hormone (TRH), which stimulates the
anterior pituitary to release a trophic hormone (TSH), which affects the target gland.
Which of the following describes the negative feedback loop for this process?
a) Pituitary hormones reduce the effect of the hypothalamus
b) Pituitary hormones reduce the effect of the hypothalamus and pituitary
c) Target gland hormones reduce the effect of the pituitary
d) Target gland hormones reduce the effect of the pituitary and hypothalamus
e) A & D
f) B & C
g) B & D
8) The hypothalamus releases a cortisol-releasing hormone (CRH), which stimulates the
pituitary to release ACTH, which stimulates the adrenal cortex to release cortisol. Which
of the following describes the negative feedback loop for this process?
a) Pituitary hormones reduce the effect of the hypothalamus
b) Pituitary hormones reduce the effect of the hypothalamus and pituitary
c) Adrenal cortex hormones reduce the effect of the pituitary
d) Adrenal cortex hormones reduce the effect of the pituitary and hypothalamus
e) A & D
f) B & C
g) B & D
9) If the adrenal cortex were disable such that it could not participate in the negative
feedback loop, which of the following would occur?
a) Decreased CRH and increased cortisol
b) Decreased CRH and decreased cortisol
c) Increased CRH and absence of ACTH
d) Decreased ACTH
e) Increased ACTH
10) If the pituitary gland were disabled such that it could not participate in the cortisol
negative feedback loop, which of the following would occur?
a) Increased CRH and increased cortisol
b) Increased CRH and decreased cortisol
c) Decreased CRH and increased cortisol
Endocrine 05May2009
DO NOT DISTRIBUTE - 8 -
d) Decreased CRH and decreased cortisol
11) A 40-year-old truck driver has had difficulty using his side mirrors for traffic behind
him. He has never had any major medical problems in the past. He visits an optometrist,
who determines he has bitemporal hemianopsia, but his vision is 20/20. A head CT scan
reveals slight enlargement of the sella turcica. Which of the following hormones is most
likely being secreted in excessive amounts in this man?
a) Antidiuretic hormone
b) Prolactin
c) ACTH
d) Growth hormone
e) Luteinizing hormone
12) Which of the following targets to the anterior pituitary is inhibitory?
a) CRH (corticotropin-releasing hormone)
b) TRH (thyrotropin-releasing hormone)
c) DA (dopamine acting on prolactin)
d) PRH (prolactin-releasing hormone)
13) Hypothalamic nerve stimulation to the posterior pituitary would lead to all of the
following EXCEPT:
a) Water retention
b) Decreased blood osmolality
c) Uterine contraction
d) Thyroid stimulation
e) Breast milk ejection
14) A patient presents with dilute, high volume urine (polyuria) and extreme thirst
(polydipsia). Which of the following is most likely?
a) No ADH
b) Normal ADH, low sugar
c) Normal ADH, high sugar
d) Excess ADH
15) A 20-pack-year smoker presents with concentrated, high volume urine and expanded
ICR and ECF volume. Which of the following is most likely?
a) No ADH
b) Normal ADH, low sugar
c) Normal ADH, high sugar
d) Excess ADH
16) How does the hypothalamus cause the release of anterior pituitary hormones?
a) Nerve endings directly connect to anterior pituitary
b) Nerve stimulation of the posterior pituitary causes hormone secretion that
activates the anterior pituitary
c) Nerve ending hormones release hormones directly into the anterior pituitary
d) Nerve ending hormones release hormones that follow the systemic blood
supply into the anterior pituitary
e) Nerve ending hormones release hormones that follow the portal blood supply
into the anterior pituitary
17) Which of the following hormones is derived from (created in) the posterior pituitary?
a) Oxytocin
Endocrine 05May2009
DO NOT DISTRIBUTE - 9 -
b) ACTH
c) TSH
d) ADH
e) No hormones
18) Which of the following hormones of the anterior pituitary targets the liver?
a) ACTH (adrenocorticotropic hormone)
b) TSH (thyroid stimulating hormone)
c) GH (growth hormone)
d) FSH (follicle stimulating hormone)
e) PRL (prolactin)
19) Which of the following hormones of the hypothalamic-anterior pituitary axis cause
the release of FSH?
a) TRH
b) CRH
c) GHRH
d) LHRH
e) PRH
20) If the connection between the hypothalamus and anterior pituitary were severed,
which of the following would increase?
a) TSH
b) PRL
c) ACTH
d) LH & FSH
e) GH
21) A middle-aged female patient presents to her physician complaining of weight gain,
weakness, lethargy, and dizziness. She was recently diagnosed with pulmonary
carcinoma and reports that her symptoms began within the last week. A MRI of the lung
region revealed that her lung tumor grew rapidly over the last few weeks. Physical
examination found elevated blood pressure but little to no edema. Laboratory tests
indicate severely elevated antidiuretic hormone (ADH). Further laboratory tests would
indicate:
a) Elevated renal sodium reabsorption
b) Elevated serum sodium concentration
c) Increased atrial natriuretic peptide (ANP) secretion
d) Polyuria
e) Reduced urinary osmolality
22) A 25-year-old male presents to his physician complaining of loss of peripheral vision.
Physical examination was normal, but a CT scan revealed a very large mass on the
anterior pituitary. Laboratory testing could reveal:
a) Elevated serum corticotropin releasing hormone (CRH)
b) Elevated serum growth hormone releasing hormone (GHRH)
c) Elevated serum oxytocin
d) Elevated serum prolactin
e) Elevated urine osmolality
23) A 45-year-old female is diagnosed with an anterior pituitary tumor. Her physician
explains that the best course of treatment would be surgery, but that she would have to
Endocrine 05May2009
DO NOT DISTRIBUTE - 10 -
undergo hormone replacement post-operatively. The hormones that would most likely
have to be replaced include:
a) Antidiuretic hormone (ADH)
b) Corticotropin releasing-hormone (CRH)
c) Growth hormone releasing-hormone (GHRH)
d) Oxytocin
e) Thyroid stimulating hormone (TSH)
Endocrine #3 – Physiology: Thyroid Hormones & Action
1) Which of the following is involved in a positive feedback loop to the hypothalamus?
a) Thyroid releasing factor (TRH)
b) Somatotropin release-inhibiting factor (SRIF)
c) Thyroid stimulating hormone (TSH)
d) Somatostatin
e) T4 & T3
2) Which of the following would occur without a thyroid present?
a) TRH increases and TSH increases
b) TRH increase and TSH decreases
c) TRH decreases and TSH increases
d) TRH decreases and TSH decreases
e) T4, T3, and somatostatin increase
Match the thyroid disorder: T4 TSH TRH
3) Primary hypothyroidism a) Decreased Decreased Increased
4) Pituitary hypothyroidism b) Increased Increased Decreased
5) Hypothalamic hypothyroidism c) Decreased Increased Increased
6) Pituitary hyperthyroidism d) Decreased Decreased Decreased
7) Graves disease (autoimmune) e) Increased Decreased Decreased
8) Calcitonin is produced in which of the following locations?
a) Follicular cells
b) Colloid
c) Thyroid capillary
d) Parafollicular cells
e) None of the above
9) Demineralization and glycosylation of thyroglobulin occurs at what cellular location?
a) Mitochondria
b) Smooth ER
c) Rough ER
d) Golgi apparatus
e) Vesicle before entering colloid
10) Iodine is absorbed against an electrochemical gradient (Na+/I- symporter) and
concentrated above blood levels (iodide trapping) in what location?
a) Follicular cells
b) Colloid
c) Thyroid capillary
d) Parafollicular cells
e) None of the above
Endocrine 05May2009
DO NOT DISTRIBUTE - 11 -
11) Iodide is sent to the colloid via:
a) I-/Cl- exchanger (pedrin)
b) Na+/I- symporter
c) Na+/K+ pump
d) Passive diffusion
e) Active diffusion
12) What is the role of thyroid peroxidase?
a) Releases iodine from thyroglobulin
b) Binds iodine to thyroglobulin
c) Binds two DITs to make T4
d) Binds MIT to DIT to make T3
e) Binds two iodide atoms to make iodine
13) At what location does iodide bind to tyrosine residues on thyroglobulin to form DITs
or MITs, leading to a 2-3 month stored supply of T3 and T4?
a) Follicular cells
b) Colloid
c) Thyroid capillary
d) Parafollicular cells
14) Which of the following is the active form of thyroid hormone?
a) T4
b) T3
c) rT3
15) When T4 diffuses into a cell and is deiodinized to T3, which of the following occurs
prior to gene transcription with RNA polymerase II?
a) T3 binds to a thyroid receptor
b) T3 displaces a corepressor
c) T3 recruits a coactivator
d) A & B
e) All of the above
16) Which of the following actions of thyroid hormone acts synergistically with GH?
a) Increased basal O2 consumption and heat production
b) Increased respiratory rate and minute ventilation
c) Increased cardiac output (HR & SV)
d) Increased linear bone growth
e) Increased glucose absorption
17) Which of the following, along with norepinephrine, decreases with thyroid hormone
action?
a) Hypothalamus SNS center stimulation
b) Red blood cell mass and O2 capacity
c) Total peripheral resistance
d) Cholesterol turnover (LDL decrease)
e) Epinephrine activity (lipolysis)
f) Sensory awareness and memory
18) Which of the following would occur in a patient who developed antibodies that are
bound to TSH-R and competitively inhibited TSH (Hashimoto disease)?
a) TSH increases
Endocrine 05May2009
DO NOT DISTRIBUTE - 12 -
b) TRH decreases
c) T4 production increases
d) T3 production increases
19) A patient develops antibodies that bind TSH-R and cause hyperstimulation (Graves
disease). Which of the following is most likely?
a) Increased T4, increased TSH, increased TRH
b) Increased T4, increased TSH, decreased TRH
c) Increased T4, decreased TSH, decreased TRH
d) Increased T4, decreased TSH, increased TRH
e) Decreased T4, increased TSH, decreased TRH
20) A 32-year-old woman complains of excessive tiredness, weight gain, constipation
and breast discharge. Laboratory results indicate that serum thyroid stimulating hormone
(TSH) was markedly decreased. Additional physical examination and laboratory testing
would most likely reveal:
a) Decreased thyrotropin-releasing hormone (TRH)
b) Increased respiration rate
c) Increased TRH
d) Increased triiodothyronine (T3)
e) Increased T4
Three patients with hypothyroidism arrive at the laboratory for a thyroid-releasing
hormone (TRH) test. During this test, TRH was injected at time 0 and plasma TSH was
measured for 3 hours. The following graph was created for the findings:
21) What disorder does Patient A have?
a) Thyroid
b) Hypothalamus
c) Pituitary
22) What disorder does Patient B have?
a) Thyroid
b) Hypothalamus
c) Pituitary
23) What disorder does Patient C have?
a) Thyroid
b) Hypothalamus
c) Pituitary
Endocrine 05May2009
DO NOT DISTRIBUTE - 13 -
24) An adult female patient presented to her physician complaining that she feels weak
and fatigue all the time. She reports that she and her husband are getting ready to be
divorced, and that she is feeling stressed. Laboratory tests for thyroid function revealed
no abnormalities. The physician, however, prescribed her T3. If further testing was done
2 days after beginning treatment, the findings would most likely include:
a) Elevated thyroid stimulating hormone (TSH)
b) Increased iodide uptake
c) Increased thyroglobulin synthesis
d) No change in iodide uptake
e) Reduced T4
25) An experimental animal was created to determine the effects of thyroid stimulating
hormone (TSH) deficiency on growth and development. Scientists genetically altered a
mouse model by mutating the gene responsible for TSH receptors. A colony was created
and called the TSH receptor knockout (TSH-R-KO) mouse strain. The predicted
phenotype of TSH-R-KO offspring would include:
a) Above average intellectual capabilities
b) Dwarfism
c) Elevated basal metabolic rate
d) Elevated body temperature
e) Elevated growth hormone (GH) secretion
26) A female patient presents to her physician complaining of agitation, weight loss,
increased appetite, abnormal menstrual cycles, and sleep disturbances. Physical
examination reveals that her skin is moist and she has a temperature of 100°C. She
reports that her skin itches sometimes and examination reveals pretibial myxedema. A
triiodothyronine (T3) resin uptake test (T3RU) revealed increased T3 resin uptake. The
physician diagnoses her with Graves’ disease. Further laboratory tests would reveal:
a) Decreased T3 levels
b) Decreased T4 levels
c) Elevated thyroid binding globulin concentration
d) Elevated thyroid stimulating hormone
e) Saturation of thyroid binding globulin (TBG) binding sites
Endocrine #4 – Physiology: Parathyroid Regulation & Bone Remodeling
1) Where is calcium mainly stored in the body?
a) Bound to albumin
b) Bone and teeth
c) Kidneys
d) Ionized in serum
e) Non-ionized in serum (complexed)
f) Muscle
2) What calcium turnover process results in the loss of the most calcium?
a) Bone remodeling
b) Kidney filtering
c) ECF secretion
d) Intestinal excretion
e) Exchangeable pool
Endocrine 05May2009
DO NOT DISTRIBUTE - 14 -
3) Where is phosphate mainly stored in the body?
a) Teeth
b) Muscle
c) Bone
d) Kidneys
e) Blood
4) What phosphate turnover process results in the loss of the most phosphate?
a) Bone remodeling
b) Kidney filtration
c) Exchangeable pool
d) Intestinal secretion
e) Soft tissue remodeling
5) Which of the following describes the action of parathyroid hormone (PTH)?
a) Increased Ca++ and bone reabsorption, increased phosphate reabsorption
b) Increased Ca++ and bone reabsorption, decreased phosphate reabsorption
c) Decreased Ca++ and bone reabsorption, increased phosphate reabsorption
d) Decreased Ca++ and bone reabsorption, decreased phosphate reabsorption
6) Which of the following describes the action of vitamin D (intestine & bone)?
a) Increased Ca++ and bone absorption, increased phosphate absorption
b) Increased Ca++ and bone absorption, decreased phosphate absorption
c) Decreased Ca++ and bone absorption, increased phosphate absorption
d) Decreased Ca++ and bone absorption, decreased phosphate absorption
7) Which of the following calcium changes would occur in a patient who has their
parafollicular thyroid cells removed?
a) Increased calcium stores
b) Decreased calcium stores
c) Increased serum calcium
d) Decreased serum calcium
e) No change in calcium
8) A patient is found to have a PTH tumor. Which of the following sequela may occur?
a) Difficulty urinating
b) Goiter
c) Soft tissue pain
d) Bone fracture
e) Carotid bruit
9) What is the action of calcitonin?
a) Activate osteoblasts
b) Deactivate osteoblasts
c) Activate osteoclasts
d) Deactivate osteoclasts
10) Which of the following is NOT true of parathyroid hormone?
a) Produced by chief cells of the parathyroid gland
b) Decreases ionized plasma Ca++ levels
c) Increases plasma phosphate levels
d) Inhibited by 1,25-OH2-D via negative feedback
e) Decreased vitamin D production and bone resorption
Endocrine 05May2009
DO NOT DISTRIBUTE - 15 -
11) A patient presents with complains of lethargy. Testing reveals a likely PTH-releasing
tumor. If phosphate is the likely cause of the lethargy, how is PTH affecting the kidneys?
a) Increased reabsorption of phosphate at the proximal tubule
b) Decreased reabsorption of phosphate at the proximal tubule
c) Increased reabsorption of phosphate at the distal tubule
d) Decreased reabsorption of phosphate at the distal tubule
12) What affect does PTH have on calcium in the kidneys?
a) Increased reabsorption of calcium at the proximal tubule
b) Decreased reabsorption of calcium at the proximal tubule
c) Increased reabsorption of calcium at the distal tubule
d) Decreased reabsorption of calcium at the distal tubule
13) Which of the following would be seen in a patient with hypoparathyroidism?
a) Hypercalcemia
b) Hypercalciuria
c) Renal stones
d) Hyperphosphatemia
e) Decreased bone mass
14) Vitamin D (D2 in plants/yeast, D3 in diet/skin) must be hydroxylated to its active
form, 1,25-OH2-D. Where does the first hydroxylation occur?
a) Liver
b) Blood
c) Kidney proximal tubule
d) Kidney distal tubule
e) Bone and teeth
15) Where does 1alpha-hydroxylase act?
a) Liver
b) Blood
c) Kidney proximal tubule
d) Kidney distal tubule
e) Bone and teeth
16) Which of the following describes the activity of 1,25-OH2D3?
a) Increases calcium absorption in the intestine
b) Increases calcium excretion from the intestine
c) Increases proximal tubule calcium reabsorption
d) Decreases basolateral membrane Ca++ pumps
e) Decreases calbindin synthesis
17) How is calcium transported from the basolateral side to the interstitial space?
a) Ca++/H+ ATPase
b) 3Na+/Ca++ exchanger
c) Calbindin
d) A & B
e) A & C
18) A child presents with leg-bone deformity. Blood testing shows high levels of vitamin
D in the blood. Which of the following is most likely?
a) Type I rickets
b) Type II rickets
Endocrine 05May2009
DO NOT DISTRIBUTE - 16 -
c) Renal stones
d) Excess 1,25-OH2D3
e) Calcitonin deficiency
19) A man presents to his physician complaining of muscle cramping, irritability and
tingling in the fingers and toes. Laboratory tests indicate that he is hypocalcemic and
hyperphosphatemic. All other measures are unremarkable. Further laboratory tests would
indicate:
a) Elevated calcitonin levels
b) Hyperparathyroidism
c) Hypoparathyroidism
d) Vitamin D deficiency
e) Vitamin D excess
20) A female patient presents complaining of pain in the lower right side of her back,
nausea, constipation, fatigue, muscle weakness, and lethargy. An MRI of the
thyroid/parathyroid region revealed a small tumor on the parathyroid gland causing
parathyroid over-activation. Further laboratory tests would indicate:
a) High serum phosphate, high serum calcium, and high serum parathyroid
hormone (PTH)
b) High serum phosphate, low serum calcium, and low serum PTH
c) Low serum phosphate, high serum calcium, and high serum PTH
d) Low serum phosphate, high serum calcium, and low serum PTH
e) Low serum phosphate, low serum calcium, and high serum PTH
21) A female patient with renal disease undergoes laboratory tests to measure plasma
ions and renal function. Based on the results, her physician diagnoses her with a rare
autoimmune disorder that is causing progressive kidney destruction. This destruction will
probably also result in the impairment in the conversion of:
a) 1,25-dihydroxycholecalciferol to 25-hydroxycholecalciferol
b) 25-hydroxycholecalciferol to 1,25-dihydroxycholecalciferol
c) Cholesterol to 7-dehydrocholesterol
d) Vitamin D3 to 25-hydroxycholecalciferol
e) Vitamin D3 to Vitamin D2
22) Which of the following is true?
a) Most of bone mass is cortical bone and remodeling mostly affects cortical bone
b) Most of bone mass is cortical bone and remodeling mostly affects spongy bone
c) Most of bone mass is spongy bone and remodeling mostly affects cortical bone
d) Most of bone mass is spongy bone and remodeling mostly affects spongy bone
23) Which of the following is contained within the osteoid?
a) Ground substance
b) Type I collagen
c) Proteoglycans
d) Minerals (CaPO4, Na, Carbonate, Mg)
e) All of the above
24) Which of the following is stimulated by cytokines and inhibited by calcitonin?
a) Osteoblasts
b) Osteocytes
c) Osteoclasts
Endocrine 05May2009
DO NOT DISTRIBUTE - 17 -
d) Osteocalcin
e) Osteonectin
25) Annexins are secreted by which of the following in order to recruit more of their
own?
a) Osteoblasts
b) Osteocytes
c) Osteoclasts
26) What paracrine signaling is accomplished during bone remodeling?
a) Osteocytes signal osteoblasts
b) Osteocytes signal osteoclasts
c) Osteoblasts signal osteoclasts
d) Osteoblasts signal osteocytes
e) Osteoclasts signal osteoblasts
27) PTH stimulates bone resorption by activating:
a) Osteoblasts
b) Osteocytes
c) Osteoclasts
28) Which of the following people would have the most bone mass?
a) Teenage soccer player
b) Young adult weight lifter
c) Female undergoing menopause
d) Adult male jogger
e) Elderly bedridden patient
Endocrine #5 – Physiology: Adrenal Hormones
Match the hormone with the primary location of production:
1) Catecholamines a) Adrenal cortex, zona glomerulosa
2) Aldosterone b) Adrenal cortex, zona fasciculata
3) Cortisol c) Adrenal cortex, zona reticularis
4) Androgens d) Adrenal medulla
5) A patient with hyperaldosteronism would likely have:
a) Low ANP levels
b) Low 18-OH-DOC levels
c) Hypercalcemia
d) Hypomagnesemia
e) Hypokalemia
6) Which of the following would lead to an overall decrease in sodium and water
excretion due to activation of ADH in the renin-angiotensin-aldosterone system (RAAS)?
a) Increased blood pressure
b) Decreased blood hematocrit
c) Increased blood osmolarity
d) Decreased blood osmolality
e) Increased sodium retention
7) Angiotensin II is converted to aldosterone at what location?
a) Brain
b) Adrenals
Endocrine 05May2009
DO NOT DISTRIBUTE - 18 -
c) Lungs
d) Liver
e) Kidneys
8) What is the main function of aldosterone?
a) Decrease sodium reabsorption
b) Increase K+ secretion
c) Maintain ECF volume
d) Increase H+ secretion
e) Deplete ICF volume
9) Cortisol releasing factor (CRF) from the hypothalamus would lead to all of the
following EXCEPT:
a) Increased growth
b) Increased blood pressure
c) Increased arousal
d) Increased SNS
e) Decreased reproductive hormones
10) Which of the following would inhibit ACTH?
a) Stress
b) Depression
c) Anxiety
d) Cortisol
e) Alpha-agonists
11) Which of the following would lead to a higher setpoint for cortisol due to an over-
ride of the negative feedback loop?
a) Sleeping during the day
b) Strenuous exercise
c) Constant stress
d) High fat diet
e) Decreased CRH
12) Glucocorticoids (e.g. cortisol) inhibit which of the following via a negative feedback
loop?
a) Hypothalamus
b) Corticotroph (anterior pituitary)
c) Adrenal cortex
d) A & B
e) A & C
Match the glucocorticoid disorder: Plasma cortisol Plasma ACTH
13) Primary hypercortisolism a) Increased Increased
14) Secondary hypercortisolism b) Decreased Increased
15) Primary hypocortisolism c) Increase Decreased
16) Secondary hypocortisolism d) Decreased Decreased
17) Which of the following is NOT an effect of glucocorticoids?
a) Insulin resistance
b) Lipogenesis in the trunk
c) NPY inhibition
d) Lipolysis in the extremities
Endocrine 05May2009
DO NOT DISTRIBUTE - 19 -
e) Muscle protein turnover and gluconeogenesis
18) Which of the following is NOT an effect of glucocorticoids?
a) Decrease in muscle mass and strength
b) Decreased bone formation and increased resorption
c) Thinning of skin and capillary walls
d) Decreases glomerular filtration rate
e) Decreased memory
19) Which of the following is NOT an effect of glucocorticoids?
a) Inhibits arachidonic acid synthesis
b) Decreases neutrophil number but increases effectiveness
c) Decreases fibroblast proliferation
d) Stimulates T-cell apoptosis
e) Inhibits IL, INF-gamma, COX-2 expression
20) Which of the following is true of androgens?
a) Most of male androgens exist in the adrenals
b) Female ovaries provide most of their androgens
c) A castrated male can use the adrenals to compensate for testosterone loss
d) Adrenal androgen is converted to testosterone in the female pubic and axillary
areas
e) Adrenal androgens are of great importance to a physiologic male
21) Scientists were conducting an experiment to look at the effect of surgical ablation on
kidney function. After doing 40 ablation surgeries, their student was tired and anxious to
finish. On his 41st surgery he accidentally removed a large section of the outer layer of
the adrenal gland. If the animal were allowed to recover from surgery, hormonal
measurements would probably indicate:
a) Decreased adrenocorticotropic hormone (ACTH) secretion
b) Decreased aldosterone production
c) Decreased androgen production
d) Decreased catecholamine production
e) Decreased stress-induced cortisol secretion
22) In an experiment designed to characterize the hypothalamic-pituitary-adrenal axis in
rodents, plasma adrenocorticotropic hormone (ACTH) and corticosterone were measured
in several rats following various surgical and pharmaco-
logical manipulations. The following graph plots the
relative hormone levels for each animal. Hormonal profiles
for each group reveal:
a) Group A may have adrenal hyperplasia
b) Group A may have primary adrenal insufficiency
c) Group B may have hypothalamic deficiency
d) Group B may have pituitary deficiency
e) Group C may have adrenal hyperplasia
23) A male patient presents to his physician complaining of muscle weakness and
abnormal heartbeat he describes as “beating out of tune.” He was previously been
diagnosed with Type 1 (insulin-dependent) diabetes mellitus, but reports strict adherence
to insulin treatments and eats a low-salt and low-carbohydrate diet. Laboratory tests
Endocrine 05May2009
DO NOT DISTRIBUTE - 20 -
measuring serum electrolytes revealed low sodium and elevated potassium levels. Further
laboratory tests would reveal:
a) Elevated ACTH secretion
b) Elevated aldosterone secretion
c) Elevated glucocorticoid secretion
d) Reduced ACTH secretion
e) Reduced renin secretion
24) A 49-year-old male presents to his physician complaining of a persistent “dry” cough,
muscle weakness, and reduced libido. Physical examination reveals abnormal weight gain
in the back, abdomen, and face and abdominal stria. He reports that he has smoked 2
packs of cigarettes per day for the last 30 years. Laboratory tests revealed elevated serum
glucose, a positive dexamethasone suppression test, and a hormone-secreting bronchial
carcinoma. Further laboratory tests would most likely reveal that the hormone secreted by
the tumor is:
a) Adrenocorticotropic hormone (ACTH)
b) Cortisol
c) Glucagon
d) Growth hormone
e) Thyroxine
25) A male patient is experiencing adrenal insufficiency and cannot produce adequate
amounts of cortisol, either basal or stress-induced. Under basal conditions metabolism is
not greatly affected, but he experiences severe weakness when confronted with a stressor.
Laboratory tests conducted during periods of stress would likely indicate:
a) Decreased glucose uptake in muscles and adipose
b) Hypoglycemia
c) Increased circulating amino acids
d) Increased gluconeogenesis
e) Increased lipoysis
26) Where is norepinephrine mostly produced?
a) Preganglionic sympathetic fibers
b) Postganglionic sympathetic fibers
c) Preganglionic parasympathetic fibers
d) Postganglionic parasympathetic fibers
27) Which of the following actions is decreased by epinephrine and norepinephrine?
a) Insulin action
b) Glucagon secretion
c) BMR and thermogenesis
d) Gluconeogenesis
e) Glycogenolysis
28) Which of the following actions is decreased by epinephrine and norepinephrine?
a) Heart rate
b) Renin secretion
c) Skin blood flow
d) Systolic pressure
e) Cardiac contractility
f) Bronchodilation
Endocrine 05May2009
DO NOT DISTRIBUTE - 21 -
29) A male patient presents to his physician complaining of feelings of “nervousness”
followed by fatigue. Subsequent tests revealed an adrenal tumor in the medullary region.
Further laboratory testing would most likely reveal:
a) Decreased circulating free fatty acids
b) Decreased circulating glucose levels
c) Decreased circulating insulin levels
d) Increased glycogenesis
e) Increased lipogenesis
Endocrine #6 – Physiology: Hormones Involved in Food Intake & Energy Storage
Match the hypothalamic location with the function:
1) Satiety a) Lateral nuclei
2) Regulation of feeding b) Ventromedial nuclei
3) Hunger c) Paraventricular nucleus (PVN)
4) Multi-hormone action d) Dorsomedial nuclei (DMN)
5) Increase eating e) Arcuate nuclei (ACN)
6) Which of the following components of the arcuate nucleus decreases food intake and
increases energy expenditure?
a) Alpha-MSH (melanocyte-stimulating hormone)
b) NPY (neuropeptide Y)
c) CART (cocaine- and amphetamine-related transcripts)
d) AGRP (agouti-related protein)
e) A & C
f) B & D
7) POMC neurons release alpha-MSH, which acts on MCR-3 and MCR-4 melanocortin
receptors in the paraventricular nucleus. Which of the following would occur if a patient
had an MCR-4 mutation or who receives MCR-4 antagonist (AGRP)?
a) Addiction to cocaine
b) Rapid weight loss
c) Early satiety
d) Obesity
e) Intolerance to amphetamines
8) Which of the following decreases POMC activation and is the most associated with
increased appetite (hunger)?
a) NPY
b) Alpha-MSH
c) CART
d) AGRP
9) Antagonizing which of the following could be a helpful treatment for an obese patient
whose weight problem is caused by Leptin?
a) MCH (melanin-concentrating hormone)
b) CART (cocaine- and amphetamine-related transcripts)
c) Orexin (hypocretin)
d) NPY (neuropeptide Y)
e) CB1 (cannabinoid)
10) Which of the following would increase appetite and decrease ACTH levels?
Endocrine 05May2009
DO NOT DISTRIBUTE - 22 -
a) MCH (melanin-concentrating hormone)
b) CART (cocaine- and amphetamine-related transcripts)
c) Orexin (hypocretin)
d) NPY (neuropeptide Y)
e) CB1 (cannabinoid)
11) A deficiency of orexin (hypocretin) would lead to:
a) Malnutrition
b) Neoplasm
c) Narcolepsy
d) Diabetes
e) Agitation
12) Which of the following short-term regulators that decrease eating responds to fat
entering the duodenum?
a) Peptide YY
b) Glucagon-like peptide
c) Cholecystokinin
d) Insulin
13) A patient with Prader-Willi is found to have high levels of Ghrelin. What affect does
this have?
a) Hunger
b) Double vision
c) Easy bruising
d) Hyperthyroidism
e) Hypogonadism
14) Which of the following would NOT occur in the presence of Leptin?
a) Decreased NPY and AGRP
b) POMC inhibition
c) CRH increase
d) SNS increase
e) Insulin decrease
15) The Zucker Diabetic Fatty (ZDF) rat has which of the following disorders?
a) Childhood over-nutrition to increase fat cells
b) Hypothalamic lesion
c) Hypophyseal tumor
d) MCR gene mutation
e) Leptin receptor mutation
16) Which of the following is the recommended primary obesity treatment?
a) Physical exercise
b) Amphetamines
c) Altering lipid metabolism
d) Gastric bypass
e) Gastric banding
17) A mutation in POMC would cause which of the following?
a) Increased food intake and weight gain
b) Increased food intake but weight loss
c) Decreased food intake and weight loss
Endocrine 05May2009
DO NOT DISTRIBUTE - 23 -
d) Decreased food intake but weight gain
18) What is the mechanism by which an AGRP gene mutation decreases food intake?
a) Antagonizes MCR-3 and MCR-4 receptors
b) Cause release of alpha-MSH
c) SNS activation to the nucleus tractus solitarius
d) Decreases POMC activation
e) Leptin antagonism
19) What is the mechanism by which an NPY gene mutation decreases food intake?
a) Antagonizes MCR-3 and MCR-4 receptors
b) Cause release of alpha-MSH
c) SNS activation to the nucleus tractus solitarius
d) Decreases POMC activation
e) Leptin antagonism
Endocrine #7 – Physiology: Growth
1) Which of the following would lead to an increase in growth hormone (GH)?
a) Caloric restriction
b) Increased amino acids
c) Increased glucose or free fatty acids
d) Exercise or stress
e) Ghrelin
2) Which of the following inhibits GH by acting on GHRH?
a) Somatostatin
b) Cortisol
c) Old-age
d) Obesity
e) Pregnancy
3) A woman presents with acromegaly and breast discharge. Which of the following
hormones would likely be high?
a) Prolactin
b) Oxytocin
c) ACTH
d) GH
e) ADH
4) The JAK-STAT pathway involves taking dimerized GH and transcribing it to IGF-1
(insulin-like growth factor). Where does this mainly occur?
a) Liver
b) Long bone
c) Pituitary
d) Kidney
e) Hypothalamus
5) Which of the following is a function of IGF-1, not GH?
a) Gluconeogenesis
b) Bone and tissue growth
c) Insulin resistance
d) Protein synthesis
Endocrine 05May2009
DO NOT DISTRIBUTE - 24 -
e) Lipolysis
6) Which of the following promotes the closure of long bone epiphyseal plates?
a) GH
b) IGF-1
c) Estrogen & testosterone
d) Insulin & thyroid hormones
7) Which of the following would NOT occur in an adult with high levels of growth
hormone?
a) Prominent jaw
b) Prominent brow
c) Large hands and feet
d) Increase height
e) Large nose
8) Hypothyroidism in infancy would lead to what growth effects?
a) Developmental retardation
b) Large long bones
c) Leg length discrepancies
d) Prominent jaw and nose
e) Organomegaly, especially heart
9) Which of the following would occur in a patient with congenital growth hormone
receptor dysfunction?
a) Dwarfism with low GH levels and secondary sex characteristics present
b) Dwarfism with high GH levels and secondary sex characteristics present
c) Dwarfism with low GH levels and no secondary sex characteristics
d) Dwarfism with high GH levels and no secondary sex characteristics
10) Which of the following would be seen in a patient with excess growth hormone?
a) Hypoglycemia and hypergonadism
b) Hyperglycemia and hypergonadism
c) Hypoglycemia and hypogonadism
d) Hyperglycemia and hypogonadism
11) A 42-year-old male visits a local physician for a physical exam. He reports that this is
his first physical in 5 years, and that he is concerned about changes in his appearance
over the last couple years. He explains that his feet and hands have grown larger, his jaw
is protruding, and his face is “looking strange.” He reports slight difficulty breathing,
frequent headaches, fatigue and weakness, impotence, and excessive sweating.
Laboratory tests indicate elevated insulin-like growth factor-1 (IGF-1), and an MRI scan
revealed a 1.5 cm mass on the anterior pituitary. Further physical examination and
laboratory tests may indicate:
a) Decreased hypothalamic somatostatin secretion
b) Decreased protein synthesis
c) Hypoglycemia
d) Insulin resistance
e) Muscle atrophy
12) A 25-year-old male presents to his physician for an annual physical. He was
previously diagnosed with a growth hormone (GH)-secreting tumor on his anterior
Endocrine 05May2009
DO NOT DISTRIBUTE - 25 -
pituitary, for which he is taking a somatostatin analog. Laboratory tests, however, reveal
elevated GH levels. Further laboratory tests would most likely reveal:
a) Decreased gluconeogenesis
b) Increased lipolysis
c) Increased long bone growth
d) Increased insulin action
e) Decreased insulin levels
13) A young adult female patient visits her physician for a physical exam. The physician
observes distinct changes in her appearance that suggest excess growth hormone (GH)
action. Her feet and hands appear larger than expected, and her jaw protrudes more than
normal. Laboratory tests indicate elevated insulin-like growth factor-1 (IGF-1), and an
MRI scan revealed a 1.5 cm mass on the anterior pituitary. Further physical examination
and laboratory tests may indicate:
a) Decreased circulating free fatty acids
b) Decreased hypothalamic somatostatin secretion
c) Decreased lean muscle mass
d) Increased lipogenesis
e) Hyperglycemia
14) A physician notices abnormal growth patterns in a young male child during an annual
physical. The child was severely below normal on standardized growth tests for both
height and weight. All other aspects of the physical examination were unremarkable.
Laboratory tests indicate normal thyroxine and triiodothyronine and decreased levels of
insulin-like growth factor-1. Further laboratory tests would indicate:
a) A decrease in insulin-like growth factor binding proteins (IGFBP)
b) A decrease in thyroid stimulating hormone (TSH)
c) An increase in growth hormone (GH) secretion
d) An increase in IGFBP
e) An increase in thyroid stimulating hormone (TSH)
15) An elderly male presents to his physician to discuss with his physician a new drug
that he saw on the television. He reported that he was always in very good shape but is
now “soft and flabby.” He explained that he saw a commercial about human growth
hormone (GH) administration that was described as a “fountain of youth” and he wanted
to see if he could begin taking it. Laboratory tests revealed that the patient had reduced
GH levels, and the doctor agreed to prescribe the GH replacement for him. The most
likely effect of GH replacement would be:
a) Decreased collagen synthesis
b) Decreased lipolysis
c) Decreased protein synthesis
d) Increased glucose uptake in muscle and adipose
e) Increased plasma glucose
Endocrine #8 – Physiology: Endocrine Pancreas
1) Which of the following cells secrete amylin and pancreastatin?
a) Alpha cells
b) Beta cells
c) Delta cells
Endocrine 05May2009
DO NOT DISTRIBUTE - 26 -
d) PP cells
2) C peptide has a long serum half-life so it can be helpful in determining past levels of
which of the following?
a) Insulin
b) Glucagon
c) Somatostatin
d) Pancreatic polypeptide
e) Glucose
3) Which of the following cells is the first to receive arterial blood supply?
a) Alpha cells
b) Beta cells
c) Delta cells
4) Which of the following inhibits insulin?
a) Eating
b) Glucagon
c) Intestinal hormones
d) Glucose
e) Leptin
5) Which of the following describes the insulin response associated with glucose
injection?
a) Monophasic with an initial rapid increase due to intestinal hormones
b) Monophasic with a gradual increase due to intestinal hormones
c) Biphasic with an initial rapid increase due to intestinal hormones and a
secondary increase due to beta cells
d) Biphasic with an initial rapid increase due to beta cells and a secondary
increase due to intestinal hormones
e) Monophasic with a gradual increase due to beta cells
6) Insulin causes the insertion of what glucose transporter, leading to glucose crossing
membranes to be converted to glycogen in adipose tissue and skeletal muscle?
a) GLUT1
b) GLUT2
c) GLUT3
d) GLUT4
e) GLUT5
7) Insulin causes a decrease in which of the following within the liver?
a) Glycogenolysis
b) Glycolysis
c) Glycogenesis
d) Protein synthesis
e) Lipogenesis
8) Which of the following stimulates glucagon secretion?
a) Somatostatin
b) Insulin
c) Hypoglycemia
d) Secretin
9) What is the major action of glucagon in the pancreas?
Endocrine 05May2009
DO NOT DISTRIBUTE - 27 -
a) Increased glycogenolysis
b) Gluconeogenesis
c) Increased insulin
d) Lipolysis
e) Ketogenesis
10) Which of the following would cause the insulin to glucagon ratio to increase the most
(>10.0 from normal of 2.0?
a) Pure protein meal
b) Carbohydrate loading
c) Pure fat meal
d) Prolonged exercise
e) Fasting
11) Glucagon-like peptide-1 (GLP-1) is released from the intestines and has what action?
a) Increase insulin and increase glucagon
b) Increase insulin and decrease glucagon
c) Decrease insulin and increase glucagon
d) Decrease insulin and decrease glucagon
12) Which of the following is an action of somatostatin?
a) Inhibits insulin
b) Inhibits glucagon
c) Inhibits GI motility and secretions
d) Inhibits glucose and triglyceride absorption
e) All of the above
13) Which of the following is regulated by hypoglycemia, inhibited by hyperglycemia,
and is used diagnostically as a marker for an islet cell tumor?
a) Insulin
b) Glucagon
c) Somatostatin
d) Pancreatic polypeptide
e) Glucose
14) Which of the following is NOT a symptom of Type-1 (insulin dependent) or Type-2
(non-insulin dependent) diabetes mellitus?
a) Weight gain
b) Polydipsia
c) Polyuria
d) Increased food consumption
15) Which of the following is associated with Type-1 diabetes more so than Type-2?
a) Genetic component
b) Obesity as a cause
c) Native American race
d) Autoimmune reaction
e) Sulfonylurea drugs for treatment
16) A middle aged male presented to his physician for an annual physical. Routine testing
revealed elevated plasma glucose levels. Further laboratory testing revealed elevated
insulin levels and severely elevated glucagon levels. Following an MRI, he was
Endocrine 05May2009
DO NOT DISTRIBUTE - 28 -
diagnosed with a pancreatic !-cell tumor. The elevated glucose levels in this patient is
most likely due to:
a) Decreased glycogenolysis
b) Decreased lipogenesis
c) Increased gluconeogenesis
d) Increased lipolysis
e) Increased hepatic glycogenesis
17) A 37-year-old pregnant woman visits her physician for a prenatal visit. She was in
her 13th week of gestation and had been healthy with a normal pregnancy thus far.
Urinalysis revealed elevated glucose levels and blood tests revealed elevated insulin. She
has no history of diabetes, although her mother developed Type 2 (non-insulin
dependent) diabetes later in life. Her elevated glucose levels are most likely due to:
a) Autoimmune pancreatic alpha cell destruction
b) Autoimmune pancreatic beta cell destruction
c) Elevated growth hormone (GH) secretion
d) Increased carbohydrate ingestion
e) Insulin resistance
18) An experiment was designed to determine the effects of a high-fat diet on
carbohydrate metabolism. Rats in group A were
fed a special formulated diet high in fat, and rats in
group B were fed an isocaloric low-fat diet. These
diets were fed to the rats for 2 weeks and a fasting
glucose tolerance test was performed. The
following graph illustrates the results for group A
(Time 0 is glucose administration). The condition
that would most likely produce these results is:
a) Excess glucagon release
b) Increased glycogenesis
c) Increased glycogenolysis
d) Type-1 (insulin dependent) diabetes mellitus
e) Type-2 (noninsulin dependent) diabetes mellitus
Endocrine #9 – Pharmacology: Hypothalamic & Pituitary Hormones
1) Which of the following is located directly superior to the sella turcica?
a) Hypothalamus
b) Pituitary
c) Pons
d) Cingulate gyrus
e) Medulla
2.1) Which of the following hormones released from the anterior pituitary does NOT
stimulate the production of hormones by a peripheral endocrine gland or the liver?
a) FSH
b) LH
c) ACTH
d) TSH
e) PRL
Endocrine 05May2009
DO NOT DISTRIBUTE - 29 -
2.2) Posterior pituitary hormones (Oxytocin, ADH) are released from neurons that arise
from what area?
a) Preoptic nucleus
b) Supraoptic nucleus
c) Paraventricular nucleus
d) Cingulate gyrus
e) Solitary nucleus
3.1) Dopamine from the hypothalamus inhibits which of the following?
a) Growth hormone (GH, somatotropin)
b) Thyroid-stimulating hormone (TSH)
c) Adrenocorticotropin (ACTH)
d) Follicle-stimulating hormone (FSH)
e) Luteinizing hormone (LH)
f) Prolactin (PRL)
3.2) Which hypothalamic hormone is used rarely to differentiate Cushing disease from
ectopic secretion?
a) Growth hormone-releasing hormone (GHRH)
b) Thyrotropin-releasing hormone (TRH, protirelin)
c) Corticotropin-releasing hormone (CRH)
d) Gonadotropin-releasing hormone (GnRH)
e) Dopamine
4) Which of the following conditions for which human growth hormone (hGH) treatment
is indicated is considered the most controversial?
a) Prader-Willi syndrome
b) Turner syndrome
c) Idiopathic short stature
d) None of the above
5) A patient with infertility is undergoing fertility treatment in order to conceive. On day
three of her menstrual cycle, she is started on daily injections of an FSH preparation,
which continue for about 7-12 days. Since exogenous gonadotropins are being used to
stimulate follicular development, they are administered in conjunction with either a
continuous GnRH agonist (down-regulates receptors) or a GnRH antagonist. Why?
a) To prevent a luteal surge
b) To reduce endometrial thickness
c) To increase FSH levels
d) To reduce serum estradiol levels
e) To cause rapid ovulation
6) Which of the following are D2 agonists and thus are useful for hyperprolactinemia?
a) Fenoldopam
b) Bromocriptine and cabergoline
c) Metoclopramide, droperidol, and domperidone
d) Clozapine, risperidone, and olanzapine
e) Amoxapine
7) A post-partum woman is having continual uterine bleeding. Which of the following
could be given to help reduce hemorrhage?
a) Oxytocin
Endocrine 05May2009
DO NOT DISTRIBUTE - 30 -
b) Dopamine agonist
c) ADH (vasopressin)
d) Prolactin (PRL)
e) Luteinizing hormone (LH)
8) Antidiuretic hormone (ADH, vasopressin) may be indicated in which of the following
patients?
a) Diabetes mellitus type-1
b) Diabetes mellitus type-2
c) Diabetes insipidus
d) Hypertension
e) Galactorrhea
Endocrine #10 – Pharmacology: Thyroid & Antithyroid Drugs
1.1) Iodine peroxidase, which converts I- to I2, occurs in what location?
a) Thyroid gland membrane
b) Within the thyroid gland
c) Blood
d) Peripheral tissues
1.2) Amiodarone, corticosteroids, beta-blockers, and ipodate affect thyroid hormone
formation at what location?
a) Thyroid iodine transporter
b) Thyroid peroxidase enzyme
c) Thyroid T3/T4 proteolysis to blood
d) T4 to T3 conversion in peripheral tissues
e) B & C
1.3) Thioamides affect thyroid hormone formation at what location?
a) Thyroid iodine transporter
b) Thyroid peroxidase enzyme
c) Thyroid T3/T4 proteolysis to blood
d) T4 to T3 conversion in peripheral tissues
e) B & C
1.4) Iodides affect thyroid hormone formation at what location?
a) Thyroid iodine transporter
b) Thyroid peroxidase enzyme
c) Thyroid T3/T4 proteolysis to blood
d) T4 to T3 conversion in peripheral tissues
e) B & C
2) Levothyroxine (Synthroid, Levoxyl) is the preparation of choice for thyroid
replacement. Which of the following is NOT true of levothyroxine?
a) Short half-life
b) Low drug cost
c) Lack of allergenic foreign protein
d) Uniformity and stability
e) Easy lab measurement of serum levels
3) All thioamines have a simple molecular structure in common. What is the name of this
structure (shown)?
Endocrine 05May2009
DO NOT DISTRIBUTE - 31 -
a) Thioacetamide
b) Thionamide
c) Triiodothyronine
d) Thiocarbamide
e) Thioacetamide
4) An elderly patient is receiving levothyroxine and having their serum TSH and free
thyroxine monitored. Which of the following signs or symptoms may signify an adverse
effect to the medication?
a) Weight gain
b) Bradycardia
c) Heart palpitations
d) Cold intolerance
e) Somnolence
5) A patient with known hypothyroidism presents in a near-coma condition with edema,
bradycardia, and low core body temperature (myxedema coma). Which of the following
would be considered appropriate treatment?
a) Oral levothyroxine, 50-100mcg (low dose)
b) Oral levothyroxine, 300-400mcg (high dose)
c) IV levothyroxine, 50-100mcg (low dose)
d) IV levothyroxine, 300-400mcg (high dose)
e) IV fluids and glucose only
6) A non-pregnant patient with Graves disease is being examined for treatment options.
What is the preferred drug for this patient, which requires 12-18 months of therapy?
a) Desmopressin (DDAVP)
b) Fludrocortisone (Florinef)
c) Levothyroxine (Synthroid)
d) Methimazole (Tapazole)
e) Octreotide (Sandostatin)
Endocrine #11 – Pharmacology: Corticosteroids & Antagonists
1.1) Deficiency of the enzyme 17alpha-hydroxylase would prevent the formation of
which of the following and have what effect on blood pressure?
a) No aldosterone or cortisol, hypotension
b) No aldosterone or cortisol, hypertension
c) No cortisol or sex hormones (estradiol/testosterone), hypotension
d) No cortisol or sex hormones (estradiol/testosterone), hypertension
1.2) Deficiency of the enzyme 21-hydroxylase would prevent the formation of which of
the following and have what effect on blood pressure?
a) No aldosterone or cortisol, hypotension
b) No aldosterone or cortisol, hypertension
c) No cortisol or sex hormones (estradiol/testosterone), hypotension
d) No cortisol or sex hormones (estradiol/testosterone), hypertension
1.3) Deficiency of the enzyme 11beta-hydroxylase would prevent the formation of which
of the following and have what effect on blood pressure?
a) No aldosterone or cortisol, hypotension
b) No aldosterone or cortisol, hypertension
Endocrine 05May2009
DO NOT DISTRIBUTE - 32 -
c) No cortisol or sex hormones (estradiol/testosterone), hypotension
d) No cortisol or sex hormones (estradiol/testosterone), hypertension
2) Which of the following describes the hierarchy of anti-inflammatory effects for the
included drugs?
a) Hydrocortisone > Prednisone > Triamcinolone > Dexamethasone
b) Triamcinolone > Dexamethasone > Hydrocortisone > Prednisone
c) Dexamethasone > Triamcinolone > Prednisone > Hydrocortisone
d) Hydrocortisone > Triamcinolone > Dexamethasone > Prednisone
e) Dexamethasone > Hydrocortisone > Prednisone > Triamcinolone
3) Which of the following describes the hierarch of salt-retaining efficacy for the
included drugs?
a) Fludrocortisone > Hydrocortisone > Triamcinolone
b) Fludrocortisone > Triamcinolone > Hydrocortisone
c) Hydrocortisone > Fludrocortisone > Triamcinolone
d) Hydrocortisone > Triamcinolone > Fludrocortisone
e) Triamcinolone > Hydrocortisone > Fludrocortisone
4) In the fasting state, glucocorticoids stimulate phosphoenolypyruvate, carboxykinase,
and G6P. What major effect does this have?
a) Increased urine amino acids
b) Increased serum amino acids
c) Decreased gluconeogenesis
d) Increased gluconeogenesis
e) Decreased glycogen synthesis
5) Which of the following would occur in serum (blood) after injection of a short-acting
glucocorticoid?
a) Increased neutrophils and increased lymphocytes
b) Increased neutrophils and decreased lymphocytes
c) Decreased neutrophils and increased lymphocytes
d) Decreased neutrophils and decreased lymphocytes
6) A defect in 21beta-hydroxylase would prevent formation of cortisol, which leads to a
compensatory increase in ACTH. This causes the adrenal gland to become hyperplastic
and secrete large amounts of precursors. What effect would this have?
a) Hypotension
b) Hypokalemia
c) Salt wasting
d) Alopecia
e) Virilization
7) Cushing disease (ACTH-secreting pituitary adenoma) can be treated with metyrapone
(Metopirone) or which of the following?
a) Desmopressin (DDAVP)
b) Ketoconazole (Nizoral)
c) Levothyroxine (Synthroid)
d) Methimazole (Tapazole)
e) Somatotropin (Humatrope)
Endocrine 05May2009
DO NOT DISTRIBUTE - 33 -
8) Mifepristone (RU-486) is a progesterone antagonist with some anti-glucocorticoid
properties at higher doses. Which of the following is describes the mechanism by which
mifepristone acts with blockage of glucocorticoid receptors?
a) Stabilizes the Hsp-glucocorticoid receptor complex
b) Inhibits dissociation of the RU-486-bound glucocorticoid receptor from the
Hsp chaperone proteins
c) Alters the interaction of the glucocorticoid receptor with coregulators, favoring
the formation of a transcriptionally inactive complex
d) All of the above
Endocrine #12 – Pharmacology: Gonadal Hormones & Inhibitors
1.1) Which of the following does NOT peak at day 14 of a 28-day menstrual cycle?
a) LH
b) FSH
c) Estradiol
d) Progesterone
1.2) At which of the following days during a 28-day menstrual cycle would endometrial
lining be the thickest?
a) 7
b) 14
c) 21
d) 28
2) Which of the following steroid reactions is NOT correct?
a) Androstenedione can convert to testosterone without an enzyme
b) Testosterone requires aromatase to create estradiol
c) Estradiol requires aromatase to create estrone
d) Androstenedione requires aromatase to create estrone
e) Testosterone can convert to androstenedione without an enzyme
3) What effect does FSH have on the follicle of the mature ovary?
a) Stimulates growth of ovarian follicles and estrogen secretion
b) Stimulates growth of ovarian follicles and progesterone secretion
c) Stimulates recession of ovarian follicles and estrogen secretion
d) Stimulates recession of ovarian follicles and progesterone secretion
4) Estrogen (estradiol) receptors are found in the nucleus in what stable conformation?
a) Bound to estrogen response elements (EREs)
b) Bound to heat shock protein (Hsp90)
c) As a dimer (estradiol-estradiol)
d) Bound to estrone
e) In an aromatic ring-form pattern
5) According to the Women’s Health Initiative (WHI), estrogen treatment or hormone
replacement therapy (HRT) in peri- and post-menopausal women can increase the risk of
cardiovascular disease as well as which of the following?
a) Increased risk of Alzheimer disease
b) Negative effect on circulating lipids
c) Increased risk of breast cancer
d) Increased risk of colon cancer
Endocrine 05May2009
DO NOT DISTRIBUTE - 34 -
e) Negative effect on circulating lipoproteins
6) What is the claimed advantage of third-generation synthetic progestins (19-not, 13-
ethyl), including desogestrel, gestodene, and norgestimate, when compared with older
synthetic progestins?
a) Less effective gonadotropin inhibitors
b) Less estrogenic activity
c) More estrogenic activity
d) Less androgenic activity
e) More androgenic activity
7) Which of the following is an adverse effect of oral contraceptives (estrogenic
compounds), which has increased risk in smokers over aged 35?
a) Bleeding disorders
b) Small cell lung cancer
c) Duodenal ulcers
d) Rheumatic fever
e) Venous embolism
8) Which of the following post-coital contraceptives is considered abortifacient?
a) Conjugate estrogen
b) Ethinyl estradiol
c) Mifepristone
d) Diethylstilbestrol
e) Norgestrel
9.1) What is the role of LH on the testis?
a) Promote sperm maturation
b) Promote testosterone synthesis
c) Promote testosterone secretion
d) A & B
e) B & C
9.2) What is the role of FSH on the testis?
a) Promote sperm maturation
b) Promote testosterone synthesis
c) Promote testosterone secretion
d) A & B
e) B & C
10) In the skin, prostate, and seminal vesicles, testosterone is converted to which of the
following?
a) DHEA
b) DHT
c) ACTH
d) Cortisol
e) Estradiol
11) Which of the following is NOT correct regarding the dosing regimen of androgens in
cases of pubertal hypogonadism?
a) Long-acting agents such as testosterone enanthate or cypionate are used
b) Doses are 50mg IM injection
Endocrine 05May2009
DO NOT DISTRIBUTE - 35 -
c) Initial doses are every 4-weeks, then every 3-weeks, then every 2-weeks until
maturation is complete
d) Adult doses are 200mg at 4-week intervals
12.1) Finasteride (Proscar) inhibits the production of DHT by inhibiting what enzyme?
a) 17alpha-hydroxylase
b) Aromatase
c) 11beta-hydroxylase
d) 5alpha-reductase
e) 21-hydroxylase
12.2) Flutamide (Eulexin) has which of the following effects?
a) Agonist at the androgen receptor
b) Competitive antagonist at the androgen receptor
c) Inhibitor of 5alpha-reductase
d) Activator of 5alpha-reductase
Endocrine #13 – Pharmacology: Pancreatic Hormones & Antidiabetic Drugs
1.1) Which of the following is found in beta cells of the pancreas and regulates insulin
release as well as other aspects of glucose homeostasis?
a) GLUT1
b) GLUT2
c) GLUT3
d) GLUT4
e) GLUT5
1.2) Which of the following is found in muscle and adipose tissue and functions in
insulin-mediated uptake of glucose?
a) GLUT1
b) GLUT2
c) GLUT3
d) GLUT4
e) GLUT5
2) Which of the following describes the hierarchy of time to peak action for the included
insulin formations?
a) Lispro insulin > Regular insulin > NPH > Glargine insulin
b) Regular insulin > Lispro insulin > NPH > Glargine insulin
c) NPH > Glargine insulin > Lispro insulin > Regular insulin
d) Regular insulin > Glargine insulin > Lispro insulin > NPH
e) Glargine insulin > NPH > Regular insulin > Lispro insulin
3) How do sulfonylurea drugs affect insulin release?
a) Blocks GLUT2
b) Depolarizes (closes) K+ channels
c) Polarizes (opens) K+ channels
d) Depolarizes (opens) Ca++ channels
e) Polarizes (closes) Ca++ channels
f) Blocks insulin exocytosis
4) Why are children under the age of seven exempt from the American Diabetes
Association’s Benefits of Tight Glycemic Control of Diabetes guidelines?
Endocrine 05May2009
DO NOT DISTRIBUTE - 36 -
a) Risk of thromboembolic events
b) Risk of brain damage due to hypoglycemia
c) Risk of neuropathy due to hyperglycemia
d) Risk of renal failure due to glycemic fluctuations
e) Risk of permanent bone mass loss
5) Which of the following is NOT an alternative to insulin syringe injections currently
available for Type-1 diabetics?
a) Subcutaneous insulin infusion devices
b) Portable pen injectors
c) Ocular insulin drops
d) Inhaled insulin
6) Risk is most reduced in what system for patients who adhere to the guidelines for
Benefits of Tight Glycemic Control of Diabetes?
a) Neurologic
b) Gastrointestinal
c) Renal
d) Reproductive
e) Cardiovascular
7) Which of the following is NOT a symptom of hypoglycemia?
a) Difficulty speaking
b) Tachycardia
c) Convulsions
d) Pinpoint pupils (miosis)
e) Headache
8) When are alpha-glucosidase inhibitors indicated?
a) Initial therapy for Type-1 diabetics
b) Adjunctive therapy for Type-1 diabetics
c) Initial therapy for Type-2 diabetics
d) Adjunctive therapy for Type-2 diabetics
e) Never due to their side-effect profile
9) Which of the following describes the hierarchy of maximal duration of action for the
included sulfonylureas?
a) Tolbutamide > Glimepiride > Chlorpropamide
b) Glimepiride > Chlorpropamide > Tolbutamide
c) Chlorpropamide > Glimepiride > Tolbutamide
d) Chlorpropamide > Tolbutamide > Glimepiride
e) Tolbutamide > Chlorpropamide > Glimepiride
10) Which of the following is the safest sulfonylurea for elderly diabetics and why?
a) Chlorpropamide due to its long half-life
b) Chlorpropamide due to its short half-life
c) Tolbutamide due to its long half-life
d) Tolbutamide due to its short half-life
e) Glimepiride due to its long half-life
11) Which of the following is NOT a typical reason for secondary failure (failure to
maintain a good response to sulfonylurea over the long term) in patients on sulfonylurea
therapy?
Endocrine 05May2009
DO NOT DISTRIBUTE - 37 -
a) Reduction in physical activity
b) Decline in lean body mass
c) Decline in fat in chronic type-2 diabetes
d) Decrease in B-cell mass
12) Why should meglitinides (repaglinide) be avoided as adjunctive treatment in patients
taking a sulfonylurea drug?
a) Their binding sites overlap
b) They bind to each other forming a non-soluble product
c) They bind to each other, which inactivates the sulfonylurea agent
d) They bind to each other, which inactivates the meglitinide agent
e) They can be used safely in combination
13) Which of the following is NOT a postulated mechanism of action for biguanides?
a) Reduced hepatic and renal gluconeogenesis
b) Inhibition of pancreatic beta cells
c) Slowing of glucose absorption from GI tract with increased glucose to lactate
conversion by enterocytes
d) Direct stimulation of glycolysis in tissues with increased glucose removal from
blood
e) Reduction of plasma glucagon levels
14) Thiazolidinediones (Tzds) bind to peroxisome proliferator-activated receptor-gamma
(PPAR-g) receptors in muscle, fat, and liver. Which of the following is NOT a receptor
involved in gene activation by Tzds?
a) Lipid metabolism
b) Insulin signal tranduction
c) Adipose tissue differentiation
d) Glucose metabolism
e) Glucagon signal tranduction
15) Which of the following is NOT a postulated mechanism of action for exenatide?
a) Central loss of appetite
b) Potentiating of glucose-mediated insulin secretion
c) Enhanced gastric emptying
d) Suppression of postprandial glucagon release
Endocrine #14 – Pharmacology: Agents That Affect Bone Mineral Homeostasis
1) Which of the following reactions is inhibitory?
a) Serum calcium/phosphate to bone via vitamin D
b) Serum calcium/phosphate to bone via PTH
c) Bone to serum calcium/phosphate via vitamin D
d) Bone to serum calcium/phosphate via PTH
e) Bone to serum calcium/phosphate via calcitonin
2) What affect do bisphosphonates have on farnesyl-pyrophosphate synthase?
a) Inhibitory leading to inhibited osteoclast function
b) Inhibitory leading to enhanced osteoclast function
c) Stimulatory leading to inhibited osteoclast function
d) Stimulatory leading to enhanced osteoclast function
Endocrine 05May2009
DO NOT DISTRIBUTE - 38 -
3) Which of the following is a selective estrogen receptor modulator (SERM) approved
for the treatment of osteoporosis?
a) Octreotide (Sandostatin)
b) Alendronate (Fosemax)
c) Raloxifene (Evista)
d) Ergocalciferol (Calciferol)
e) Calcitonin (Calcimar)
4) What advantage does teriparatide have over fluoride in the treatment of osteoporosis?
a) Reduced incidence of hypocalcemia
b) Reduced incidence of hypercalcemia
c) Reduced incidence of CNS symptoms
d) Reduced incidence of bone fractures
e) Reduced incidence of prolonged bleeding
5) Which of the following is a bisphosphonate approved for osteoporosis?
a) Octreotide (Sandostatin)
b) Alendronate (Fosemax)
c) Raloxifene (Evista)
d) Ergocalciferol (Calciferol)
e) Calcitonin (Calcimar)
6) Bisphosphonates may be administered via IV or orally. If administered orally, which
of the following protocols should be followed?
a) Take on an empty stomach without any fluids
b) Take on an empty stomach with water and stay upright
c) Take with food and avoid staying still for prolonged periods
d) Take with milk only or a lactose-like liquid for vegan patients
e) Take with a full glass of red wine or shot of 80-proof liquor
7) Which of the following drugs is used for osteoporosis and stimulates new bone
formation, rather than inhibiting bone resorption?
a) Reveromycin A
b) Indomethacin
c) Diphenylhydantoin
d) Teriparatide
e) Alendronate
Endocrine #15 – Pathology
1) Which of the following is more characteristic of Type-2 diabetes mellitus compared
with Type-1 pancreatic morphologic changes?
a) Leukocyte infiltration of the islets
b) Great reduction in islet cell number and size
c) Amyloid replacement of islet cells
d) Beta cell degranulation
2.1) What is the number one complication for patients with diabetes mellitus?
a) Neuropathy
b) Foot gangrene
c) Atherosclerosis
d) Glomerulosclerosis
Endocrine 05May2009
DO NOT DISTRIBUTE - 39 -
e) Hyperlipidemia
2.2) What is the most common cause of death in diabetics?
a) Myocardial infarction
b) Stroke
c) Renal failure
d) Kimmelstein-Wilson lesions
e) Hypertensive crisis
2.3) Which of the following complications of diabetes is caused by hypertension?
a) Diabetic macrovascular disease
b) Hyaline arteriosclerosis
c) Diabetic microangiopathy
d) Diabetic nephropathy
e) Diffuse mesangial sclerosis
f) Nodular glomerulosclerosis
2.4) Which of the following involves PAS-positive lesions which may trap mesangial
cells (Kimmelstein-Wilson lesions) and if present is essentially pathognomonic for
diabetes?
a) Diabetic macrovascular disease
b) Hyaline arteriosclerosis
c) Diabetic microangiopathy
d) Diabetic nephropathy
e) Diffuse mesangial sclerosis
f) Nodular glomerulosclerosis
2.5) Which of the following complications of diabetes is responsible for the disease-
defining neuropathy, nephropathy, and retinopathy?
a) Diabetic macrovascular disease
b) Hyaline arteriosclerosis
c) Diabetic microangiopathy
d) Diabetic nephropathy
e) Diffuse mesangial sclerosis
f) Nodular glomerulosclerosis
2.6) Which of the following is NOT characteristic of Type-1 diabetes mellitus?
a) Ketoacidosis
b) Polyuria
c) Polydipsia
d) Polyphagia
e) Obesity
2.7) Which of the following describes the type of coma seen in dehydrated Type-2
diabetes mellitus patients?
a) Hypo-osmolar, non-ketonic
b) Hyper-osmolar, non-ketonic
c) Hypo-osmolar, ketonic
d) Hyper-osmolar, ketonic
2.8) What is the first sign of diabetic nephropathy in Type-2 diabetics?
a) Failure to concentrate urine
b) Subtle amounts of albumin in urine
Endocrine 05May2009
DO NOT DISTRIBUTE - 40 -
c) Dark, strong smelling urine
d) High LDL and TG blood levels
e) Low HDL blood levels
3.1) A patient presents with confusion and stupor after running a marathon. Testing
shows a blood glucose level below 50mg/dL. The patient’s friend says these symptoms
have occurred before during marathon training. Pancreatic imaging shows small,
encapsulated, solitary tumors. Which of the following is most likely?
a) Beta-cell tumor (insulinoma)
b) Zollinger-Ellison syndrome
c) Alpha-cell tumor (glucagonoma)
d) Delta-cell tumor (somatostatinoma)
e) VIPoma
3.2) A patient presents with steatorrhea, hypochlorhydria, cholelithiasis, and diabetes
mellitus. Which of the following is most likely?
a) Beta-cell tumor (insulinoma)
b) Zollinger-Ellison syndrome
c) Alpha-cell tumor (glucagonoma)
d) Delta-cell tumor (somatostatinoma)
e) VIPoma
3.3) A patient with a history of multiple endocrine neoplasia type 1 (MEN-1, Wermer
syndrome) presents with peptic ulcers and diarrhea. Testing shows tumor metastasis.
Which of the following is most likely?
a) Beta-cell tumor (insulinoma)
b) Zollinger-Ellison syndrome
c) Alpha-cell tumor (glucagonoma)
d) Delta-cell tumor (somatostatinoma)
e) VIPoma
3.4) A post-menopausal woman presents with a skin rash. Testing shows anemia and a
glucose level suggestive of diabetes mellitus. Which of the following is most likely?
a) Beta-cell tumor (insulinoma)
b) Zollinger-Ellison syndrome
c) Alpha-cell tumor (glucagonoma)
d) Delta-cell tumor (somatostatinoma)
e) VIPoma
3.5) Which of the following is associated with watery diarrhea, hypokalemia, and
achlorhydria (WDHA syndrome)?
a) Beta-cell tumor (insulinoma)
b) Zollinger-Ellison syndrome
c) Alpha-cell tumor (glucagonoma)
d) Delta-cell tumor (somatostatinoma)
e) VIPoma
4.1) A young woman is being treated in the emergency department for psychosis and is
given haloperidol and reserpine. After developing galactorrhea, the patient admits to not
having her period in some time (amenorrhea). A pregnancy test is negative. After a CT
scan of the brain returns, the clinician opts to avoid invasive prodecures but speculates
she would have found dystrophic calcification. Which of the following is most likely?
Endocrine 05May2009
DO NOT DISTRIBUTE - 41 -
a) Somatotrophic adenoma
b) Prolactinoma
c) Corticotrophic adenoma
d) Empty sella syndrome
e) Sheehan syndrome
4.2) A patient presents with a history of Cushing syndrome which was resolved after
removal of the adrenal glands. CT scan shows an anterior lobe pituitary adenoma that has
grown since the operation (Nelsons syndrome). A post-mortem biopsy of the tumor
shows densely granulated and PAS-positive staining. Which of the following is most
likely?
a) Somatotrophic adenoma
b) Prolactinoma
c) Corticotrophic adenoma
d) Empty sella syndrome
e) Sheehan syndrome
4.3) An adult presents with acromegaly and a broad face. Testing shows high levels of
serum GH and IGF-1. An oral load of glucose is administered, but serum GH levels are
still high. A CT scan shows a large pituitary adenoma of the anterior lobe. What is the
mechanism by which this tumor arose?
a) Defect in p53 gene causing inability to force cell apoptosis
b) Defect in GTPase of the alpha-subunit of G-protein (Gs)
c) Defect in bcl-2 and bak genes
d) Defect in proto-oncogenes bcl-2 and c-myc
e) Gain on function mutation resulting in cAMP overproduction
5) During parturition, a woman develops obstetrical hemorrhage, which is controlled by
the hospital staff. Shortly after birth, the woman develops hypotension, starts losing her
pubic hair, and is unable to breast feed. CT scanning of the head shows likely ischemic
necrosis in the sella turcica. Which of the following is most likely?
a) Somatotrophic adenoma
b) Prolactinoma
c) Corticotrophic adenoma
d) Empty sella syndrome
e) Sheehan syndrome
6) An obese woman with hypertension presents with complains of visual defects. Testing
shows hyperprolactinemia. CT scan shows herniation of the arachnoid matter and
cerebrospinal fluid into the sella turcica. Which of the following is most likely?
a) Somatotrophic adenoma
b) Craniopharyngioma
c) Corticotrophic adenoma
d) Empty sella syndrome
e) Sheehan syndrome
7) A child presents with poor peripheral vision. A CT scan shows a cystic, multi-
lobulated tumor above the sella turcica. Surgical removal is performed and the tumor is
sent to pathology. Testing shows dystrophic calcification and cysts lined with squamous
epithelium. The cysts are filled with a yellow, viscous fluid that is rich in cholesterol
Endocrine 05May2009
DO NOT DISTRIBUTE - 42 -
crystals. The tumor likely developed from vestigial remnants of Rathkes pouch. Which of
the following is most likely?
a) Somatotrophic adenoma
b) Craniopharyngioma
c) Corticotrophic adenoma
d) Empty sella syndrome
e) Sheehan syndrome
8) An ICU patient with recent head trauma is found to be creating excess urine. Testing
of the urine shows a low specific gravity. Serum testing shows elevated sodium. The
patient complains of being very thirsty. ADH deficiency is suspected. Which of the
following is most likely?
a) Central diabetes insipidus
b) Nephrogenic diabetes insipidus
c) Type-1 diabetes mellitus
d) Type-2 diabetes mellitus
e) Syndrome of inappropriate ADH secretion (SIADH)
9) A patient presents with shortness of breath. Testing reveals high ADH levels. Chest x-
ray reveals small cell carcinoma of the lung. If SIADH is suspected, which of the
following would be expected on labs and during physical exam?
a) Hypernatremia and edema
b) Hyponatremia and edema
c) Hypernatremia without edema
d) Hyponatremia without edema
10) Which of the following is a sign of hypothyroidism?
a) Lateral eyebrow hair loss
b) Patient always feels hot (hyperpyrexia)
c) Excessive perspiration
d) Palpitations, tachycardia
e) High systolic pressure with normal diastolic pressure (high pulse pressure)
11) Which of the following is a sign of hyperthyroidism (e.g. thyrotoxicosis)?
a) Retardation and deficient growth (cretinism)
b) Puffy eyelids, narrow palpebral fissure, protruded tongue
c) Fine-amplitude hand tremors
d) Cold intolerance and scaly dermatitis
e) Deftness, night blindness, somnolence, dulled reflexes
12) Which of the following would NOT cause symptoms similar to hypothyroidism?
a) Thyroid hormone resistance syndrome
b) Autoimmune (Hashimoto) thyroiditis
c) Use of lithium, iodides, or p-aminosalicyclic acid
d) Excessive iodine
e) Pituitary failure
13) A middle-aged woman from a third world country presents complaints of feeling
tired. History reveals Sjögren syndrome. Physical exam reveals an enlarged, non-nodular
thyroid. Labs reveal euthyroid. Biopsy shows inflammatory destruction of thyroid
follicles and Hürthle cells (large eosinophilic cells showing oxyphilic metaplasia of
thyroid follicular cells). Which of the following is most likely?
Endocrine 05May2009
DO NOT DISTRIBUTE - 43 -
a) Subacute (DeQuervain) thyroiditis
b) Primary hyperthyroidism
c) Graves disease
d) Lymphocytic (Hashimoto) thyroiditis
e) Primary hypothyroidism
14) A patient presents with jaw and neck pain with swallowing. History reveals a recent
influenza infection. Exam reveals hoarseness and an exquisitely tender and enlarged
thyroid. A 131I-uptake test shows decreased uptake of iodide diffusely. Although a
biopsy is not performed, granulomatous inflammation with multinucleated giant cells
would be expected as well as microabscesses with neutrophilic infiltration. Which of the
following is most likely?
a) Subacute (DeQuervain) thyroiditis
b) Primary hyperthyroidism
c) Graves disease
d) Lymphocytic (Hashimoto) thyroiditis
e) Primary hypothyroidism
15) A middle-aged female presents with agitation and hyperpyrexia. On ocular exam, the
patient appears to be wide-eyed and extremely alert. Labs reveal HLA-B8 and DR3. IgG
autoantibodies to the TSH receptor is suspected. Although a biopsy is not performed, a
scalloped “moth-eaten” thyroid appearance would be expected. Which of the following is
most likely?
a) Subacute (DeQuervain) thyroiditis
b) Primary hyperthyroidism
c) Graves disease
d) Lymphocytic (Hashimoto) thyroiditis
e) Primary hypothyroidism
16) A patient from central Asia presents with an enormous, multinodular neck mass.
History reveals a diet rich in cabbage, turnips, and Brussel sprouts. Biopsy reveals the
plunging goiter is very rick in colloid material. Which of the following demographics is
most likely for this patient?
a) Young boy
b) Young girl
c) Elderly man
d) Elderly woman
e) Pregnant woman
17) What is the hallmark of a follicular adenoma?
a) Painless neck mass with dysphagia
b) Hürthle cells with eosinophilic granular cytoplasm
c) Well-formed capsule around the tumor
d) Multinodular with a capsule that is penetrated by tissue
e) Overproduction of cAMP with signs of malignancy
18.1) Which of the following carcinomas of the thyroid gland is derived from
parafollicular or C cells and has a strong association with MEN-IIa and IIb?
a) Papillary carcinoma
b) Follicular carcinoma
c) Medullary carcinoma
Endocrine 05May2009
DO NOT DISTRIBUTE - 44 -
d) Anaplastic carcinoma
18.2) A 30-year-old woman presents with complaints of a neck lump. Physical exam
reveals a mass in the cervical lymph node. A solitary thyroid nodule is found and is freely
moveable. Resection of the mass reveals papillae (stalks covered by cuboidal epithelium)
and psammoma bodies. Microscopy shows empty cells and ground glass or Orphan
Annie eye nuclei. Which of the following is most likely?
a) Papillary carcinoma
b) Follicular carcinoma
c) Medullary carcinoma
d) Anaplastic carcinoma
18.3) A 65-year-old patient presents with a history of multinodular goiter. Testing reveals
a highly anaplastic undifferentiated neoplasm with giant cells. What is the prognosis for
this patient?
a) 0% rate of mortality
b) 25% rate of mortality
c) 50% rate of mortality
d) 75% rate of mortality
e) 100% rate of mortality
18.4) A 50-year-old woman presents with a thyroid carcinoma that has traveled
hematogenously to the bones of the shoulder, pelvis, sternum, and skull. Which of the
following is most likely?
a) Papillary carcinoma
b) Follicular carcinoma
c) Medullary carcinoma
d) Anaplastic carcinoma
19.1) Which of the following is the most common cause of hyperparathyroidism?
a) MEN-1 inactivation on chromosome 11
b) MEN-2 tyrosine kinase receptor mutation
c) Familial hypocalcinuric hypercalcemia (FHH) due to CASR gene mutation
d) Parathyroid adenoma due to relocation of PRAD1 proto-oncogene, over-
expressing cyclin D1 protein
19.2) Which of the following affects all four parathyroid glands and results in water-clear
cells?
a) Parathyroid adenoma
b) Parathyroid hyperplasia
c) Parathyroid carcinoma
19.3) A 45-year-old woman presents with bone pain, abdominal pain, and depression.
Testing reveals nephrolithiasis and elevated PTH levels. Which of the following is most
likely?
a) Hypercalcemia of malignancy
b) Vitamin D toxicity
c) Thiazide diuretic use
d) Hyperparathyroidism
e) Granulomatous disease
20) A hospitalized patient is found to have chronic depression of serum calcium levels.
Parathyroid gland analysis reveals hyperplasia with increased chief cells and water-clear
Endocrine 05May2009
DO NOT DISTRIBUTE - 45 -
cells. Further testing reveals bone changes and metastatic calcification (calciphylaxis).
What is the most common cause of this patient’s secondary hyperparathyroidism?
a) Vitamin D deficiency
b) Inadequate intake of calcium
c) Renal failure
d) Steatorrhea
21) A patient with DiGeorge syndrome undergoes surgery to remove lymph nodes in the
neck. Long after the surgery, the patient develops emotional instability and Parkinson-
like movements. Physical exam reveals papilledema and cataracts. Testing reveals
prolonged QT interval and calcification of the basal ganglion. Tapping on the facial nerve
induces ipsilateral muscle contraction (Chvostek sign). Blood pressure cuff occlusion
reveals carpal spasm (Trousseau sign), which disappears as soon as the cuff is removed.
Which of the following is most likely?
a) Hyperthyroidism
b) Hypothyroidism
c) Hyperparathyroidism
d) Hypoparathyroidism
e) Pseudohypoparathyroidism
f) Pseudopseudohypoparathyroidism
22.1) Pseudohypoparathyroidism is due to end-organ resistance to the actions of PTH,
thus PTH levels may be normal or elevated. This is due to G,alpha-mediated pathway
expression in the kidney of GNAS1 gene. Which of the following describes
pseudohypoparathyroidism type-1A? AHO = Albright hereditary osteodysrophy
a) AHO with multi-hormone resistance and maternal allele mutation
b) AHO with multi-hormone resistance and paternal allele mutation
c) AHO without multi-hormone resistance and maternal allele mutation
d) AHO without multi-hormone resistance and paternal allele mutation
22.2) Which of the following describes pseudopseudohypoparathyroidism?
a) AHO with multi-hormone resistance and maternal allele mutation
b) AHO with multi-hormone resistance and paternal allele mutation
c) AHO without multi-hormone resistance and maternal allele mutation
d) AHO without multi-hormone resistance and paternal allele mutation
23.1) Which of the following is the most common cause of Cushing syndrome?
a) Iatrogenic (exogenous corticosteroid administration)
b) Adrenal (hypersecreting cortisol adenoma)
c) Paraneoplastic (ACTH secretion from small cell lung cancer)
d) Pituitary (ACTH anterior pituitary tumor)
23.2) Which of the following forms of Cushing syndrome would have high serum cortisol
and low ACTH, and is also called “independent Cushing syndrome”?
a) Iatrogenic (exogenous corticosteroid administration)
b) Adrenal (hypersecreting cortisol adenoma)
c) Paraneoplastic (ACTH secretion from small cell lung cancer)
d) Pituitary (ACTH anterior pituitary tumor)
23.3) Which of the following forms of Cushing syndrome would have high serum ACTH
and cortical hyperplasia?
a) Iatrogenic (exogenous corticosteroid administration)
Endocrine 05May2009
DO NOT DISTRIBUTE - 46 -
b) Adrenal (hypersecreting cortisol adenoma)
c) Paraneoplastic (ACTH secretion from small cell lung cancer)
d) Pituitary (ACTH anterior pituitary tumor)
23.4) Which of the following would occur in Cushing syndrome caused by small cell
cancer of the lung?
a) Increased serum ACTH and cortisol
b) Decreased serum ACTH and cortisol
c) Increased ACTH and decreased cortisol
d) Decreased ACTH and increased cortisol
e) No change in serum ACTH or corisol
23.5) A patient presents with central trunk obesity, a “buffalo hump,” moon facies,
muscle wasting, hirsutism, and hypertension. A 24-hours urine free cortisol level suggests
Cushing syndrome. Dexamethasone is given to the patient and ACTH levels are
suppressed. Which of the following forms is most likely?
a) Iatrogenic (exogenous corticosteroid administration)
b) Adrenal (hypersecreting cortisol adenoma)
c) Paraneoplastic (ACTH secretion from small cell lung cancer)
d) Pituitary (ACTH anterior pituitary tumor)
24) After finding high levels of serum aldosterone and low levels of serum renin,
hyperaldosteronism is suspected. Dexamethasone is found to suppress aldosterone
production, so an adrenal CT is performed, which finds an adenoma (Conn syndrome).
Excision reveals bright-yellow, lipid-laden cortical cells with spironolactone inclusion
bodies. Which of the following would also be expected in this patient?
a) Hypotension and hypokalemia
b) Hypotension and hyperkalemia
c) Hypertension and hypokalemia
d) Hypertension and hyperkalemia
25) A female newborn is found to have increased androgens, decreased cortisol, and
increased ACTH secretion. She develops virilization (e.g. hirsutism) and bilateral
hyperplasia of the adrenal glands (congenital hyperplasia). Which of the following
enzymes is most likely deficient?
a) 17alpha-hydroxylase
b) Aromatase
c) 11beta-hydroxylase
d) 5alpha-reductase
e) 21-hydroxylase
26) Which of the following would NOT cause adrenocortical insufficiency?
a) Acute hemorrhagic necrosis (Waterhouse-Friderichsen syndrome)
b) Congenital adrenal hyperplasia
c) Adrenoleukodystrophy (ALD gene on Xq28)
d) Autoimmune poly-endocrinopathy syndrome
e) AIDS, infection, fungi, tuberculosis
27) A patient presents with photophobia, fatigue, and neck pain. Testing and exam
reveals severe hypotension, purpura, and disseminated intravascular coagulation (DIC).
Antibiotics are started immediately in hopes of preventing massive bilateral adrenal
Endocrine 05May2009
DO NOT DISTRIBUTE - 47 -
hemorrhage (Waterhouse-Friderichsen syndrome). What bacterium is most likely
responsible?
a) Streptococcus agalactiae
b) Mycobacterium tuberculosis
c) Neisseria meningitidis
d) Escherichia coli
e) Listeria monocytogenes
28) A patient presents with progressive weakness and easy fatigability. They complain of
GI disturbances and have hyperpigmentation. Labs show hypokalemia, hyponatremia,
and volume depletion. Exam reveals hypotension. Further testing reveals shrunken
adrenal glands. The clinician suspects that the adrenal glands have lost over 90% of their
function. Which of the following is most likely?
a) Primary adrenocortical insufficiency (Addison disease)
b) Autoimmune poly-endocrinopathy syndrome
c) Adrenoleukodystrophy (ALD gene on Xq28)
d) X-linked adrenal hypoplasia (DAX-1 gene on Xp21)
e) Acute hemorrhagic necrosis (Waterhouse-Friderichsen syndrome)
29) Which of the following would NOT be seen in secondary adrenocortical
insufficiency, as aldosterone synthesis is normal?
a) Decreased ACTH levels
b) Deficiency in cortisol output
c) Deficiency in androgen output
d) Hyponatremia, hypokalemia, hyperpigmentation
e) Exogenous ACTH elevates plasma cortisol levels
30) Which of the following does NOT describes an adrenal cortical carcinoma?
a) Associated with virilism
b) Can be caused by LiFraumeni syndrome or Beckwith-Wiedemann syndrome
c) Involves a large and invasive area of necrosis and hemorrhage
d) Hyperplastic, well circumscribed, yellow (lipids), nodular lesion
e) Commonly invades the adrenal vein and inferior vena cava
31) A patient presents with palpitations and excessive sweating. Testing shows
orthostatic hypertension. History reveals dizziness when lifting weights at the gym and
sometimes (paradoxically) while defecating. Further testing shows large balls of
neoplastic chromaffin cells in the adrenal gland (zellballen). Urinalysis reveals excess
vanillylmandelic acid (VMA) due to epinephrine and norepinephrine breakdown. Where
is the tumor most likely located?
a) Adrenal cortex, zona glomerulosa
b) Adrenal cortex, zona fasciculata
c) Adrenal cortex, zona reticularis
d) Adrenal medulla
e) Extra-adrenal
32) Where is a common location to find an extramedullary paraganglioma?
a) Sternoclavicular joint
b) Carotid body
c) Cisterna chyle
d) Ligaments of the knee
Endocrine 05May2009
DO NOT DISTRIBUTE - 48 -
e) GI tract at splenic flexure
33) Which of the following patients is most likely to develop thymic hyperplasia, which
is lymphoid follicles containing mostly B cells?
a) Hashimoto thyroiditis
b) Sjögren disease
c) Multiple sclerosis
d) Myasthenia gravis
e) Pregnant diabetic
34) Thymomas typically present either due to impingement of the anterior superior
mediastinum or due to:
a) Hashimoto thyroiditis
b) Sjögren disease
c) Multiple sclerosis
d) Myasthenia gravis
e) Pregnant diabetic
35) Which of the following is NOT associated with thymoma?
a) Graves disease
b) Cushing syndrome
c) Pernicious anemia
d) Megaloblastic anemia
e) Dermatomyositis-polymyositis
f) Acquired hypogammaglobulinemia
36) An adult presents with visual disturbances, mental deterioration, and dementia-like
behavior. Head CT shows a tumor near the third ventricle that is nearly impeding the
aqueduct of Sylvius. Treatment is started to prevent internal hydrocephalus. Which of the
following is most likely?
a) Medulloblastoma
b) Retinoblastoma
c) Thymoma
d) Pineoblastoma
e) Pineocytoma
37.1) Which of the following areas is NOT affected by MEN-1 (Wermer) syndrome?
a) Pituitary
b) Parathyroid
c) Thyroid
d) Pancreatic islets
e) Adrenal gland
37.2) MEN-2A (Sipple syndrome) and MEN-2B (MEN-III) most often affect what areas?
a) Pituitary and parathyroid
b) Parathyroid and pancreatic islets
c) Pancreatic islets and adrenal
d) Adrenal and thyroid
e) Thyroid and pituitary
37.3) MEN-2B does not induce hyperparathyroidism like MEN-2A does. However, both
types involve mutations in what gene?
a) p53
Endocrine 05May2009
DO NOT DISTRIBUTE - 49 -
b) MEN-1
c) RET
d) Bcl-2
e) c-myc
Endocrine #16 – Clinical: Hypothalamic Pituitary Disorders
1) Which of the following is NOT a common cause of hypopituitarism?
a) Radiotherapy
b) Autoimmunity
c) Pituitary tumors
d) Pituitary operations
2) Which of the following is NOT a symptom of gonadotropin deficiency in women?
a) Amenorrhea
b) Loss of libido
c) Dyspareunia
d) Hirsutism
e) Vaginal dryness
3) Which of the following is NOT a symptom of gonadotropin deficiency in men?
a) Loss of libido
b) Erectile dysfunction
c) Gynecomastia
d) Infertility
e) Hypertrophy of the testes
4) Which of the following would NOT be low (or inappropriately normal) in
hypopituitarism?
a) TSH
b) FSH
c) LH
d) Estradiol
e) Testosterone
5) Which of the following hormone abnormalities would be expected in supporting a
diagnosis of central hypothyroidism?
a) Increased serum FT4 and increased serum TSH
b) Increased serum FT4 and decreased serum TSH
c) Decreased serum FT4 and increased serum TSH
d) Decreased serum FT4 and decreased serum TSH
6) Which of the following is a recently developed test to stimulate growth hormone
secretion?
a) Tryptophan plus GHRH
b) Arginine plus GHRH
c) Alanine plus GHRH
d) Aspartate plus GHRH
e) Glutamine plus GHRH
7) What hormone replacement is essential in ACTH deficiency?
a) Glucocorticoid replacement (Hydrocortisone)
b) Levothroxine sodium (Synthroid)
Endocrine 05May2009
DO NOT DISTRIBUTE - 50 -
c) Testosterone (IM or transdermal patch)
d) Somatotropin (Humatrope)
e) Oxytocin (Pitocin)
8) What is the drug of choice for TSH deficiency?
a) Glucocorticoid replacement (Hydrocortisone)
b) Levothroxine sodium (Synthroid)
c) Testosterone (IM or transdermal patch)
d) Somatotropin (Humatrope)
e) Oxytocin (Pitocin)
9) For women who do not desire fertility, what is the hormone replacement therapy used
for gonadotropin deficiency?
a) Estrogen therapy
b) Progesterone therapy
c) Gonadotropin therapy
d) A & B
e) B &C
10) What is the hormone replacement therapy used in males with gonadotropin
deficiency?
a) Estrogen therapy
b) Progesterone therapy
c) Testosterone therapy
d) FSH therapy
e) LH therapy
Endocrine #17 – Clinical: Pituitary Tumors
1) What tumor size distinguishes a microadenoma from a macroadenoma?
a) 1cm
b) 2cm
c) 3cm
d) 4cm
e) 5cm
2) Which of the following is NOT a key question to ask when evaluating a pituitary
tumor?
a) Is the tumor causing a local mass effect?
b) Is hypopituitarism present?
c) Is there evidence of hormone excess?
d) Is the patient pregnant and at risk for Sheehan syndrome?
3) Which of the following is NOT a neurological defect that can occur with a pituitary
tumor?
a) Visual field and acuity defects (superior tumor extension)
b) Hypothalamic syndrome (superior tumor extension)
c) Diplopia (lateral tumor extension)
d) Parkinson-like movements (lateral tumor extension)
e) CSF rhinorrhea (inferior tumor extension)
4) Which of the following hyperfunctioning pituitary tumors is the most common?
a) Prolactinomas
14240896 quiz-endocrine
14240896 quiz-endocrine
14240896 quiz-endocrine
14240896 quiz-endocrine
14240896 quiz-endocrine
14240896 quiz-endocrine
14240896 quiz-endocrine
14240896 quiz-endocrine
14240896 quiz-endocrine
14240896 quiz-endocrine
14240896 quiz-endocrine
14240896 quiz-endocrine
14240896 quiz-endocrine
14240896 quiz-endocrine
14240896 quiz-endocrine
14240896 quiz-endocrine
14240896 quiz-endocrine
14240896 quiz-endocrine
14240896 quiz-endocrine
14240896 quiz-endocrine
14240896 quiz-endocrine
14240896 quiz-endocrine
14240896 quiz-endocrine
14240896 quiz-endocrine
14240896 quiz-endocrine
14240896 quiz-endocrine
14240896 quiz-endocrine
14240896 quiz-endocrine
14240896 quiz-endocrine
14240896 quiz-endocrine
14240896 quiz-endocrine
14240896 quiz-endocrine
14240896 quiz-endocrine
14240896 quiz-endocrine

Mais conteúdo relacionado

Mais procurados

2 amino acids mcq
2 amino acids mcq2 amino acids mcq
2 amino acids mcq
dream10f
 
2nd year MBBS anatomy mcqs with explanation
2nd year MBBS anatomy mcqs with explanation2nd year MBBS anatomy mcqs with explanation
2nd year MBBS anatomy mcqs with explanation
Farhan Ali
 

Mais procurados (20)

MCQs on CNS physiology
MCQs on CNS physiologyMCQs on CNS physiology
MCQs on CNS physiology
 
2 amino acids mcq
2 amino acids mcq2 amino acids mcq
2 amino acids mcq
 
CNS MCQs
CNS MCQsCNS MCQs
CNS MCQs
 
MCQs on Cardiovascular system physiology
MCQs on Cardiovascular system physiologyMCQs on Cardiovascular system physiology
MCQs on Cardiovascular system physiology
 
042 physiology mcq_acem_primary_nervous_system
042 physiology mcq_acem_primary_nervous_system042 physiology mcq_acem_primary_nervous_system
042 physiology mcq_acem_primary_nervous_system
 
Anatomy mcqs thorax
Anatomy mcqs thoraxAnatomy mcqs thorax
Anatomy mcqs thorax
 
Anatomy KMU Mcqs...2.docx
Anatomy KMU Mcqs...2.docxAnatomy KMU Mcqs...2.docx
Anatomy KMU Mcqs...2.docx
 
MCQs Neurotransmitters And Neuropeptides
MCQs Neurotransmitters And NeuropeptidesMCQs Neurotransmitters And Neuropeptides
MCQs Neurotransmitters And Neuropeptides
 
Physiology mcqs
Physiology mcqs Physiology mcqs
Physiology mcqs
 
Bicohemistry quiz
Bicohemistry quizBicohemistry quiz
Bicohemistry quiz
 
CNS.physio2.pdf
CNS.physio2.pdfCNS.physio2.pdf
CNS.physio2.pdf
 
Anatomy KMU Mcqs...1.docx
Anatomy KMU Mcqs...1.docxAnatomy KMU Mcqs...1.docx
Anatomy KMU Mcqs...1.docx
 
Anatomy abdomen Mcqs
Anatomy abdomen McqsAnatomy abdomen Mcqs
Anatomy abdomen Mcqs
 
Anatomy mcqs
Anatomy mcqsAnatomy mcqs
Anatomy mcqs
 
Biochemistry Questions pool with Answer Key.docx
Biochemistry Questions pool with Answer Key.docxBiochemistry Questions pool with Answer Key.docx
Biochemistry Questions pool with Answer Key.docx
 
Pathology Practice Examination
Pathology Practice ExaminationPathology Practice Examination
Pathology Practice Examination
 
Physiology
Physiology Physiology
Physiology
 
Neet pg -mcq's renal physiology
Neet pg -mcq's renal physiologyNeet pg -mcq's renal physiology
Neet pg -mcq's renal physiology
 
2nd year MBBS anatomy mcqs with explanation
2nd year MBBS anatomy mcqs with explanation2nd year MBBS anatomy mcqs with explanation
2nd year MBBS anatomy mcqs with explanation
 
A revision and quiz on heme synthesis and degradation
A revision and quiz on heme synthesis and degradationA revision and quiz on heme synthesis and degradation
A revision and quiz on heme synthesis and degradation
 

Semelhante a 14240896 quiz-endocrine

Luteinizing Hormone Function
Luteinizing Hormone FunctionLuteinizing Hormone Function
Luteinizing Hormone Function
quicksweet
 
B.Sc.(Micro+Biotech) II Animal & Plant Physiology Unit 3.2 Endocrine System
B.Sc.(Micro+Biotech) II Animal & Plant Physiology Unit 3.2 Endocrine SystemB.Sc.(Micro+Biotech) II Animal & Plant Physiology Unit 3.2 Endocrine System
B.Sc.(Micro+Biotech) II Animal & Plant Physiology Unit 3.2 Endocrine System
Rai University
 
Lecture 25 the endocrine system
Lecture 25    the endocrine systemLecture 25    the endocrine system
Lecture 25 the endocrine system
Mahesh Thakur
 
Best Hormones and the Endocrine System-1.ppt
Best  Hormones and the Endocrine System-1.pptBest  Hormones and the Endocrine System-1.ppt
Best Hormones and the Endocrine System-1.ppt
AnthonyMatu1
 
Best Hormones and the Endocrine System-1.ppt
Best  Hormones and the Endocrine System-1.pptBest  Hormones and the Endocrine System-1.ppt
Best Hormones and the Endocrine System-1.ppt
AnthonyMatu1
 

Semelhante a 14240896 quiz-endocrine (20)

Physiology. 4 th semester kmu
Physiology. 4 th semester kmuPhysiology. 4 th semester kmu
Physiology. 4 th semester kmu
 
SIPPFINALREVIEW
SIPPFINALREVIEWSIPPFINALREVIEW
SIPPFINALREVIEW
 
Document from lama
Document from lamaDocument from lama
Document from lama
 
Internal medicine exams Document from lama
Internal medicine exams Document from lamaInternal medicine exams Document from lama
Internal medicine exams Document from lama
 
final_exam.pdf
final_exam.pdffinal_exam.pdf
final_exam.pdf
 
Luteinizing Hormone Function
Luteinizing Hormone FunctionLuteinizing Hormone Function
Luteinizing Hormone Function
 
Adrenal Gland Lecture (2).ppt
Adrenal Gland Lecture (2).pptAdrenal Gland Lecture (2).ppt
Adrenal Gland Lecture (2).ppt
 
Thyroid gland
Thyroid glandThyroid gland
Thyroid gland
 
B.Sc.(Micro+Biotech) II Animal & Plant Physiology Unit 3.2 Endocrine System
B.Sc.(Micro+Biotech) II Animal & Plant Physiology Unit 3.2 Endocrine SystemB.Sc.(Micro+Biotech) II Animal & Plant Physiology Unit 3.2 Endocrine System
B.Sc.(Micro+Biotech) II Animal & Plant Physiology Unit 3.2 Endocrine System
 
Endocrinology
EndocrinologyEndocrinology
Endocrinology
 
Lecture 25 the endocrine system
Lecture 25    the endocrine systemLecture 25    the endocrine system
Lecture 25 the endocrine system
 
Endocrine system
Endocrine systemEndocrine system
Endocrine system
 
Endocrinology simplified
Endocrinology simplifiedEndocrinology simplified
Endocrinology simplified
 
ENDOCRINE SYSTEM
ENDOCRINE SYSTEMENDOCRINE SYSTEM
ENDOCRINE SYSTEM
 
Hartifact
HartifactHartifact
Hartifact
 
Hormonal cascade system involving hypothalamus
Hormonal cascade system involving hypothalamusHormonal cascade system involving hypothalamus
Hormonal cascade system involving hypothalamus
 
- The Endocrine System.ppt
-  The Endocrine System.ppt-  The Endocrine System.ppt
- The Endocrine System.ppt
 
Endocrine system
Endocrine systemEndocrine system
Endocrine system
 
Best Hormones and the Endocrine System-1.ppt
Best  Hormones and the Endocrine System-1.pptBest  Hormones and the Endocrine System-1.ppt
Best Hormones and the Endocrine System-1.ppt
 
Best Hormones and the Endocrine System-1.ppt
Best  Hormones and the Endocrine System-1.pptBest  Hormones and the Endocrine System-1.ppt
Best Hormones and the Endocrine System-1.ppt
 

Último

Call Girl in Indore 8827247818 {LowPrice} ❤️ (ahana) Indore Call Girls * UPA...
Call Girl in Indore 8827247818 {LowPrice} ❤️ (ahana) Indore Call Girls  * UPA...Call Girl in Indore 8827247818 {LowPrice} ❤️ (ahana) Indore Call Girls  * UPA...
Call Girl in Indore 8827247818 {LowPrice} ❤️ (ahana) Indore Call Girls * UPA...
mahaiklolahd
 
🌹Attapur⬅️ Vip Call Girls Hyderabad 📱9352852248 Book Well Trand Call Girls In...
🌹Attapur⬅️ Vip Call Girls Hyderabad 📱9352852248 Book Well Trand Call Girls In...🌹Attapur⬅️ Vip Call Girls Hyderabad 📱9352852248 Book Well Trand Call Girls In...
🌹Attapur⬅️ Vip Call Girls Hyderabad 📱9352852248 Book Well Trand Call Girls In...
Call Girls In Delhi Whatsup 9873940964 Enjoy Unlimited Pleasure
 

Último (20)

Mumbai ] (Call Girls) in Mumbai 10k @ I'm VIP Independent Escorts Girls 98333...
Mumbai ] (Call Girls) in Mumbai 10k @ I'm VIP Independent Escorts Girls 98333...Mumbai ] (Call Girls) in Mumbai 10k @ I'm VIP Independent Escorts Girls 98333...
Mumbai ] (Call Girls) in Mumbai 10k @ I'm VIP Independent Escorts Girls 98333...
 
Model Call Girls In Chennai WhatsApp Booking 7427069034 call girl service 24 ...
Model Call Girls In Chennai WhatsApp Booking 7427069034 call girl service 24 ...Model Call Girls In Chennai WhatsApp Booking 7427069034 call girl service 24 ...
Model Call Girls In Chennai WhatsApp Booking 7427069034 call girl service 24 ...
 
Most Beautiful Call Girl in Bangalore Contact on Whatsapp
Most Beautiful Call Girl in Bangalore Contact on WhatsappMost Beautiful Call Girl in Bangalore Contact on Whatsapp
Most Beautiful Call Girl in Bangalore Contact on Whatsapp
 
Call Girls Service Jaipur {9521753030 } ❤️VVIP BHAWNA Call Girl in Jaipur Raj...
Call Girls Service Jaipur {9521753030 } ❤️VVIP BHAWNA Call Girl in Jaipur Raj...Call Girls Service Jaipur {9521753030 } ❤️VVIP BHAWNA Call Girl in Jaipur Raj...
Call Girls Service Jaipur {9521753030 } ❤️VVIP BHAWNA Call Girl in Jaipur Raj...
 
Independent Call Girls Service Mohali Sector 116 | 6367187148 | Call Girl Ser...
Independent Call Girls Service Mohali Sector 116 | 6367187148 | Call Girl Ser...Independent Call Girls Service Mohali Sector 116 | 6367187148 | Call Girl Ser...
Independent Call Girls Service Mohali Sector 116 | 6367187148 | Call Girl Ser...
 
Low Rate Call Girls Bangalore {7304373326} ❤️VVIP NISHA Call Girls in Bangalo...
Low Rate Call Girls Bangalore {7304373326} ❤️VVIP NISHA Call Girls in Bangalo...Low Rate Call Girls Bangalore {7304373326} ❤️VVIP NISHA Call Girls in Bangalo...
Low Rate Call Girls Bangalore {7304373326} ❤️VVIP NISHA Call Girls in Bangalo...
 
Manyata Tech Park ( Call Girls ) Bangalore ✔ 6297143586 ✔ Hot Model With Sexy...
Manyata Tech Park ( Call Girls ) Bangalore ✔ 6297143586 ✔ Hot Model With Sexy...Manyata Tech Park ( Call Girls ) Bangalore ✔ 6297143586 ✔ Hot Model With Sexy...
Manyata Tech Park ( Call Girls ) Bangalore ✔ 6297143586 ✔ Hot Model With Sexy...
 
Independent Call Girls In Jaipur { 8445551418 } ✔ ANIKA MEHTA ✔ Get High Prof...
Independent Call Girls In Jaipur { 8445551418 } ✔ ANIKA MEHTA ✔ Get High Prof...Independent Call Girls In Jaipur { 8445551418 } ✔ ANIKA MEHTA ✔ Get High Prof...
Independent Call Girls In Jaipur { 8445551418 } ✔ ANIKA MEHTA ✔ Get High Prof...
 
Trichy Call Girls Book Now 9630942363 Top Class Trichy Escort Service Available
Trichy Call Girls Book Now 9630942363 Top Class Trichy Escort Service AvailableTrichy Call Girls Book Now 9630942363 Top Class Trichy Escort Service Available
Trichy Call Girls Book Now 9630942363 Top Class Trichy Escort Service Available
 
Call Girls Service Jaipur {9521753030} ❤️VVIP RIDDHI Call Girl in Jaipur Raja...
Call Girls Service Jaipur {9521753030} ❤️VVIP RIDDHI Call Girl in Jaipur Raja...Call Girls Service Jaipur {9521753030} ❤️VVIP RIDDHI Call Girl in Jaipur Raja...
Call Girls Service Jaipur {9521753030} ❤️VVIP RIDDHI Call Girl in Jaipur Raja...
 
Call Girl in Indore 8827247818 {LowPrice} ❤️ (ahana) Indore Call Girls * UPA...
Call Girl in Indore 8827247818 {LowPrice} ❤️ (ahana) Indore Call Girls  * UPA...Call Girl in Indore 8827247818 {LowPrice} ❤️ (ahana) Indore Call Girls  * UPA...
Call Girl in Indore 8827247818 {LowPrice} ❤️ (ahana) Indore Call Girls * UPA...
 
Premium Bangalore Call Girls Jigani Dail 6378878445 Escort Service For Hot Ma...
Premium Bangalore Call Girls Jigani Dail 6378878445 Escort Service For Hot Ma...Premium Bangalore Call Girls Jigani Dail 6378878445 Escort Service For Hot Ma...
Premium Bangalore Call Girls Jigani Dail 6378878445 Escort Service For Hot Ma...
 
🌹Attapur⬅️ Vip Call Girls Hyderabad 📱9352852248 Book Well Trand Call Girls In...
🌹Attapur⬅️ Vip Call Girls Hyderabad 📱9352852248 Book Well Trand Call Girls In...🌹Attapur⬅️ Vip Call Girls Hyderabad 📱9352852248 Book Well Trand Call Girls In...
🌹Attapur⬅️ Vip Call Girls Hyderabad 📱9352852248 Book Well Trand Call Girls In...
 
Call Girls Vasai Virar Just Call 9630942363 Top Class Call Girl Service Avail...
Call Girls Vasai Virar Just Call 9630942363 Top Class Call Girl Service Avail...Call Girls Vasai Virar Just Call 9630942363 Top Class Call Girl Service Avail...
Call Girls Vasai Virar Just Call 9630942363 Top Class Call Girl Service Avail...
 
Call Girls Raipur Just Call 9630942363 Top Class Call Girl Service Available
Call Girls Raipur Just Call 9630942363 Top Class Call Girl Service AvailableCall Girls Raipur Just Call 9630942363 Top Class Call Girl Service Available
Call Girls Raipur Just Call 9630942363 Top Class Call Girl Service Available
 
The Most Attractive Hyderabad Call Girls Kothapet 𖠋 9332606886 𖠋 Will You Mis...
The Most Attractive Hyderabad Call Girls Kothapet 𖠋 9332606886 𖠋 Will You Mis...The Most Attractive Hyderabad Call Girls Kothapet 𖠋 9332606886 𖠋 Will You Mis...
The Most Attractive Hyderabad Call Girls Kothapet 𖠋 9332606886 𖠋 Will You Mis...
 
Premium Call Girls In Jaipur {8445551418} ❤️VVIP SEEMA Call Girl in Jaipur Ra...
Premium Call Girls In Jaipur {8445551418} ❤️VVIP SEEMA Call Girl in Jaipur Ra...Premium Call Girls In Jaipur {8445551418} ❤️VVIP SEEMA Call Girl in Jaipur Ra...
Premium Call Girls In Jaipur {8445551418} ❤️VVIP SEEMA Call Girl in Jaipur Ra...
 
Coimbatore Call Girls in Thudiyalur : 7427069034 High Profile Model Escorts |...
Coimbatore Call Girls in Thudiyalur : 7427069034 High Profile Model Escorts |...Coimbatore Call Girls in Thudiyalur : 7427069034 High Profile Model Escorts |...
Coimbatore Call Girls in Thudiyalur : 7427069034 High Profile Model Escorts |...
 
Call Girls Rishikesh Just Call 8250077686 Top Class Call Girl Service Available
Call Girls Rishikesh Just Call 8250077686 Top Class Call Girl Service AvailableCall Girls Rishikesh Just Call 8250077686 Top Class Call Girl Service Available
Call Girls Rishikesh Just Call 8250077686 Top Class Call Girl Service Available
 
Jogeshwari ! Call Girls Service Mumbai - 450+ Call Girl Cash Payment 90042684...
Jogeshwari ! Call Girls Service Mumbai - 450+ Call Girl Cash Payment 90042684...Jogeshwari ! Call Girls Service Mumbai - 450+ Call Girl Cash Payment 90042684...
Jogeshwari ! Call Girls Service Mumbai - 450+ Call Girl Cash Payment 90042684...
 

14240896 quiz-endocrine

  • 1. Endocrine 05May2009 DO NOT DISTRIBUTE - 1 - Endocrine #1 – Histology 1) Which of the following coordinates most of the endocrine functions of the body? a) Anterior pituitary b) Posterior pituitary c) Hypothalamus d) Cerebral cortex e) Autocrine cells 2.1) Which of the following hormones does not dissolve in the blood and thus requires plasma proteins or specialized carrier proteins? a) Steroid hormones b) Thyroid hormones c) Small peptides, proteins, and glycoproteins d) Catecholamines e) A & B f) C & D 2.2) Where are amino acid analogs and their derivatives (including catecholamines) mainly produced? a) Ovaries, testes, and adrenal cortex b) Hypothalamus, pituitary, and pancreas c) Thyroid and parathyroid d) Neurons and adrenal medulla e) Enteroendocrine cells of the GI tract and respiratory system 3.1) Most protein hormones and catecholamines use which of the following second messenger systems for initiating the hormone-receptor interaction? a) Adenylate cyclase/camp system b) Tyrosine kinase system c) Phosphatidylinositol system d) Activation of ion channels e) None of the above (intracellular receptors) 3.2) Which of the following is NOT a second messenger? a) cAMP b) 5-HT c) DAG d) IP3 e) Ca2+ 3.3) Which of the following is NOT true of the intracellular hormone receptor system? a) Usually localized in the cell nucleus b) Is used by steroid and thyroid hormones (hydrophobic) c) Receptors contain complexes with three binding sites d) Binding causes allosteric receptor transformation that binds chromosomal DNA and activates or inhibits RNA polymerase activity e) Receptors influence gene expression but require second messengers 4.1) What portion of the pituitary gland (hypophysis) is most anterior? a) Pars distalis b) Pars intermedia c) Pars tuberalis
  • 2. Endocrine 05May2009 DO NOT DISTRIBUTE - 2 - d) Pars nervosa e) Infundibulum 4.2) What portion of the hypophysis contains neurosecretory axons forming the hypothalamo-hypophyseal tracts? a) Pars distalis b) Pars intermedia c) Pars tuberalis d) Pars nervosa e) Infundibulum 4.3) Which of the following describe the embryology of the hypophysis? a) The anterior lobe grows down from the neuroectoderm forming the Rathke pouch and the posterior lobe grows up from the oropharynx ectoderm b) The anterior lobe grows down from the neuroectoderm and the posterior lobe grows up from the oropharynx ectoderm forming the Rathke pouch c) The anterior lobe grows up from the oropharynx ectoderm forming the Rathke pouch and the posterior lobe grows down from the neuroectoderm d) The anterior lobe grows up from the oropharynx ectoderm and the posterior lobe grows down from the neuroectoderm forming the Rathke pouch e) Both the anterior and posterior lobes extend from the floor of the third ventricle 4.4) The superior hypophyseal arteries arising from the internal carotid and posterior communicating, supply the areas around and including the pituitary stalk. The inferior hypophyseal arteries, arising from the internal carotid, supply the pars nervosa. The hypothalamo-hypophyseal portal system serves to carry the neuroendocrine secretions directly to what portion of the hypophysis, without entering systemic circulation? a) Pars distalis b) Pars intermedia c) Pars tuberalis d) Pars nervosa e) Infundibulum 4.5) Nerves entering the anterior lobe of the pituitary gland have vasomotor function and arise from: a) Supraoptic nucleus b) Paraventricular nucleus c) Hypothalamo-hypophyseal tract d) Autonomic postganglionic fibers e) Autonomic preganglionic fibers Match the pars distalis hormones with the histologic cell description: 4.6) FSH & LH a) Medium sized, oval cells, round central nuclei 4.7) ACTH b) Large polygonal cells with oval nuclei 4.8) TSH c) Medium sized, polygonal cells with round eccentric nuclei 4.9) PRL d) Small oval cells with round eccentric nuclei 4.10) GH e) Large polygonal cells with round eccentric nuclei 4.11) The pars tuberalis, an extension of the anterior lobe along the pituitary stalk, often shows immunoreactivity for which of the following? a) ACTH, FSH, LH b) TSH, PRL, GH
  • 3. Endocrine 05May2009 DO NOT DISTRIBUTE - 3 - c) ACTH, GH, LH d) FSH, PRL, TSH e) LH, TSH, GH 4.12) Which of the following is a small polypeptide (4,000kDa) and stimulates secretion of glucocorticoids and gonadocorticoids by the zona fasciculata and zona reticularis? a) Growth hormone (somatotropin, GH) b) Prolactin (PRL) c) Adrenocorticotropin hormone (ACTH) d) Follicle-stimulating hormone (FSH) e) Luteinizing hormone (LH) f) Thyrotropic hormone (TSH) g) Oxytocin h) Antidiuretic hormone (ADH, vasopressin) 4.13) Which of the following stimulates spermatogenesis in the testis? a) Growth hormone (somatotropin, GH) b) Prolactin (PRL) c) Adrenocorticotropin hormone (ACTH) d) Follicle-stimulating hormone (FSH) e) Luteinizing hormone (LH) f) Thyrotropic hormone (TSH) g) Oxytocin h) Antidiuretic hormone (ADH, vasopressin) 4.14) Which of the following maintains androgen secretion by the Leydig (interstitial) cells of the testis and regulates final maturation of the ovarian follicle? a) Growth hormone (somatotropin, GH) b) Prolactin (PRL) c) Adrenocorticotropin hormone (ACTH) d) Follicle-stimulating hormone (FSH) e) Luteinizing hormone (LH) f) Thyrotropic hormone (TSH) g) Oxytocin h) Antidiuretic hormone (ADH, vasopressin) 4.15) Oxytocin stimulates ejection of milk from the glands and stimulates contraction of the smooth muscle cells in the pregnant uterus. Which of the following, like oxytocin, is produced then stored in the neurohypophysis? a) Antidiuretic hormone (ADH, vasopressin) b) Adrenocorticotropin hormone (ACTH) c) Growth hormone (somatotropin, GH) d) Follicle-stimulating hormone (FSH) e) Thyrotropic hormone (TSH) f) Luteinizing hormone (LH) g) Prolactin (PRL) 5.1) Which of the following hypothalamic regulating hormones inhibits secretion of PRL by lactotropes? a) Growth hormone-releasing hormone (GHRH) b) Dopamine
  • 4. Endocrine 05May2009 DO NOT DISTRIBUTE - 4 - c) Corticotropin-releasing hormone (CRH) d) Somatostatin e) Thyrotropin-releasing hormone (TRH) f) Gonadotropin-releasing hormone (GnRH) 5.2) Which of the following hypothalamic regulating hormones inhibits secretion of GH? a) Growth hormone-releasing hormone (GHRH) b) Dopamine c) Corticotropin-releasing hormone (CRH) d) Somatostatin e) Thyrotropin-releasing hormone (TRH) f) Gonadotropin-releasing hormone (GnRH) 5.3) Which of the following stimulates secretion of LH and FSH? a) Growth hormone-releasing hormone (GHRH) b) Dopamine c) Corticotropin-releasing hormone (CRH) d) Somatostatin e) Thyrotropin-releasing hormone (TRH) f) Gonadotropin-releasing hormone (GnRH) 6.1) Which of the following is NOT true of the pineal gland? a) Located on the posterior wall of the third ventricle b) Shaped like a flattened pinecone c) Develops from oropharynx ectoderm d) Pinealocytes have a large deeply infolded nucleus with one or more nuclei e) Functions in regulation of daily body rhythm (circadian) 6.2) Which of the following describes the release and action of melatonin? a) Darkness inhibits the production of melatonin and melatonin stimulates the neurons in the hypothalamus that produce GnRH b) Darkness inhibits the production of melatonin and melatonin inhibits the neurons in the hypothalamus that produce GnRH c) Daylight inhibits the production of melatonin and melatonin stimulates the neurons in the hypothalamus that produce GnRH d) Daylight inhibits the production of melatonin and melatonin inhibits the neurons in the hypothalamus that produce GnRH 7.1) Which of the following occurs at week 14 of development with regard to the thyroid gland? a) Colloid is produced and the gland starts working b) Thyroid hormone is lost leading to cretinism c) Follicles develop from endodermal cells d) Thyroglossal duct descends from endoderm pharynx e) Thyroglossal duct divides into two lobes at the trachea 7.2) Which of the following is NOT true? a) Thyroxine (T4) and triiodothyronine (T3) regulate cell basal metabolism and heat production b) Calcitonin (thyrocalcitonin) increase blood calcium levels c) Follicular cells produce T3 and T4 d) Parafollicular cells secrete calcitonin, the physiologic antagonist to PTH
  • 5. Endocrine 05May2009 DO NOT DISTRIBUTE - 5 - e) The thyroid follicle produces the colloid thyroglobin, which is the inactive storage form for the thyroid hormones 7.3) Which of the following is true of tetraiodothyronine (T4)? a) Formed by the addition of four iodine atoms to thyroglobulin b) Formed by oxidative coupling of four monoiodotyrosine (MITs) c) Formed by oxidative coupling of two diiodotyrosines (DITs) d) Formed by oxidative coupling of two MITs and a DIT e) Secreted in a T4:T3 ration of 1:20 8.1) Which of the following is true of the parathyroid glands? a) Superior and inferior glands develop from the third brachial pouch b) Superior and inferior glands develop from the fourth brachial pouch c) Superior glands develop from third brachial pouch and inferior from fourth d) Superior glands develop from fourth brachial pouch and inferior from third e) All four glands will receive full blood supply without the inferior thyroid artery 8.2) Which of the following is NOT true of parathyroid hormone (PTH)? a) Promotes calcium release from bone b) Stimulates calcium reabsorption at distal kidney tubule c) Increases formation of 1,25-dihydroxycholecalciferol (1,25-OH2) d) Secreted by chief (principle) cells e) Absorbed by oxyphil cells f) Reduces serum phosphate 9.1) Which of the following describes the embryonic development of the adrenal gland? a) Cortical cells arise from mesoderm; medulla cells originate from endoderm b) Cortical cells arise from neural crest; medulla cells originate from ectoderm c) Cortical cells arise from ectoderm; medulla cells originate from neural crest d) Cortical cells arise from mesoderm; medulla cells originate from neural crest e) Cortical cells arise from neural crest; medulla cells originate from endoderm 9.2) Which of the following layers of the adrenal gland does NOT receive a direct blood supply, but receives blood via sinusoids? a) Capsule b) Cortex c) Medulla Match the adrenal gland secreted hormone with the location: 9.3) Catecholamines (epinephrine) a) Zona glomerulosa 9.4) Mineralcorticoids (aldosterone) b) Zona fasciculata 9.5) Gonadocorticoids (DHEA) c) Zona reticularis 9.6) Glucocorticoids (Cortisol) d) Medullary chromaffin cells Match the function with the adrenal gland secreted hormone: 9.7) Induces weak masculinizing effect a) Mineralcorticoids 9.8) Aid in controlling electrolyte homeostasis b) Glucocorticoids 9.9) Promote normal metabolism c) Gonadocorticoids 10.1) Which of the following is formed from the ventral bud from the hepatic duct (not dorsal bud from the foregut)? a) Uncinate process b) Body c) Tail
  • 6. Endocrine 05May2009 DO NOT DISTRIBUTE - 6 - d) Anterior head 10.2) What is the function of somatostatin (from D cells)? a) Activates insulin and glucagon secretions b) Inhibits insulin and glucagon secretions c) Activates insulin secretions and inhibits glucagon secretions d) Activates glucagon secretions and inhibits insulin secretions 10.3) Which of the following is a function of insulin, secreted from beta cells? a) Stimulates gluconeogenesis b) Stimulates glycogenolysis c) Mobilizes fat d) Phosphorylation and use of glucose e) Stimulates hepatic lipase f) Increases blood glucose level 11) A patient presents to the Emergency Room after exposure to radiation from an atomic bomb. This explosion has likely created radioactive iodine. Flooding which of the following locations with potassium iodine (e.g. IOSAT pill) would help prevent side effects? a) Islet of Langerhans alpha cells b) Islet of Langerhans beta cells c) Adrenal cortex d) Thyroid gland e) Pineal gland Endocrine #2 – Physiology: Hypothalamus & Pituitary 1) Which of the following would be involved in a negative feedback signaling loop located on the same cell? a) Autocrine signaling b) Paracrine signaling c) Endocrine signaling d) Neurocrine signaling 2) The release of oxytocin to cause contractions during birth is an example of: a) Autocrine signaling b) Paracrine signaling c) Endocrine signaling d) Neurocrine signaling 3) Pancreatic beta cells secreting insulin that acts on the alpha cells is an example of: a) Autocrine signaling b) Paracrine signaling c) Endocrine signaling d) Neurocrine signaling 4) The circadian cycle stimulates hormone release (GH and cortisol) in the absence of external stimulation. This is an example of: a) Neurotransmitter stimulated hormone release b) Hormone stimulated hormone release c) Pulsatile stimulated hormone release d) Stress stimulated hormone release
  • 7. Endocrine 05May2009 DO NOT DISTRIBUTE - 7 - 5) Labs are taken from a patient every hour over a 24-hour period. The labs include cortisol from the blood and GH via binding protein. When would these values most likely be the highest? a) Night (2am) b) Morning (8am) c) Afternoon (2pm) d) Evening (8pm) 6) What GLUT transporter is inserted into target cells (muscle, adipose) when insulin is released from beta cells, which drops blood glucose and causes a negative feedback loop? a) GLUT1 b) GLUT2 c) GLUT3 d) GLUT4 e) GLUT5 7) The hypothalamus releases a thyroid-releasing hormone (TRH), which stimulates the anterior pituitary to release a trophic hormone (TSH), which affects the target gland. Which of the following describes the negative feedback loop for this process? a) Pituitary hormones reduce the effect of the hypothalamus b) Pituitary hormones reduce the effect of the hypothalamus and pituitary c) Target gland hormones reduce the effect of the pituitary d) Target gland hormones reduce the effect of the pituitary and hypothalamus e) A & D f) B & C g) B & D 8) The hypothalamus releases a cortisol-releasing hormone (CRH), which stimulates the pituitary to release ACTH, which stimulates the adrenal cortex to release cortisol. Which of the following describes the negative feedback loop for this process? a) Pituitary hormones reduce the effect of the hypothalamus b) Pituitary hormones reduce the effect of the hypothalamus and pituitary c) Adrenal cortex hormones reduce the effect of the pituitary d) Adrenal cortex hormones reduce the effect of the pituitary and hypothalamus e) A & D f) B & C g) B & D 9) If the adrenal cortex were disable such that it could not participate in the negative feedback loop, which of the following would occur? a) Decreased CRH and increased cortisol b) Decreased CRH and decreased cortisol c) Increased CRH and absence of ACTH d) Decreased ACTH e) Increased ACTH 10) If the pituitary gland were disabled such that it could not participate in the cortisol negative feedback loop, which of the following would occur? a) Increased CRH and increased cortisol b) Increased CRH and decreased cortisol c) Decreased CRH and increased cortisol
  • 8. Endocrine 05May2009 DO NOT DISTRIBUTE - 8 - d) Decreased CRH and decreased cortisol 11) A 40-year-old truck driver has had difficulty using his side mirrors for traffic behind him. He has never had any major medical problems in the past. He visits an optometrist, who determines he has bitemporal hemianopsia, but his vision is 20/20. A head CT scan reveals slight enlargement of the sella turcica. Which of the following hormones is most likely being secreted in excessive amounts in this man? a) Antidiuretic hormone b) Prolactin c) ACTH d) Growth hormone e) Luteinizing hormone 12) Which of the following targets to the anterior pituitary is inhibitory? a) CRH (corticotropin-releasing hormone) b) TRH (thyrotropin-releasing hormone) c) DA (dopamine acting on prolactin) d) PRH (prolactin-releasing hormone) 13) Hypothalamic nerve stimulation to the posterior pituitary would lead to all of the following EXCEPT: a) Water retention b) Decreased blood osmolality c) Uterine contraction d) Thyroid stimulation e) Breast milk ejection 14) A patient presents with dilute, high volume urine (polyuria) and extreme thirst (polydipsia). Which of the following is most likely? a) No ADH b) Normal ADH, low sugar c) Normal ADH, high sugar d) Excess ADH 15) A 20-pack-year smoker presents with concentrated, high volume urine and expanded ICR and ECF volume. Which of the following is most likely? a) No ADH b) Normal ADH, low sugar c) Normal ADH, high sugar d) Excess ADH 16) How does the hypothalamus cause the release of anterior pituitary hormones? a) Nerve endings directly connect to anterior pituitary b) Nerve stimulation of the posterior pituitary causes hormone secretion that activates the anterior pituitary c) Nerve ending hormones release hormones directly into the anterior pituitary d) Nerve ending hormones release hormones that follow the systemic blood supply into the anterior pituitary e) Nerve ending hormones release hormones that follow the portal blood supply into the anterior pituitary 17) Which of the following hormones is derived from (created in) the posterior pituitary? a) Oxytocin
  • 9. Endocrine 05May2009 DO NOT DISTRIBUTE - 9 - b) ACTH c) TSH d) ADH e) No hormones 18) Which of the following hormones of the anterior pituitary targets the liver? a) ACTH (adrenocorticotropic hormone) b) TSH (thyroid stimulating hormone) c) GH (growth hormone) d) FSH (follicle stimulating hormone) e) PRL (prolactin) 19) Which of the following hormones of the hypothalamic-anterior pituitary axis cause the release of FSH? a) TRH b) CRH c) GHRH d) LHRH e) PRH 20) If the connection between the hypothalamus and anterior pituitary were severed, which of the following would increase? a) TSH b) PRL c) ACTH d) LH & FSH e) GH 21) A middle-aged female patient presents to her physician complaining of weight gain, weakness, lethargy, and dizziness. She was recently diagnosed with pulmonary carcinoma and reports that her symptoms began within the last week. A MRI of the lung region revealed that her lung tumor grew rapidly over the last few weeks. Physical examination found elevated blood pressure but little to no edema. Laboratory tests indicate severely elevated antidiuretic hormone (ADH). Further laboratory tests would indicate: a) Elevated renal sodium reabsorption b) Elevated serum sodium concentration c) Increased atrial natriuretic peptide (ANP) secretion d) Polyuria e) Reduced urinary osmolality 22) A 25-year-old male presents to his physician complaining of loss of peripheral vision. Physical examination was normal, but a CT scan revealed a very large mass on the anterior pituitary. Laboratory testing could reveal: a) Elevated serum corticotropin releasing hormone (CRH) b) Elevated serum growth hormone releasing hormone (GHRH) c) Elevated serum oxytocin d) Elevated serum prolactin e) Elevated urine osmolality 23) A 45-year-old female is diagnosed with an anterior pituitary tumor. Her physician explains that the best course of treatment would be surgery, but that she would have to
  • 10. Endocrine 05May2009 DO NOT DISTRIBUTE - 10 - undergo hormone replacement post-operatively. The hormones that would most likely have to be replaced include: a) Antidiuretic hormone (ADH) b) Corticotropin releasing-hormone (CRH) c) Growth hormone releasing-hormone (GHRH) d) Oxytocin e) Thyroid stimulating hormone (TSH) Endocrine #3 – Physiology: Thyroid Hormones & Action 1) Which of the following is involved in a positive feedback loop to the hypothalamus? a) Thyroid releasing factor (TRH) b) Somatotropin release-inhibiting factor (SRIF) c) Thyroid stimulating hormone (TSH) d) Somatostatin e) T4 & T3 2) Which of the following would occur without a thyroid present? a) TRH increases and TSH increases b) TRH increase and TSH decreases c) TRH decreases and TSH increases d) TRH decreases and TSH decreases e) T4, T3, and somatostatin increase Match the thyroid disorder: T4 TSH TRH 3) Primary hypothyroidism a) Decreased Decreased Increased 4) Pituitary hypothyroidism b) Increased Increased Decreased 5) Hypothalamic hypothyroidism c) Decreased Increased Increased 6) Pituitary hyperthyroidism d) Decreased Decreased Decreased 7) Graves disease (autoimmune) e) Increased Decreased Decreased 8) Calcitonin is produced in which of the following locations? a) Follicular cells b) Colloid c) Thyroid capillary d) Parafollicular cells e) None of the above 9) Demineralization and glycosylation of thyroglobulin occurs at what cellular location? a) Mitochondria b) Smooth ER c) Rough ER d) Golgi apparatus e) Vesicle before entering colloid 10) Iodine is absorbed against an electrochemical gradient (Na+/I- symporter) and concentrated above blood levels (iodide trapping) in what location? a) Follicular cells b) Colloid c) Thyroid capillary d) Parafollicular cells e) None of the above
  • 11. Endocrine 05May2009 DO NOT DISTRIBUTE - 11 - 11) Iodide is sent to the colloid via: a) I-/Cl- exchanger (pedrin) b) Na+/I- symporter c) Na+/K+ pump d) Passive diffusion e) Active diffusion 12) What is the role of thyroid peroxidase? a) Releases iodine from thyroglobulin b) Binds iodine to thyroglobulin c) Binds two DITs to make T4 d) Binds MIT to DIT to make T3 e) Binds two iodide atoms to make iodine 13) At what location does iodide bind to tyrosine residues on thyroglobulin to form DITs or MITs, leading to a 2-3 month stored supply of T3 and T4? a) Follicular cells b) Colloid c) Thyroid capillary d) Parafollicular cells 14) Which of the following is the active form of thyroid hormone? a) T4 b) T3 c) rT3 15) When T4 diffuses into a cell and is deiodinized to T3, which of the following occurs prior to gene transcription with RNA polymerase II? a) T3 binds to a thyroid receptor b) T3 displaces a corepressor c) T3 recruits a coactivator d) A & B e) All of the above 16) Which of the following actions of thyroid hormone acts synergistically with GH? a) Increased basal O2 consumption and heat production b) Increased respiratory rate and minute ventilation c) Increased cardiac output (HR & SV) d) Increased linear bone growth e) Increased glucose absorption 17) Which of the following, along with norepinephrine, decreases with thyroid hormone action? a) Hypothalamus SNS center stimulation b) Red blood cell mass and O2 capacity c) Total peripheral resistance d) Cholesterol turnover (LDL decrease) e) Epinephrine activity (lipolysis) f) Sensory awareness and memory 18) Which of the following would occur in a patient who developed antibodies that are bound to TSH-R and competitively inhibited TSH (Hashimoto disease)? a) TSH increases
  • 12. Endocrine 05May2009 DO NOT DISTRIBUTE - 12 - b) TRH decreases c) T4 production increases d) T3 production increases 19) A patient develops antibodies that bind TSH-R and cause hyperstimulation (Graves disease). Which of the following is most likely? a) Increased T4, increased TSH, increased TRH b) Increased T4, increased TSH, decreased TRH c) Increased T4, decreased TSH, decreased TRH d) Increased T4, decreased TSH, increased TRH e) Decreased T4, increased TSH, decreased TRH 20) A 32-year-old woman complains of excessive tiredness, weight gain, constipation and breast discharge. Laboratory results indicate that serum thyroid stimulating hormone (TSH) was markedly decreased. Additional physical examination and laboratory testing would most likely reveal: a) Decreased thyrotropin-releasing hormone (TRH) b) Increased respiration rate c) Increased TRH d) Increased triiodothyronine (T3) e) Increased T4 Three patients with hypothyroidism arrive at the laboratory for a thyroid-releasing hormone (TRH) test. During this test, TRH was injected at time 0 and plasma TSH was measured for 3 hours. The following graph was created for the findings: 21) What disorder does Patient A have? a) Thyroid b) Hypothalamus c) Pituitary 22) What disorder does Patient B have? a) Thyroid b) Hypothalamus c) Pituitary 23) What disorder does Patient C have? a) Thyroid b) Hypothalamus c) Pituitary
  • 13. Endocrine 05May2009 DO NOT DISTRIBUTE - 13 - 24) An adult female patient presented to her physician complaining that she feels weak and fatigue all the time. She reports that she and her husband are getting ready to be divorced, and that she is feeling stressed. Laboratory tests for thyroid function revealed no abnormalities. The physician, however, prescribed her T3. If further testing was done 2 days after beginning treatment, the findings would most likely include: a) Elevated thyroid stimulating hormone (TSH) b) Increased iodide uptake c) Increased thyroglobulin synthesis d) No change in iodide uptake e) Reduced T4 25) An experimental animal was created to determine the effects of thyroid stimulating hormone (TSH) deficiency on growth and development. Scientists genetically altered a mouse model by mutating the gene responsible for TSH receptors. A colony was created and called the TSH receptor knockout (TSH-R-KO) mouse strain. The predicted phenotype of TSH-R-KO offspring would include: a) Above average intellectual capabilities b) Dwarfism c) Elevated basal metabolic rate d) Elevated body temperature e) Elevated growth hormone (GH) secretion 26) A female patient presents to her physician complaining of agitation, weight loss, increased appetite, abnormal menstrual cycles, and sleep disturbances. Physical examination reveals that her skin is moist and she has a temperature of 100°C. She reports that her skin itches sometimes and examination reveals pretibial myxedema. A triiodothyronine (T3) resin uptake test (T3RU) revealed increased T3 resin uptake. The physician diagnoses her with Graves’ disease. Further laboratory tests would reveal: a) Decreased T3 levels b) Decreased T4 levels c) Elevated thyroid binding globulin concentration d) Elevated thyroid stimulating hormone e) Saturation of thyroid binding globulin (TBG) binding sites Endocrine #4 – Physiology: Parathyroid Regulation & Bone Remodeling 1) Where is calcium mainly stored in the body? a) Bound to albumin b) Bone and teeth c) Kidneys d) Ionized in serum e) Non-ionized in serum (complexed) f) Muscle 2) What calcium turnover process results in the loss of the most calcium? a) Bone remodeling b) Kidney filtering c) ECF secretion d) Intestinal excretion e) Exchangeable pool
  • 14. Endocrine 05May2009 DO NOT DISTRIBUTE - 14 - 3) Where is phosphate mainly stored in the body? a) Teeth b) Muscle c) Bone d) Kidneys e) Blood 4) What phosphate turnover process results in the loss of the most phosphate? a) Bone remodeling b) Kidney filtration c) Exchangeable pool d) Intestinal secretion e) Soft tissue remodeling 5) Which of the following describes the action of parathyroid hormone (PTH)? a) Increased Ca++ and bone reabsorption, increased phosphate reabsorption b) Increased Ca++ and bone reabsorption, decreased phosphate reabsorption c) Decreased Ca++ and bone reabsorption, increased phosphate reabsorption d) Decreased Ca++ and bone reabsorption, decreased phosphate reabsorption 6) Which of the following describes the action of vitamin D (intestine & bone)? a) Increased Ca++ and bone absorption, increased phosphate absorption b) Increased Ca++ and bone absorption, decreased phosphate absorption c) Decreased Ca++ and bone absorption, increased phosphate absorption d) Decreased Ca++ and bone absorption, decreased phosphate absorption 7) Which of the following calcium changes would occur in a patient who has their parafollicular thyroid cells removed? a) Increased calcium stores b) Decreased calcium stores c) Increased serum calcium d) Decreased serum calcium e) No change in calcium 8) A patient is found to have a PTH tumor. Which of the following sequela may occur? a) Difficulty urinating b) Goiter c) Soft tissue pain d) Bone fracture e) Carotid bruit 9) What is the action of calcitonin? a) Activate osteoblasts b) Deactivate osteoblasts c) Activate osteoclasts d) Deactivate osteoclasts 10) Which of the following is NOT true of parathyroid hormone? a) Produced by chief cells of the parathyroid gland b) Decreases ionized plasma Ca++ levels c) Increases plasma phosphate levels d) Inhibited by 1,25-OH2-D via negative feedback e) Decreased vitamin D production and bone resorption
  • 15. Endocrine 05May2009 DO NOT DISTRIBUTE - 15 - 11) A patient presents with complains of lethargy. Testing reveals a likely PTH-releasing tumor. If phosphate is the likely cause of the lethargy, how is PTH affecting the kidneys? a) Increased reabsorption of phosphate at the proximal tubule b) Decreased reabsorption of phosphate at the proximal tubule c) Increased reabsorption of phosphate at the distal tubule d) Decreased reabsorption of phosphate at the distal tubule 12) What affect does PTH have on calcium in the kidneys? a) Increased reabsorption of calcium at the proximal tubule b) Decreased reabsorption of calcium at the proximal tubule c) Increased reabsorption of calcium at the distal tubule d) Decreased reabsorption of calcium at the distal tubule 13) Which of the following would be seen in a patient with hypoparathyroidism? a) Hypercalcemia b) Hypercalciuria c) Renal stones d) Hyperphosphatemia e) Decreased bone mass 14) Vitamin D (D2 in plants/yeast, D3 in diet/skin) must be hydroxylated to its active form, 1,25-OH2-D. Where does the first hydroxylation occur? a) Liver b) Blood c) Kidney proximal tubule d) Kidney distal tubule e) Bone and teeth 15) Where does 1alpha-hydroxylase act? a) Liver b) Blood c) Kidney proximal tubule d) Kidney distal tubule e) Bone and teeth 16) Which of the following describes the activity of 1,25-OH2D3? a) Increases calcium absorption in the intestine b) Increases calcium excretion from the intestine c) Increases proximal tubule calcium reabsorption d) Decreases basolateral membrane Ca++ pumps e) Decreases calbindin synthesis 17) How is calcium transported from the basolateral side to the interstitial space? a) Ca++/H+ ATPase b) 3Na+/Ca++ exchanger c) Calbindin d) A & B e) A & C 18) A child presents with leg-bone deformity. Blood testing shows high levels of vitamin D in the blood. Which of the following is most likely? a) Type I rickets b) Type II rickets
  • 16. Endocrine 05May2009 DO NOT DISTRIBUTE - 16 - c) Renal stones d) Excess 1,25-OH2D3 e) Calcitonin deficiency 19) A man presents to his physician complaining of muscle cramping, irritability and tingling in the fingers and toes. Laboratory tests indicate that he is hypocalcemic and hyperphosphatemic. All other measures are unremarkable. Further laboratory tests would indicate: a) Elevated calcitonin levels b) Hyperparathyroidism c) Hypoparathyroidism d) Vitamin D deficiency e) Vitamin D excess 20) A female patient presents complaining of pain in the lower right side of her back, nausea, constipation, fatigue, muscle weakness, and lethargy. An MRI of the thyroid/parathyroid region revealed a small tumor on the parathyroid gland causing parathyroid over-activation. Further laboratory tests would indicate: a) High serum phosphate, high serum calcium, and high serum parathyroid hormone (PTH) b) High serum phosphate, low serum calcium, and low serum PTH c) Low serum phosphate, high serum calcium, and high serum PTH d) Low serum phosphate, high serum calcium, and low serum PTH e) Low serum phosphate, low serum calcium, and high serum PTH 21) A female patient with renal disease undergoes laboratory tests to measure plasma ions and renal function. Based on the results, her physician diagnoses her with a rare autoimmune disorder that is causing progressive kidney destruction. This destruction will probably also result in the impairment in the conversion of: a) 1,25-dihydroxycholecalciferol to 25-hydroxycholecalciferol b) 25-hydroxycholecalciferol to 1,25-dihydroxycholecalciferol c) Cholesterol to 7-dehydrocholesterol d) Vitamin D3 to 25-hydroxycholecalciferol e) Vitamin D3 to Vitamin D2 22) Which of the following is true? a) Most of bone mass is cortical bone and remodeling mostly affects cortical bone b) Most of bone mass is cortical bone and remodeling mostly affects spongy bone c) Most of bone mass is spongy bone and remodeling mostly affects cortical bone d) Most of bone mass is spongy bone and remodeling mostly affects spongy bone 23) Which of the following is contained within the osteoid? a) Ground substance b) Type I collagen c) Proteoglycans d) Minerals (CaPO4, Na, Carbonate, Mg) e) All of the above 24) Which of the following is stimulated by cytokines and inhibited by calcitonin? a) Osteoblasts b) Osteocytes c) Osteoclasts
  • 17. Endocrine 05May2009 DO NOT DISTRIBUTE - 17 - d) Osteocalcin e) Osteonectin 25) Annexins are secreted by which of the following in order to recruit more of their own? a) Osteoblasts b) Osteocytes c) Osteoclasts 26) What paracrine signaling is accomplished during bone remodeling? a) Osteocytes signal osteoblasts b) Osteocytes signal osteoclasts c) Osteoblasts signal osteoclasts d) Osteoblasts signal osteocytes e) Osteoclasts signal osteoblasts 27) PTH stimulates bone resorption by activating: a) Osteoblasts b) Osteocytes c) Osteoclasts 28) Which of the following people would have the most bone mass? a) Teenage soccer player b) Young adult weight lifter c) Female undergoing menopause d) Adult male jogger e) Elderly bedridden patient Endocrine #5 – Physiology: Adrenal Hormones Match the hormone with the primary location of production: 1) Catecholamines a) Adrenal cortex, zona glomerulosa 2) Aldosterone b) Adrenal cortex, zona fasciculata 3) Cortisol c) Adrenal cortex, zona reticularis 4) Androgens d) Adrenal medulla 5) A patient with hyperaldosteronism would likely have: a) Low ANP levels b) Low 18-OH-DOC levels c) Hypercalcemia d) Hypomagnesemia e) Hypokalemia 6) Which of the following would lead to an overall decrease in sodium and water excretion due to activation of ADH in the renin-angiotensin-aldosterone system (RAAS)? a) Increased blood pressure b) Decreased blood hematocrit c) Increased blood osmolarity d) Decreased blood osmolality e) Increased sodium retention 7) Angiotensin II is converted to aldosterone at what location? a) Brain b) Adrenals
  • 18. Endocrine 05May2009 DO NOT DISTRIBUTE - 18 - c) Lungs d) Liver e) Kidneys 8) What is the main function of aldosterone? a) Decrease sodium reabsorption b) Increase K+ secretion c) Maintain ECF volume d) Increase H+ secretion e) Deplete ICF volume 9) Cortisol releasing factor (CRF) from the hypothalamus would lead to all of the following EXCEPT: a) Increased growth b) Increased blood pressure c) Increased arousal d) Increased SNS e) Decreased reproductive hormones 10) Which of the following would inhibit ACTH? a) Stress b) Depression c) Anxiety d) Cortisol e) Alpha-agonists 11) Which of the following would lead to a higher setpoint for cortisol due to an over- ride of the negative feedback loop? a) Sleeping during the day b) Strenuous exercise c) Constant stress d) High fat diet e) Decreased CRH 12) Glucocorticoids (e.g. cortisol) inhibit which of the following via a negative feedback loop? a) Hypothalamus b) Corticotroph (anterior pituitary) c) Adrenal cortex d) A & B e) A & C Match the glucocorticoid disorder: Plasma cortisol Plasma ACTH 13) Primary hypercortisolism a) Increased Increased 14) Secondary hypercortisolism b) Decreased Increased 15) Primary hypocortisolism c) Increase Decreased 16) Secondary hypocortisolism d) Decreased Decreased 17) Which of the following is NOT an effect of glucocorticoids? a) Insulin resistance b) Lipogenesis in the trunk c) NPY inhibition d) Lipolysis in the extremities
  • 19. Endocrine 05May2009 DO NOT DISTRIBUTE - 19 - e) Muscle protein turnover and gluconeogenesis 18) Which of the following is NOT an effect of glucocorticoids? a) Decrease in muscle mass and strength b) Decreased bone formation and increased resorption c) Thinning of skin and capillary walls d) Decreases glomerular filtration rate e) Decreased memory 19) Which of the following is NOT an effect of glucocorticoids? a) Inhibits arachidonic acid synthesis b) Decreases neutrophil number but increases effectiveness c) Decreases fibroblast proliferation d) Stimulates T-cell apoptosis e) Inhibits IL, INF-gamma, COX-2 expression 20) Which of the following is true of androgens? a) Most of male androgens exist in the adrenals b) Female ovaries provide most of their androgens c) A castrated male can use the adrenals to compensate for testosterone loss d) Adrenal androgen is converted to testosterone in the female pubic and axillary areas e) Adrenal androgens are of great importance to a physiologic male 21) Scientists were conducting an experiment to look at the effect of surgical ablation on kidney function. After doing 40 ablation surgeries, their student was tired and anxious to finish. On his 41st surgery he accidentally removed a large section of the outer layer of the adrenal gland. If the animal were allowed to recover from surgery, hormonal measurements would probably indicate: a) Decreased adrenocorticotropic hormone (ACTH) secretion b) Decreased aldosterone production c) Decreased androgen production d) Decreased catecholamine production e) Decreased stress-induced cortisol secretion 22) In an experiment designed to characterize the hypothalamic-pituitary-adrenal axis in rodents, plasma adrenocorticotropic hormone (ACTH) and corticosterone were measured in several rats following various surgical and pharmaco- logical manipulations. The following graph plots the relative hormone levels for each animal. Hormonal profiles for each group reveal: a) Group A may have adrenal hyperplasia b) Group A may have primary adrenal insufficiency c) Group B may have hypothalamic deficiency d) Group B may have pituitary deficiency e) Group C may have adrenal hyperplasia 23) A male patient presents to his physician complaining of muscle weakness and abnormal heartbeat he describes as “beating out of tune.” He was previously been diagnosed with Type 1 (insulin-dependent) diabetes mellitus, but reports strict adherence to insulin treatments and eats a low-salt and low-carbohydrate diet. Laboratory tests
  • 20. Endocrine 05May2009 DO NOT DISTRIBUTE - 20 - measuring serum electrolytes revealed low sodium and elevated potassium levels. Further laboratory tests would reveal: a) Elevated ACTH secretion b) Elevated aldosterone secretion c) Elevated glucocorticoid secretion d) Reduced ACTH secretion e) Reduced renin secretion 24) A 49-year-old male presents to his physician complaining of a persistent “dry” cough, muscle weakness, and reduced libido. Physical examination reveals abnormal weight gain in the back, abdomen, and face and abdominal stria. He reports that he has smoked 2 packs of cigarettes per day for the last 30 years. Laboratory tests revealed elevated serum glucose, a positive dexamethasone suppression test, and a hormone-secreting bronchial carcinoma. Further laboratory tests would most likely reveal that the hormone secreted by the tumor is: a) Adrenocorticotropic hormone (ACTH) b) Cortisol c) Glucagon d) Growth hormone e) Thyroxine 25) A male patient is experiencing adrenal insufficiency and cannot produce adequate amounts of cortisol, either basal or stress-induced. Under basal conditions metabolism is not greatly affected, but he experiences severe weakness when confronted with a stressor. Laboratory tests conducted during periods of stress would likely indicate: a) Decreased glucose uptake in muscles and adipose b) Hypoglycemia c) Increased circulating amino acids d) Increased gluconeogenesis e) Increased lipoysis 26) Where is norepinephrine mostly produced? a) Preganglionic sympathetic fibers b) Postganglionic sympathetic fibers c) Preganglionic parasympathetic fibers d) Postganglionic parasympathetic fibers 27) Which of the following actions is decreased by epinephrine and norepinephrine? a) Insulin action b) Glucagon secretion c) BMR and thermogenesis d) Gluconeogenesis e) Glycogenolysis 28) Which of the following actions is decreased by epinephrine and norepinephrine? a) Heart rate b) Renin secretion c) Skin blood flow d) Systolic pressure e) Cardiac contractility f) Bronchodilation
  • 21. Endocrine 05May2009 DO NOT DISTRIBUTE - 21 - 29) A male patient presents to his physician complaining of feelings of “nervousness” followed by fatigue. Subsequent tests revealed an adrenal tumor in the medullary region. Further laboratory testing would most likely reveal: a) Decreased circulating free fatty acids b) Decreased circulating glucose levels c) Decreased circulating insulin levels d) Increased glycogenesis e) Increased lipogenesis Endocrine #6 – Physiology: Hormones Involved in Food Intake & Energy Storage Match the hypothalamic location with the function: 1) Satiety a) Lateral nuclei 2) Regulation of feeding b) Ventromedial nuclei 3) Hunger c) Paraventricular nucleus (PVN) 4) Multi-hormone action d) Dorsomedial nuclei (DMN) 5) Increase eating e) Arcuate nuclei (ACN) 6) Which of the following components of the arcuate nucleus decreases food intake and increases energy expenditure? a) Alpha-MSH (melanocyte-stimulating hormone) b) NPY (neuropeptide Y) c) CART (cocaine- and amphetamine-related transcripts) d) AGRP (agouti-related protein) e) A & C f) B & D 7) POMC neurons release alpha-MSH, which acts on MCR-3 and MCR-4 melanocortin receptors in the paraventricular nucleus. Which of the following would occur if a patient had an MCR-4 mutation or who receives MCR-4 antagonist (AGRP)? a) Addiction to cocaine b) Rapid weight loss c) Early satiety d) Obesity e) Intolerance to amphetamines 8) Which of the following decreases POMC activation and is the most associated with increased appetite (hunger)? a) NPY b) Alpha-MSH c) CART d) AGRP 9) Antagonizing which of the following could be a helpful treatment for an obese patient whose weight problem is caused by Leptin? a) MCH (melanin-concentrating hormone) b) CART (cocaine- and amphetamine-related transcripts) c) Orexin (hypocretin) d) NPY (neuropeptide Y) e) CB1 (cannabinoid) 10) Which of the following would increase appetite and decrease ACTH levels?
  • 22. Endocrine 05May2009 DO NOT DISTRIBUTE - 22 - a) MCH (melanin-concentrating hormone) b) CART (cocaine- and amphetamine-related transcripts) c) Orexin (hypocretin) d) NPY (neuropeptide Y) e) CB1 (cannabinoid) 11) A deficiency of orexin (hypocretin) would lead to: a) Malnutrition b) Neoplasm c) Narcolepsy d) Diabetes e) Agitation 12) Which of the following short-term regulators that decrease eating responds to fat entering the duodenum? a) Peptide YY b) Glucagon-like peptide c) Cholecystokinin d) Insulin 13) A patient with Prader-Willi is found to have high levels of Ghrelin. What affect does this have? a) Hunger b) Double vision c) Easy bruising d) Hyperthyroidism e) Hypogonadism 14) Which of the following would NOT occur in the presence of Leptin? a) Decreased NPY and AGRP b) POMC inhibition c) CRH increase d) SNS increase e) Insulin decrease 15) The Zucker Diabetic Fatty (ZDF) rat has which of the following disorders? a) Childhood over-nutrition to increase fat cells b) Hypothalamic lesion c) Hypophyseal tumor d) MCR gene mutation e) Leptin receptor mutation 16) Which of the following is the recommended primary obesity treatment? a) Physical exercise b) Amphetamines c) Altering lipid metabolism d) Gastric bypass e) Gastric banding 17) A mutation in POMC would cause which of the following? a) Increased food intake and weight gain b) Increased food intake but weight loss c) Decreased food intake and weight loss
  • 23. Endocrine 05May2009 DO NOT DISTRIBUTE - 23 - d) Decreased food intake but weight gain 18) What is the mechanism by which an AGRP gene mutation decreases food intake? a) Antagonizes MCR-3 and MCR-4 receptors b) Cause release of alpha-MSH c) SNS activation to the nucleus tractus solitarius d) Decreases POMC activation e) Leptin antagonism 19) What is the mechanism by which an NPY gene mutation decreases food intake? a) Antagonizes MCR-3 and MCR-4 receptors b) Cause release of alpha-MSH c) SNS activation to the nucleus tractus solitarius d) Decreases POMC activation e) Leptin antagonism Endocrine #7 – Physiology: Growth 1) Which of the following would lead to an increase in growth hormone (GH)? a) Caloric restriction b) Increased amino acids c) Increased glucose or free fatty acids d) Exercise or stress e) Ghrelin 2) Which of the following inhibits GH by acting on GHRH? a) Somatostatin b) Cortisol c) Old-age d) Obesity e) Pregnancy 3) A woman presents with acromegaly and breast discharge. Which of the following hormones would likely be high? a) Prolactin b) Oxytocin c) ACTH d) GH e) ADH 4) The JAK-STAT pathway involves taking dimerized GH and transcribing it to IGF-1 (insulin-like growth factor). Where does this mainly occur? a) Liver b) Long bone c) Pituitary d) Kidney e) Hypothalamus 5) Which of the following is a function of IGF-1, not GH? a) Gluconeogenesis b) Bone and tissue growth c) Insulin resistance d) Protein synthesis
  • 24. Endocrine 05May2009 DO NOT DISTRIBUTE - 24 - e) Lipolysis 6) Which of the following promotes the closure of long bone epiphyseal plates? a) GH b) IGF-1 c) Estrogen & testosterone d) Insulin & thyroid hormones 7) Which of the following would NOT occur in an adult with high levels of growth hormone? a) Prominent jaw b) Prominent brow c) Large hands and feet d) Increase height e) Large nose 8) Hypothyroidism in infancy would lead to what growth effects? a) Developmental retardation b) Large long bones c) Leg length discrepancies d) Prominent jaw and nose e) Organomegaly, especially heart 9) Which of the following would occur in a patient with congenital growth hormone receptor dysfunction? a) Dwarfism with low GH levels and secondary sex characteristics present b) Dwarfism with high GH levels and secondary sex characteristics present c) Dwarfism with low GH levels and no secondary sex characteristics d) Dwarfism with high GH levels and no secondary sex characteristics 10) Which of the following would be seen in a patient with excess growth hormone? a) Hypoglycemia and hypergonadism b) Hyperglycemia and hypergonadism c) Hypoglycemia and hypogonadism d) Hyperglycemia and hypogonadism 11) A 42-year-old male visits a local physician for a physical exam. He reports that this is his first physical in 5 years, and that he is concerned about changes in his appearance over the last couple years. He explains that his feet and hands have grown larger, his jaw is protruding, and his face is “looking strange.” He reports slight difficulty breathing, frequent headaches, fatigue and weakness, impotence, and excessive sweating. Laboratory tests indicate elevated insulin-like growth factor-1 (IGF-1), and an MRI scan revealed a 1.5 cm mass on the anterior pituitary. Further physical examination and laboratory tests may indicate: a) Decreased hypothalamic somatostatin secretion b) Decreased protein synthesis c) Hypoglycemia d) Insulin resistance e) Muscle atrophy 12) A 25-year-old male presents to his physician for an annual physical. He was previously diagnosed with a growth hormone (GH)-secreting tumor on his anterior
  • 25. Endocrine 05May2009 DO NOT DISTRIBUTE - 25 - pituitary, for which he is taking a somatostatin analog. Laboratory tests, however, reveal elevated GH levels. Further laboratory tests would most likely reveal: a) Decreased gluconeogenesis b) Increased lipolysis c) Increased long bone growth d) Increased insulin action e) Decreased insulin levels 13) A young adult female patient visits her physician for a physical exam. The physician observes distinct changes in her appearance that suggest excess growth hormone (GH) action. Her feet and hands appear larger than expected, and her jaw protrudes more than normal. Laboratory tests indicate elevated insulin-like growth factor-1 (IGF-1), and an MRI scan revealed a 1.5 cm mass on the anterior pituitary. Further physical examination and laboratory tests may indicate: a) Decreased circulating free fatty acids b) Decreased hypothalamic somatostatin secretion c) Decreased lean muscle mass d) Increased lipogenesis e) Hyperglycemia 14) A physician notices abnormal growth patterns in a young male child during an annual physical. The child was severely below normal on standardized growth tests for both height and weight. All other aspects of the physical examination were unremarkable. Laboratory tests indicate normal thyroxine and triiodothyronine and decreased levels of insulin-like growth factor-1. Further laboratory tests would indicate: a) A decrease in insulin-like growth factor binding proteins (IGFBP) b) A decrease in thyroid stimulating hormone (TSH) c) An increase in growth hormone (GH) secretion d) An increase in IGFBP e) An increase in thyroid stimulating hormone (TSH) 15) An elderly male presents to his physician to discuss with his physician a new drug that he saw on the television. He reported that he was always in very good shape but is now “soft and flabby.” He explained that he saw a commercial about human growth hormone (GH) administration that was described as a “fountain of youth” and he wanted to see if he could begin taking it. Laboratory tests revealed that the patient had reduced GH levels, and the doctor agreed to prescribe the GH replacement for him. The most likely effect of GH replacement would be: a) Decreased collagen synthesis b) Decreased lipolysis c) Decreased protein synthesis d) Increased glucose uptake in muscle and adipose e) Increased plasma glucose Endocrine #8 – Physiology: Endocrine Pancreas 1) Which of the following cells secrete amylin and pancreastatin? a) Alpha cells b) Beta cells c) Delta cells
  • 26. Endocrine 05May2009 DO NOT DISTRIBUTE - 26 - d) PP cells 2) C peptide has a long serum half-life so it can be helpful in determining past levels of which of the following? a) Insulin b) Glucagon c) Somatostatin d) Pancreatic polypeptide e) Glucose 3) Which of the following cells is the first to receive arterial blood supply? a) Alpha cells b) Beta cells c) Delta cells 4) Which of the following inhibits insulin? a) Eating b) Glucagon c) Intestinal hormones d) Glucose e) Leptin 5) Which of the following describes the insulin response associated with glucose injection? a) Monophasic with an initial rapid increase due to intestinal hormones b) Monophasic with a gradual increase due to intestinal hormones c) Biphasic with an initial rapid increase due to intestinal hormones and a secondary increase due to beta cells d) Biphasic with an initial rapid increase due to beta cells and a secondary increase due to intestinal hormones e) Monophasic with a gradual increase due to beta cells 6) Insulin causes the insertion of what glucose transporter, leading to glucose crossing membranes to be converted to glycogen in adipose tissue and skeletal muscle? a) GLUT1 b) GLUT2 c) GLUT3 d) GLUT4 e) GLUT5 7) Insulin causes a decrease in which of the following within the liver? a) Glycogenolysis b) Glycolysis c) Glycogenesis d) Protein synthesis e) Lipogenesis 8) Which of the following stimulates glucagon secretion? a) Somatostatin b) Insulin c) Hypoglycemia d) Secretin 9) What is the major action of glucagon in the pancreas?
  • 27. Endocrine 05May2009 DO NOT DISTRIBUTE - 27 - a) Increased glycogenolysis b) Gluconeogenesis c) Increased insulin d) Lipolysis e) Ketogenesis 10) Which of the following would cause the insulin to glucagon ratio to increase the most (>10.0 from normal of 2.0? a) Pure protein meal b) Carbohydrate loading c) Pure fat meal d) Prolonged exercise e) Fasting 11) Glucagon-like peptide-1 (GLP-1) is released from the intestines and has what action? a) Increase insulin and increase glucagon b) Increase insulin and decrease glucagon c) Decrease insulin and increase glucagon d) Decrease insulin and decrease glucagon 12) Which of the following is an action of somatostatin? a) Inhibits insulin b) Inhibits glucagon c) Inhibits GI motility and secretions d) Inhibits glucose and triglyceride absorption e) All of the above 13) Which of the following is regulated by hypoglycemia, inhibited by hyperglycemia, and is used diagnostically as a marker for an islet cell tumor? a) Insulin b) Glucagon c) Somatostatin d) Pancreatic polypeptide e) Glucose 14) Which of the following is NOT a symptom of Type-1 (insulin dependent) or Type-2 (non-insulin dependent) diabetes mellitus? a) Weight gain b) Polydipsia c) Polyuria d) Increased food consumption 15) Which of the following is associated with Type-1 diabetes more so than Type-2? a) Genetic component b) Obesity as a cause c) Native American race d) Autoimmune reaction e) Sulfonylurea drugs for treatment 16) A middle aged male presented to his physician for an annual physical. Routine testing revealed elevated plasma glucose levels. Further laboratory testing revealed elevated insulin levels and severely elevated glucagon levels. Following an MRI, he was
  • 28. Endocrine 05May2009 DO NOT DISTRIBUTE - 28 - diagnosed with a pancreatic !-cell tumor. The elevated glucose levels in this patient is most likely due to: a) Decreased glycogenolysis b) Decreased lipogenesis c) Increased gluconeogenesis d) Increased lipolysis e) Increased hepatic glycogenesis 17) A 37-year-old pregnant woman visits her physician for a prenatal visit. She was in her 13th week of gestation and had been healthy with a normal pregnancy thus far. Urinalysis revealed elevated glucose levels and blood tests revealed elevated insulin. She has no history of diabetes, although her mother developed Type 2 (non-insulin dependent) diabetes later in life. Her elevated glucose levels are most likely due to: a) Autoimmune pancreatic alpha cell destruction b) Autoimmune pancreatic beta cell destruction c) Elevated growth hormone (GH) secretion d) Increased carbohydrate ingestion e) Insulin resistance 18) An experiment was designed to determine the effects of a high-fat diet on carbohydrate metabolism. Rats in group A were fed a special formulated diet high in fat, and rats in group B were fed an isocaloric low-fat diet. These diets were fed to the rats for 2 weeks and a fasting glucose tolerance test was performed. The following graph illustrates the results for group A (Time 0 is glucose administration). The condition that would most likely produce these results is: a) Excess glucagon release b) Increased glycogenesis c) Increased glycogenolysis d) Type-1 (insulin dependent) diabetes mellitus e) Type-2 (noninsulin dependent) diabetes mellitus Endocrine #9 – Pharmacology: Hypothalamic & Pituitary Hormones 1) Which of the following is located directly superior to the sella turcica? a) Hypothalamus b) Pituitary c) Pons d) Cingulate gyrus e) Medulla 2.1) Which of the following hormones released from the anterior pituitary does NOT stimulate the production of hormones by a peripheral endocrine gland or the liver? a) FSH b) LH c) ACTH d) TSH e) PRL
  • 29. Endocrine 05May2009 DO NOT DISTRIBUTE - 29 - 2.2) Posterior pituitary hormones (Oxytocin, ADH) are released from neurons that arise from what area? a) Preoptic nucleus b) Supraoptic nucleus c) Paraventricular nucleus d) Cingulate gyrus e) Solitary nucleus 3.1) Dopamine from the hypothalamus inhibits which of the following? a) Growth hormone (GH, somatotropin) b) Thyroid-stimulating hormone (TSH) c) Adrenocorticotropin (ACTH) d) Follicle-stimulating hormone (FSH) e) Luteinizing hormone (LH) f) Prolactin (PRL) 3.2) Which hypothalamic hormone is used rarely to differentiate Cushing disease from ectopic secretion? a) Growth hormone-releasing hormone (GHRH) b) Thyrotropin-releasing hormone (TRH, protirelin) c) Corticotropin-releasing hormone (CRH) d) Gonadotropin-releasing hormone (GnRH) e) Dopamine 4) Which of the following conditions for which human growth hormone (hGH) treatment is indicated is considered the most controversial? a) Prader-Willi syndrome b) Turner syndrome c) Idiopathic short stature d) None of the above 5) A patient with infertility is undergoing fertility treatment in order to conceive. On day three of her menstrual cycle, she is started on daily injections of an FSH preparation, which continue for about 7-12 days. Since exogenous gonadotropins are being used to stimulate follicular development, they are administered in conjunction with either a continuous GnRH agonist (down-regulates receptors) or a GnRH antagonist. Why? a) To prevent a luteal surge b) To reduce endometrial thickness c) To increase FSH levels d) To reduce serum estradiol levels e) To cause rapid ovulation 6) Which of the following are D2 agonists and thus are useful for hyperprolactinemia? a) Fenoldopam b) Bromocriptine and cabergoline c) Metoclopramide, droperidol, and domperidone d) Clozapine, risperidone, and olanzapine e) Amoxapine 7) A post-partum woman is having continual uterine bleeding. Which of the following could be given to help reduce hemorrhage? a) Oxytocin
  • 30. Endocrine 05May2009 DO NOT DISTRIBUTE - 30 - b) Dopamine agonist c) ADH (vasopressin) d) Prolactin (PRL) e) Luteinizing hormone (LH) 8) Antidiuretic hormone (ADH, vasopressin) may be indicated in which of the following patients? a) Diabetes mellitus type-1 b) Diabetes mellitus type-2 c) Diabetes insipidus d) Hypertension e) Galactorrhea Endocrine #10 – Pharmacology: Thyroid & Antithyroid Drugs 1.1) Iodine peroxidase, which converts I- to I2, occurs in what location? a) Thyroid gland membrane b) Within the thyroid gland c) Blood d) Peripheral tissues 1.2) Amiodarone, corticosteroids, beta-blockers, and ipodate affect thyroid hormone formation at what location? a) Thyroid iodine transporter b) Thyroid peroxidase enzyme c) Thyroid T3/T4 proteolysis to blood d) T4 to T3 conversion in peripheral tissues e) B & C 1.3) Thioamides affect thyroid hormone formation at what location? a) Thyroid iodine transporter b) Thyroid peroxidase enzyme c) Thyroid T3/T4 proteolysis to blood d) T4 to T3 conversion in peripheral tissues e) B & C 1.4) Iodides affect thyroid hormone formation at what location? a) Thyroid iodine transporter b) Thyroid peroxidase enzyme c) Thyroid T3/T4 proteolysis to blood d) T4 to T3 conversion in peripheral tissues e) B & C 2) Levothyroxine (Synthroid, Levoxyl) is the preparation of choice for thyroid replacement. Which of the following is NOT true of levothyroxine? a) Short half-life b) Low drug cost c) Lack of allergenic foreign protein d) Uniformity and stability e) Easy lab measurement of serum levels 3) All thioamines have a simple molecular structure in common. What is the name of this structure (shown)?
  • 31. Endocrine 05May2009 DO NOT DISTRIBUTE - 31 - a) Thioacetamide b) Thionamide c) Triiodothyronine d) Thiocarbamide e) Thioacetamide 4) An elderly patient is receiving levothyroxine and having their serum TSH and free thyroxine monitored. Which of the following signs or symptoms may signify an adverse effect to the medication? a) Weight gain b) Bradycardia c) Heart palpitations d) Cold intolerance e) Somnolence 5) A patient with known hypothyroidism presents in a near-coma condition with edema, bradycardia, and low core body temperature (myxedema coma). Which of the following would be considered appropriate treatment? a) Oral levothyroxine, 50-100mcg (low dose) b) Oral levothyroxine, 300-400mcg (high dose) c) IV levothyroxine, 50-100mcg (low dose) d) IV levothyroxine, 300-400mcg (high dose) e) IV fluids and glucose only 6) A non-pregnant patient with Graves disease is being examined for treatment options. What is the preferred drug for this patient, which requires 12-18 months of therapy? a) Desmopressin (DDAVP) b) Fludrocortisone (Florinef) c) Levothyroxine (Synthroid) d) Methimazole (Tapazole) e) Octreotide (Sandostatin) Endocrine #11 – Pharmacology: Corticosteroids & Antagonists 1.1) Deficiency of the enzyme 17alpha-hydroxylase would prevent the formation of which of the following and have what effect on blood pressure? a) No aldosterone or cortisol, hypotension b) No aldosterone or cortisol, hypertension c) No cortisol or sex hormones (estradiol/testosterone), hypotension d) No cortisol or sex hormones (estradiol/testosterone), hypertension 1.2) Deficiency of the enzyme 21-hydroxylase would prevent the formation of which of the following and have what effect on blood pressure? a) No aldosterone or cortisol, hypotension b) No aldosterone or cortisol, hypertension c) No cortisol or sex hormones (estradiol/testosterone), hypotension d) No cortisol or sex hormones (estradiol/testosterone), hypertension 1.3) Deficiency of the enzyme 11beta-hydroxylase would prevent the formation of which of the following and have what effect on blood pressure? a) No aldosterone or cortisol, hypotension b) No aldosterone or cortisol, hypertension
  • 32. Endocrine 05May2009 DO NOT DISTRIBUTE - 32 - c) No cortisol or sex hormones (estradiol/testosterone), hypotension d) No cortisol or sex hormones (estradiol/testosterone), hypertension 2) Which of the following describes the hierarchy of anti-inflammatory effects for the included drugs? a) Hydrocortisone > Prednisone > Triamcinolone > Dexamethasone b) Triamcinolone > Dexamethasone > Hydrocortisone > Prednisone c) Dexamethasone > Triamcinolone > Prednisone > Hydrocortisone d) Hydrocortisone > Triamcinolone > Dexamethasone > Prednisone e) Dexamethasone > Hydrocortisone > Prednisone > Triamcinolone 3) Which of the following describes the hierarch of salt-retaining efficacy for the included drugs? a) Fludrocortisone > Hydrocortisone > Triamcinolone b) Fludrocortisone > Triamcinolone > Hydrocortisone c) Hydrocortisone > Fludrocortisone > Triamcinolone d) Hydrocortisone > Triamcinolone > Fludrocortisone e) Triamcinolone > Hydrocortisone > Fludrocortisone 4) In the fasting state, glucocorticoids stimulate phosphoenolypyruvate, carboxykinase, and G6P. What major effect does this have? a) Increased urine amino acids b) Increased serum amino acids c) Decreased gluconeogenesis d) Increased gluconeogenesis e) Decreased glycogen synthesis 5) Which of the following would occur in serum (blood) after injection of a short-acting glucocorticoid? a) Increased neutrophils and increased lymphocytes b) Increased neutrophils and decreased lymphocytes c) Decreased neutrophils and increased lymphocytes d) Decreased neutrophils and decreased lymphocytes 6) A defect in 21beta-hydroxylase would prevent formation of cortisol, which leads to a compensatory increase in ACTH. This causes the adrenal gland to become hyperplastic and secrete large amounts of precursors. What effect would this have? a) Hypotension b) Hypokalemia c) Salt wasting d) Alopecia e) Virilization 7) Cushing disease (ACTH-secreting pituitary adenoma) can be treated with metyrapone (Metopirone) or which of the following? a) Desmopressin (DDAVP) b) Ketoconazole (Nizoral) c) Levothyroxine (Synthroid) d) Methimazole (Tapazole) e) Somatotropin (Humatrope)
  • 33. Endocrine 05May2009 DO NOT DISTRIBUTE - 33 - 8) Mifepristone (RU-486) is a progesterone antagonist with some anti-glucocorticoid properties at higher doses. Which of the following is describes the mechanism by which mifepristone acts with blockage of glucocorticoid receptors? a) Stabilizes the Hsp-glucocorticoid receptor complex b) Inhibits dissociation of the RU-486-bound glucocorticoid receptor from the Hsp chaperone proteins c) Alters the interaction of the glucocorticoid receptor with coregulators, favoring the formation of a transcriptionally inactive complex d) All of the above Endocrine #12 – Pharmacology: Gonadal Hormones & Inhibitors 1.1) Which of the following does NOT peak at day 14 of a 28-day menstrual cycle? a) LH b) FSH c) Estradiol d) Progesterone 1.2) At which of the following days during a 28-day menstrual cycle would endometrial lining be the thickest? a) 7 b) 14 c) 21 d) 28 2) Which of the following steroid reactions is NOT correct? a) Androstenedione can convert to testosterone without an enzyme b) Testosterone requires aromatase to create estradiol c) Estradiol requires aromatase to create estrone d) Androstenedione requires aromatase to create estrone e) Testosterone can convert to androstenedione without an enzyme 3) What effect does FSH have on the follicle of the mature ovary? a) Stimulates growth of ovarian follicles and estrogen secretion b) Stimulates growth of ovarian follicles and progesterone secretion c) Stimulates recession of ovarian follicles and estrogen secretion d) Stimulates recession of ovarian follicles and progesterone secretion 4) Estrogen (estradiol) receptors are found in the nucleus in what stable conformation? a) Bound to estrogen response elements (EREs) b) Bound to heat shock protein (Hsp90) c) As a dimer (estradiol-estradiol) d) Bound to estrone e) In an aromatic ring-form pattern 5) According to the Women’s Health Initiative (WHI), estrogen treatment or hormone replacement therapy (HRT) in peri- and post-menopausal women can increase the risk of cardiovascular disease as well as which of the following? a) Increased risk of Alzheimer disease b) Negative effect on circulating lipids c) Increased risk of breast cancer d) Increased risk of colon cancer
  • 34. Endocrine 05May2009 DO NOT DISTRIBUTE - 34 - e) Negative effect on circulating lipoproteins 6) What is the claimed advantage of third-generation synthetic progestins (19-not, 13- ethyl), including desogestrel, gestodene, and norgestimate, when compared with older synthetic progestins? a) Less effective gonadotropin inhibitors b) Less estrogenic activity c) More estrogenic activity d) Less androgenic activity e) More androgenic activity 7) Which of the following is an adverse effect of oral contraceptives (estrogenic compounds), which has increased risk in smokers over aged 35? a) Bleeding disorders b) Small cell lung cancer c) Duodenal ulcers d) Rheumatic fever e) Venous embolism 8) Which of the following post-coital contraceptives is considered abortifacient? a) Conjugate estrogen b) Ethinyl estradiol c) Mifepristone d) Diethylstilbestrol e) Norgestrel 9.1) What is the role of LH on the testis? a) Promote sperm maturation b) Promote testosterone synthesis c) Promote testosterone secretion d) A & B e) B & C 9.2) What is the role of FSH on the testis? a) Promote sperm maturation b) Promote testosterone synthesis c) Promote testosterone secretion d) A & B e) B & C 10) In the skin, prostate, and seminal vesicles, testosterone is converted to which of the following? a) DHEA b) DHT c) ACTH d) Cortisol e) Estradiol 11) Which of the following is NOT correct regarding the dosing regimen of androgens in cases of pubertal hypogonadism? a) Long-acting agents such as testosterone enanthate or cypionate are used b) Doses are 50mg IM injection
  • 35. Endocrine 05May2009 DO NOT DISTRIBUTE - 35 - c) Initial doses are every 4-weeks, then every 3-weeks, then every 2-weeks until maturation is complete d) Adult doses are 200mg at 4-week intervals 12.1) Finasteride (Proscar) inhibits the production of DHT by inhibiting what enzyme? a) 17alpha-hydroxylase b) Aromatase c) 11beta-hydroxylase d) 5alpha-reductase e) 21-hydroxylase 12.2) Flutamide (Eulexin) has which of the following effects? a) Agonist at the androgen receptor b) Competitive antagonist at the androgen receptor c) Inhibitor of 5alpha-reductase d) Activator of 5alpha-reductase Endocrine #13 – Pharmacology: Pancreatic Hormones & Antidiabetic Drugs 1.1) Which of the following is found in beta cells of the pancreas and regulates insulin release as well as other aspects of glucose homeostasis? a) GLUT1 b) GLUT2 c) GLUT3 d) GLUT4 e) GLUT5 1.2) Which of the following is found in muscle and adipose tissue and functions in insulin-mediated uptake of glucose? a) GLUT1 b) GLUT2 c) GLUT3 d) GLUT4 e) GLUT5 2) Which of the following describes the hierarchy of time to peak action for the included insulin formations? a) Lispro insulin > Regular insulin > NPH > Glargine insulin b) Regular insulin > Lispro insulin > NPH > Glargine insulin c) NPH > Glargine insulin > Lispro insulin > Regular insulin d) Regular insulin > Glargine insulin > Lispro insulin > NPH e) Glargine insulin > NPH > Regular insulin > Lispro insulin 3) How do sulfonylurea drugs affect insulin release? a) Blocks GLUT2 b) Depolarizes (closes) K+ channels c) Polarizes (opens) K+ channels d) Depolarizes (opens) Ca++ channels e) Polarizes (closes) Ca++ channels f) Blocks insulin exocytosis 4) Why are children under the age of seven exempt from the American Diabetes Association’s Benefits of Tight Glycemic Control of Diabetes guidelines?
  • 36. Endocrine 05May2009 DO NOT DISTRIBUTE - 36 - a) Risk of thromboembolic events b) Risk of brain damage due to hypoglycemia c) Risk of neuropathy due to hyperglycemia d) Risk of renal failure due to glycemic fluctuations e) Risk of permanent bone mass loss 5) Which of the following is NOT an alternative to insulin syringe injections currently available for Type-1 diabetics? a) Subcutaneous insulin infusion devices b) Portable pen injectors c) Ocular insulin drops d) Inhaled insulin 6) Risk is most reduced in what system for patients who adhere to the guidelines for Benefits of Tight Glycemic Control of Diabetes? a) Neurologic b) Gastrointestinal c) Renal d) Reproductive e) Cardiovascular 7) Which of the following is NOT a symptom of hypoglycemia? a) Difficulty speaking b) Tachycardia c) Convulsions d) Pinpoint pupils (miosis) e) Headache 8) When are alpha-glucosidase inhibitors indicated? a) Initial therapy for Type-1 diabetics b) Adjunctive therapy for Type-1 diabetics c) Initial therapy for Type-2 diabetics d) Adjunctive therapy for Type-2 diabetics e) Never due to their side-effect profile 9) Which of the following describes the hierarchy of maximal duration of action for the included sulfonylureas? a) Tolbutamide > Glimepiride > Chlorpropamide b) Glimepiride > Chlorpropamide > Tolbutamide c) Chlorpropamide > Glimepiride > Tolbutamide d) Chlorpropamide > Tolbutamide > Glimepiride e) Tolbutamide > Chlorpropamide > Glimepiride 10) Which of the following is the safest sulfonylurea for elderly diabetics and why? a) Chlorpropamide due to its long half-life b) Chlorpropamide due to its short half-life c) Tolbutamide due to its long half-life d) Tolbutamide due to its short half-life e) Glimepiride due to its long half-life 11) Which of the following is NOT a typical reason for secondary failure (failure to maintain a good response to sulfonylurea over the long term) in patients on sulfonylurea therapy?
  • 37. Endocrine 05May2009 DO NOT DISTRIBUTE - 37 - a) Reduction in physical activity b) Decline in lean body mass c) Decline in fat in chronic type-2 diabetes d) Decrease in B-cell mass 12) Why should meglitinides (repaglinide) be avoided as adjunctive treatment in patients taking a sulfonylurea drug? a) Their binding sites overlap b) They bind to each other forming a non-soluble product c) They bind to each other, which inactivates the sulfonylurea agent d) They bind to each other, which inactivates the meglitinide agent e) They can be used safely in combination 13) Which of the following is NOT a postulated mechanism of action for biguanides? a) Reduced hepatic and renal gluconeogenesis b) Inhibition of pancreatic beta cells c) Slowing of glucose absorption from GI tract with increased glucose to lactate conversion by enterocytes d) Direct stimulation of glycolysis in tissues with increased glucose removal from blood e) Reduction of plasma glucagon levels 14) Thiazolidinediones (Tzds) bind to peroxisome proliferator-activated receptor-gamma (PPAR-g) receptors in muscle, fat, and liver. Which of the following is NOT a receptor involved in gene activation by Tzds? a) Lipid metabolism b) Insulin signal tranduction c) Adipose tissue differentiation d) Glucose metabolism e) Glucagon signal tranduction 15) Which of the following is NOT a postulated mechanism of action for exenatide? a) Central loss of appetite b) Potentiating of glucose-mediated insulin secretion c) Enhanced gastric emptying d) Suppression of postprandial glucagon release Endocrine #14 – Pharmacology: Agents That Affect Bone Mineral Homeostasis 1) Which of the following reactions is inhibitory? a) Serum calcium/phosphate to bone via vitamin D b) Serum calcium/phosphate to bone via PTH c) Bone to serum calcium/phosphate via vitamin D d) Bone to serum calcium/phosphate via PTH e) Bone to serum calcium/phosphate via calcitonin 2) What affect do bisphosphonates have on farnesyl-pyrophosphate synthase? a) Inhibitory leading to inhibited osteoclast function b) Inhibitory leading to enhanced osteoclast function c) Stimulatory leading to inhibited osteoclast function d) Stimulatory leading to enhanced osteoclast function
  • 38. Endocrine 05May2009 DO NOT DISTRIBUTE - 38 - 3) Which of the following is a selective estrogen receptor modulator (SERM) approved for the treatment of osteoporosis? a) Octreotide (Sandostatin) b) Alendronate (Fosemax) c) Raloxifene (Evista) d) Ergocalciferol (Calciferol) e) Calcitonin (Calcimar) 4) What advantage does teriparatide have over fluoride in the treatment of osteoporosis? a) Reduced incidence of hypocalcemia b) Reduced incidence of hypercalcemia c) Reduced incidence of CNS symptoms d) Reduced incidence of bone fractures e) Reduced incidence of prolonged bleeding 5) Which of the following is a bisphosphonate approved for osteoporosis? a) Octreotide (Sandostatin) b) Alendronate (Fosemax) c) Raloxifene (Evista) d) Ergocalciferol (Calciferol) e) Calcitonin (Calcimar) 6) Bisphosphonates may be administered via IV or orally. If administered orally, which of the following protocols should be followed? a) Take on an empty stomach without any fluids b) Take on an empty stomach with water and stay upright c) Take with food and avoid staying still for prolonged periods d) Take with milk only or a lactose-like liquid for vegan patients e) Take with a full glass of red wine or shot of 80-proof liquor 7) Which of the following drugs is used for osteoporosis and stimulates new bone formation, rather than inhibiting bone resorption? a) Reveromycin A b) Indomethacin c) Diphenylhydantoin d) Teriparatide e) Alendronate Endocrine #15 – Pathology 1) Which of the following is more characteristic of Type-2 diabetes mellitus compared with Type-1 pancreatic morphologic changes? a) Leukocyte infiltration of the islets b) Great reduction in islet cell number and size c) Amyloid replacement of islet cells d) Beta cell degranulation 2.1) What is the number one complication for patients with diabetes mellitus? a) Neuropathy b) Foot gangrene c) Atherosclerosis d) Glomerulosclerosis
  • 39. Endocrine 05May2009 DO NOT DISTRIBUTE - 39 - e) Hyperlipidemia 2.2) What is the most common cause of death in diabetics? a) Myocardial infarction b) Stroke c) Renal failure d) Kimmelstein-Wilson lesions e) Hypertensive crisis 2.3) Which of the following complications of diabetes is caused by hypertension? a) Diabetic macrovascular disease b) Hyaline arteriosclerosis c) Diabetic microangiopathy d) Diabetic nephropathy e) Diffuse mesangial sclerosis f) Nodular glomerulosclerosis 2.4) Which of the following involves PAS-positive lesions which may trap mesangial cells (Kimmelstein-Wilson lesions) and if present is essentially pathognomonic for diabetes? a) Diabetic macrovascular disease b) Hyaline arteriosclerosis c) Diabetic microangiopathy d) Diabetic nephropathy e) Diffuse mesangial sclerosis f) Nodular glomerulosclerosis 2.5) Which of the following complications of diabetes is responsible for the disease- defining neuropathy, nephropathy, and retinopathy? a) Diabetic macrovascular disease b) Hyaline arteriosclerosis c) Diabetic microangiopathy d) Diabetic nephropathy e) Diffuse mesangial sclerosis f) Nodular glomerulosclerosis 2.6) Which of the following is NOT characteristic of Type-1 diabetes mellitus? a) Ketoacidosis b) Polyuria c) Polydipsia d) Polyphagia e) Obesity 2.7) Which of the following describes the type of coma seen in dehydrated Type-2 diabetes mellitus patients? a) Hypo-osmolar, non-ketonic b) Hyper-osmolar, non-ketonic c) Hypo-osmolar, ketonic d) Hyper-osmolar, ketonic 2.8) What is the first sign of diabetic nephropathy in Type-2 diabetics? a) Failure to concentrate urine b) Subtle amounts of albumin in urine
  • 40. Endocrine 05May2009 DO NOT DISTRIBUTE - 40 - c) Dark, strong smelling urine d) High LDL and TG blood levels e) Low HDL blood levels 3.1) A patient presents with confusion and stupor after running a marathon. Testing shows a blood glucose level below 50mg/dL. The patient’s friend says these symptoms have occurred before during marathon training. Pancreatic imaging shows small, encapsulated, solitary tumors. Which of the following is most likely? a) Beta-cell tumor (insulinoma) b) Zollinger-Ellison syndrome c) Alpha-cell tumor (glucagonoma) d) Delta-cell tumor (somatostatinoma) e) VIPoma 3.2) A patient presents with steatorrhea, hypochlorhydria, cholelithiasis, and diabetes mellitus. Which of the following is most likely? a) Beta-cell tumor (insulinoma) b) Zollinger-Ellison syndrome c) Alpha-cell tumor (glucagonoma) d) Delta-cell tumor (somatostatinoma) e) VIPoma 3.3) A patient with a history of multiple endocrine neoplasia type 1 (MEN-1, Wermer syndrome) presents with peptic ulcers and diarrhea. Testing shows tumor metastasis. Which of the following is most likely? a) Beta-cell tumor (insulinoma) b) Zollinger-Ellison syndrome c) Alpha-cell tumor (glucagonoma) d) Delta-cell tumor (somatostatinoma) e) VIPoma 3.4) A post-menopausal woman presents with a skin rash. Testing shows anemia and a glucose level suggestive of diabetes mellitus. Which of the following is most likely? a) Beta-cell tumor (insulinoma) b) Zollinger-Ellison syndrome c) Alpha-cell tumor (glucagonoma) d) Delta-cell tumor (somatostatinoma) e) VIPoma 3.5) Which of the following is associated with watery diarrhea, hypokalemia, and achlorhydria (WDHA syndrome)? a) Beta-cell tumor (insulinoma) b) Zollinger-Ellison syndrome c) Alpha-cell tumor (glucagonoma) d) Delta-cell tumor (somatostatinoma) e) VIPoma 4.1) A young woman is being treated in the emergency department for psychosis and is given haloperidol and reserpine. After developing galactorrhea, the patient admits to not having her period in some time (amenorrhea). A pregnancy test is negative. After a CT scan of the brain returns, the clinician opts to avoid invasive prodecures but speculates she would have found dystrophic calcification. Which of the following is most likely?
  • 41. Endocrine 05May2009 DO NOT DISTRIBUTE - 41 - a) Somatotrophic adenoma b) Prolactinoma c) Corticotrophic adenoma d) Empty sella syndrome e) Sheehan syndrome 4.2) A patient presents with a history of Cushing syndrome which was resolved after removal of the adrenal glands. CT scan shows an anterior lobe pituitary adenoma that has grown since the operation (Nelsons syndrome). A post-mortem biopsy of the tumor shows densely granulated and PAS-positive staining. Which of the following is most likely? a) Somatotrophic adenoma b) Prolactinoma c) Corticotrophic adenoma d) Empty sella syndrome e) Sheehan syndrome 4.3) An adult presents with acromegaly and a broad face. Testing shows high levels of serum GH and IGF-1. An oral load of glucose is administered, but serum GH levels are still high. A CT scan shows a large pituitary adenoma of the anterior lobe. What is the mechanism by which this tumor arose? a) Defect in p53 gene causing inability to force cell apoptosis b) Defect in GTPase of the alpha-subunit of G-protein (Gs) c) Defect in bcl-2 and bak genes d) Defect in proto-oncogenes bcl-2 and c-myc e) Gain on function mutation resulting in cAMP overproduction 5) During parturition, a woman develops obstetrical hemorrhage, which is controlled by the hospital staff. Shortly after birth, the woman develops hypotension, starts losing her pubic hair, and is unable to breast feed. CT scanning of the head shows likely ischemic necrosis in the sella turcica. Which of the following is most likely? a) Somatotrophic adenoma b) Prolactinoma c) Corticotrophic adenoma d) Empty sella syndrome e) Sheehan syndrome 6) An obese woman with hypertension presents with complains of visual defects. Testing shows hyperprolactinemia. CT scan shows herniation of the arachnoid matter and cerebrospinal fluid into the sella turcica. Which of the following is most likely? a) Somatotrophic adenoma b) Craniopharyngioma c) Corticotrophic adenoma d) Empty sella syndrome e) Sheehan syndrome 7) A child presents with poor peripheral vision. A CT scan shows a cystic, multi- lobulated tumor above the sella turcica. Surgical removal is performed and the tumor is sent to pathology. Testing shows dystrophic calcification and cysts lined with squamous epithelium. The cysts are filled with a yellow, viscous fluid that is rich in cholesterol
  • 42. Endocrine 05May2009 DO NOT DISTRIBUTE - 42 - crystals. The tumor likely developed from vestigial remnants of Rathkes pouch. Which of the following is most likely? a) Somatotrophic adenoma b) Craniopharyngioma c) Corticotrophic adenoma d) Empty sella syndrome e) Sheehan syndrome 8) An ICU patient with recent head trauma is found to be creating excess urine. Testing of the urine shows a low specific gravity. Serum testing shows elevated sodium. The patient complains of being very thirsty. ADH deficiency is suspected. Which of the following is most likely? a) Central diabetes insipidus b) Nephrogenic diabetes insipidus c) Type-1 diabetes mellitus d) Type-2 diabetes mellitus e) Syndrome of inappropriate ADH secretion (SIADH) 9) A patient presents with shortness of breath. Testing reveals high ADH levels. Chest x- ray reveals small cell carcinoma of the lung. If SIADH is suspected, which of the following would be expected on labs and during physical exam? a) Hypernatremia and edema b) Hyponatremia and edema c) Hypernatremia without edema d) Hyponatremia without edema 10) Which of the following is a sign of hypothyroidism? a) Lateral eyebrow hair loss b) Patient always feels hot (hyperpyrexia) c) Excessive perspiration d) Palpitations, tachycardia e) High systolic pressure with normal diastolic pressure (high pulse pressure) 11) Which of the following is a sign of hyperthyroidism (e.g. thyrotoxicosis)? a) Retardation and deficient growth (cretinism) b) Puffy eyelids, narrow palpebral fissure, protruded tongue c) Fine-amplitude hand tremors d) Cold intolerance and scaly dermatitis e) Deftness, night blindness, somnolence, dulled reflexes 12) Which of the following would NOT cause symptoms similar to hypothyroidism? a) Thyroid hormone resistance syndrome b) Autoimmune (Hashimoto) thyroiditis c) Use of lithium, iodides, or p-aminosalicyclic acid d) Excessive iodine e) Pituitary failure 13) A middle-aged woman from a third world country presents complaints of feeling tired. History reveals Sjögren syndrome. Physical exam reveals an enlarged, non-nodular thyroid. Labs reveal euthyroid. Biopsy shows inflammatory destruction of thyroid follicles and Hürthle cells (large eosinophilic cells showing oxyphilic metaplasia of thyroid follicular cells). Which of the following is most likely?
  • 43. Endocrine 05May2009 DO NOT DISTRIBUTE - 43 - a) Subacute (DeQuervain) thyroiditis b) Primary hyperthyroidism c) Graves disease d) Lymphocytic (Hashimoto) thyroiditis e) Primary hypothyroidism 14) A patient presents with jaw and neck pain with swallowing. History reveals a recent influenza infection. Exam reveals hoarseness and an exquisitely tender and enlarged thyroid. A 131I-uptake test shows decreased uptake of iodide diffusely. Although a biopsy is not performed, granulomatous inflammation with multinucleated giant cells would be expected as well as microabscesses with neutrophilic infiltration. Which of the following is most likely? a) Subacute (DeQuervain) thyroiditis b) Primary hyperthyroidism c) Graves disease d) Lymphocytic (Hashimoto) thyroiditis e) Primary hypothyroidism 15) A middle-aged female presents with agitation and hyperpyrexia. On ocular exam, the patient appears to be wide-eyed and extremely alert. Labs reveal HLA-B8 and DR3. IgG autoantibodies to the TSH receptor is suspected. Although a biopsy is not performed, a scalloped “moth-eaten” thyroid appearance would be expected. Which of the following is most likely? a) Subacute (DeQuervain) thyroiditis b) Primary hyperthyroidism c) Graves disease d) Lymphocytic (Hashimoto) thyroiditis e) Primary hypothyroidism 16) A patient from central Asia presents with an enormous, multinodular neck mass. History reveals a diet rich in cabbage, turnips, and Brussel sprouts. Biopsy reveals the plunging goiter is very rick in colloid material. Which of the following demographics is most likely for this patient? a) Young boy b) Young girl c) Elderly man d) Elderly woman e) Pregnant woman 17) What is the hallmark of a follicular adenoma? a) Painless neck mass with dysphagia b) Hürthle cells with eosinophilic granular cytoplasm c) Well-formed capsule around the tumor d) Multinodular with a capsule that is penetrated by tissue e) Overproduction of cAMP with signs of malignancy 18.1) Which of the following carcinomas of the thyroid gland is derived from parafollicular or C cells and has a strong association with MEN-IIa and IIb? a) Papillary carcinoma b) Follicular carcinoma c) Medullary carcinoma
  • 44. Endocrine 05May2009 DO NOT DISTRIBUTE - 44 - d) Anaplastic carcinoma 18.2) A 30-year-old woman presents with complaints of a neck lump. Physical exam reveals a mass in the cervical lymph node. A solitary thyroid nodule is found and is freely moveable. Resection of the mass reveals papillae (stalks covered by cuboidal epithelium) and psammoma bodies. Microscopy shows empty cells and ground glass or Orphan Annie eye nuclei. Which of the following is most likely? a) Papillary carcinoma b) Follicular carcinoma c) Medullary carcinoma d) Anaplastic carcinoma 18.3) A 65-year-old patient presents with a history of multinodular goiter. Testing reveals a highly anaplastic undifferentiated neoplasm with giant cells. What is the prognosis for this patient? a) 0% rate of mortality b) 25% rate of mortality c) 50% rate of mortality d) 75% rate of mortality e) 100% rate of mortality 18.4) A 50-year-old woman presents with a thyroid carcinoma that has traveled hematogenously to the bones of the shoulder, pelvis, sternum, and skull. Which of the following is most likely? a) Papillary carcinoma b) Follicular carcinoma c) Medullary carcinoma d) Anaplastic carcinoma 19.1) Which of the following is the most common cause of hyperparathyroidism? a) MEN-1 inactivation on chromosome 11 b) MEN-2 tyrosine kinase receptor mutation c) Familial hypocalcinuric hypercalcemia (FHH) due to CASR gene mutation d) Parathyroid adenoma due to relocation of PRAD1 proto-oncogene, over- expressing cyclin D1 protein 19.2) Which of the following affects all four parathyroid glands and results in water-clear cells? a) Parathyroid adenoma b) Parathyroid hyperplasia c) Parathyroid carcinoma 19.3) A 45-year-old woman presents with bone pain, abdominal pain, and depression. Testing reveals nephrolithiasis and elevated PTH levels. Which of the following is most likely? a) Hypercalcemia of malignancy b) Vitamin D toxicity c) Thiazide diuretic use d) Hyperparathyroidism e) Granulomatous disease 20) A hospitalized patient is found to have chronic depression of serum calcium levels. Parathyroid gland analysis reveals hyperplasia with increased chief cells and water-clear
  • 45. Endocrine 05May2009 DO NOT DISTRIBUTE - 45 - cells. Further testing reveals bone changes and metastatic calcification (calciphylaxis). What is the most common cause of this patient’s secondary hyperparathyroidism? a) Vitamin D deficiency b) Inadequate intake of calcium c) Renal failure d) Steatorrhea 21) A patient with DiGeorge syndrome undergoes surgery to remove lymph nodes in the neck. Long after the surgery, the patient develops emotional instability and Parkinson- like movements. Physical exam reveals papilledema and cataracts. Testing reveals prolonged QT interval and calcification of the basal ganglion. Tapping on the facial nerve induces ipsilateral muscle contraction (Chvostek sign). Blood pressure cuff occlusion reveals carpal spasm (Trousseau sign), which disappears as soon as the cuff is removed. Which of the following is most likely? a) Hyperthyroidism b) Hypothyroidism c) Hyperparathyroidism d) Hypoparathyroidism e) Pseudohypoparathyroidism f) Pseudopseudohypoparathyroidism 22.1) Pseudohypoparathyroidism is due to end-organ resistance to the actions of PTH, thus PTH levels may be normal or elevated. This is due to G,alpha-mediated pathway expression in the kidney of GNAS1 gene. Which of the following describes pseudohypoparathyroidism type-1A? AHO = Albright hereditary osteodysrophy a) AHO with multi-hormone resistance and maternal allele mutation b) AHO with multi-hormone resistance and paternal allele mutation c) AHO without multi-hormone resistance and maternal allele mutation d) AHO without multi-hormone resistance and paternal allele mutation 22.2) Which of the following describes pseudopseudohypoparathyroidism? a) AHO with multi-hormone resistance and maternal allele mutation b) AHO with multi-hormone resistance and paternal allele mutation c) AHO without multi-hormone resistance and maternal allele mutation d) AHO without multi-hormone resistance and paternal allele mutation 23.1) Which of the following is the most common cause of Cushing syndrome? a) Iatrogenic (exogenous corticosteroid administration) b) Adrenal (hypersecreting cortisol adenoma) c) Paraneoplastic (ACTH secretion from small cell lung cancer) d) Pituitary (ACTH anterior pituitary tumor) 23.2) Which of the following forms of Cushing syndrome would have high serum cortisol and low ACTH, and is also called “independent Cushing syndrome”? a) Iatrogenic (exogenous corticosteroid administration) b) Adrenal (hypersecreting cortisol adenoma) c) Paraneoplastic (ACTH secretion from small cell lung cancer) d) Pituitary (ACTH anterior pituitary tumor) 23.3) Which of the following forms of Cushing syndrome would have high serum ACTH and cortical hyperplasia? a) Iatrogenic (exogenous corticosteroid administration)
  • 46. Endocrine 05May2009 DO NOT DISTRIBUTE - 46 - b) Adrenal (hypersecreting cortisol adenoma) c) Paraneoplastic (ACTH secretion from small cell lung cancer) d) Pituitary (ACTH anterior pituitary tumor) 23.4) Which of the following would occur in Cushing syndrome caused by small cell cancer of the lung? a) Increased serum ACTH and cortisol b) Decreased serum ACTH and cortisol c) Increased ACTH and decreased cortisol d) Decreased ACTH and increased cortisol e) No change in serum ACTH or corisol 23.5) A patient presents with central trunk obesity, a “buffalo hump,” moon facies, muscle wasting, hirsutism, and hypertension. A 24-hours urine free cortisol level suggests Cushing syndrome. Dexamethasone is given to the patient and ACTH levels are suppressed. Which of the following forms is most likely? a) Iatrogenic (exogenous corticosteroid administration) b) Adrenal (hypersecreting cortisol adenoma) c) Paraneoplastic (ACTH secretion from small cell lung cancer) d) Pituitary (ACTH anterior pituitary tumor) 24) After finding high levels of serum aldosterone and low levels of serum renin, hyperaldosteronism is suspected. Dexamethasone is found to suppress aldosterone production, so an adrenal CT is performed, which finds an adenoma (Conn syndrome). Excision reveals bright-yellow, lipid-laden cortical cells with spironolactone inclusion bodies. Which of the following would also be expected in this patient? a) Hypotension and hypokalemia b) Hypotension and hyperkalemia c) Hypertension and hypokalemia d) Hypertension and hyperkalemia 25) A female newborn is found to have increased androgens, decreased cortisol, and increased ACTH secretion. She develops virilization (e.g. hirsutism) and bilateral hyperplasia of the adrenal glands (congenital hyperplasia). Which of the following enzymes is most likely deficient? a) 17alpha-hydroxylase b) Aromatase c) 11beta-hydroxylase d) 5alpha-reductase e) 21-hydroxylase 26) Which of the following would NOT cause adrenocortical insufficiency? a) Acute hemorrhagic necrosis (Waterhouse-Friderichsen syndrome) b) Congenital adrenal hyperplasia c) Adrenoleukodystrophy (ALD gene on Xq28) d) Autoimmune poly-endocrinopathy syndrome e) AIDS, infection, fungi, tuberculosis 27) A patient presents with photophobia, fatigue, and neck pain. Testing and exam reveals severe hypotension, purpura, and disseminated intravascular coagulation (DIC). Antibiotics are started immediately in hopes of preventing massive bilateral adrenal
  • 47. Endocrine 05May2009 DO NOT DISTRIBUTE - 47 - hemorrhage (Waterhouse-Friderichsen syndrome). What bacterium is most likely responsible? a) Streptococcus agalactiae b) Mycobacterium tuberculosis c) Neisseria meningitidis d) Escherichia coli e) Listeria monocytogenes 28) A patient presents with progressive weakness and easy fatigability. They complain of GI disturbances and have hyperpigmentation. Labs show hypokalemia, hyponatremia, and volume depletion. Exam reveals hypotension. Further testing reveals shrunken adrenal glands. The clinician suspects that the adrenal glands have lost over 90% of their function. Which of the following is most likely? a) Primary adrenocortical insufficiency (Addison disease) b) Autoimmune poly-endocrinopathy syndrome c) Adrenoleukodystrophy (ALD gene on Xq28) d) X-linked adrenal hypoplasia (DAX-1 gene on Xp21) e) Acute hemorrhagic necrosis (Waterhouse-Friderichsen syndrome) 29) Which of the following would NOT be seen in secondary adrenocortical insufficiency, as aldosterone synthesis is normal? a) Decreased ACTH levels b) Deficiency in cortisol output c) Deficiency in androgen output d) Hyponatremia, hypokalemia, hyperpigmentation e) Exogenous ACTH elevates plasma cortisol levels 30) Which of the following does NOT describes an adrenal cortical carcinoma? a) Associated with virilism b) Can be caused by LiFraumeni syndrome or Beckwith-Wiedemann syndrome c) Involves a large and invasive area of necrosis and hemorrhage d) Hyperplastic, well circumscribed, yellow (lipids), nodular lesion e) Commonly invades the adrenal vein and inferior vena cava 31) A patient presents with palpitations and excessive sweating. Testing shows orthostatic hypertension. History reveals dizziness when lifting weights at the gym and sometimes (paradoxically) while defecating. Further testing shows large balls of neoplastic chromaffin cells in the adrenal gland (zellballen). Urinalysis reveals excess vanillylmandelic acid (VMA) due to epinephrine and norepinephrine breakdown. Where is the tumor most likely located? a) Adrenal cortex, zona glomerulosa b) Adrenal cortex, zona fasciculata c) Adrenal cortex, zona reticularis d) Adrenal medulla e) Extra-adrenal 32) Where is a common location to find an extramedullary paraganglioma? a) Sternoclavicular joint b) Carotid body c) Cisterna chyle d) Ligaments of the knee
  • 48. Endocrine 05May2009 DO NOT DISTRIBUTE - 48 - e) GI tract at splenic flexure 33) Which of the following patients is most likely to develop thymic hyperplasia, which is lymphoid follicles containing mostly B cells? a) Hashimoto thyroiditis b) Sjögren disease c) Multiple sclerosis d) Myasthenia gravis e) Pregnant diabetic 34) Thymomas typically present either due to impingement of the anterior superior mediastinum or due to: a) Hashimoto thyroiditis b) Sjögren disease c) Multiple sclerosis d) Myasthenia gravis e) Pregnant diabetic 35) Which of the following is NOT associated with thymoma? a) Graves disease b) Cushing syndrome c) Pernicious anemia d) Megaloblastic anemia e) Dermatomyositis-polymyositis f) Acquired hypogammaglobulinemia 36) An adult presents with visual disturbances, mental deterioration, and dementia-like behavior. Head CT shows a tumor near the third ventricle that is nearly impeding the aqueduct of Sylvius. Treatment is started to prevent internal hydrocephalus. Which of the following is most likely? a) Medulloblastoma b) Retinoblastoma c) Thymoma d) Pineoblastoma e) Pineocytoma 37.1) Which of the following areas is NOT affected by MEN-1 (Wermer) syndrome? a) Pituitary b) Parathyroid c) Thyroid d) Pancreatic islets e) Adrenal gland 37.2) MEN-2A (Sipple syndrome) and MEN-2B (MEN-III) most often affect what areas? a) Pituitary and parathyroid b) Parathyroid and pancreatic islets c) Pancreatic islets and adrenal d) Adrenal and thyroid e) Thyroid and pituitary 37.3) MEN-2B does not induce hyperparathyroidism like MEN-2A does. However, both types involve mutations in what gene? a) p53
  • 49. Endocrine 05May2009 DO NOT DISTRIBUTE - 49 - b) MEN-1 c) RET d) Bcl-2 e) c-myc Endocrine #16 – Clinical: Hypothalamic Pituitary Disorders 1) Which of the following is NOT a common cause of hypopituitarism? a) Radiotherapy b) Autoimmunity c) Pituitary tumors d) Pituitary operations 2) Which of the following is NOT a symptom of gonadotropin deficiency in women? a) Amenorrhea b) Loss of libido c) Dyspareunia d) Hirsutism e) Vaginal dryness 3) Which of the following is NOT a symptom of gonadotropin deficiency in men? a) Loss of libido b) Erectile dysfunction c) Gynecomastia d) Infertility e) Hypertrophy of the testes 4) Which of the following would NOT be low (or inappropriately normal) in hypopituitarism? a) TSH b) FSH c) LH d) Estradiol e) Testosterone 5) Which of the following hormone abnormalities would be expected in supporting a diagnosis of central hypothyroidism? a) Increased serum FT4 and increased serum TSH b) Increased serum FT4 and decreased serum TSH c) Decreased serum FT4 and increased serum TSH d) Decreased serum FT4 and decreased serum TSH 6) Which of the following is a recently developed test to stimulate growth hormone secretion? a) Tryptophan plus GHRH b) Arginine plus GHRH c) Alanine plus GHRH d) Aspartate plus GHRH e) Glutamine plus GHRH 7) What hormone replacement is essential in ACTH deficiency? a) Glucocorticoid replacement (Hydrocortisone) b) Levothroxine sodium (Synthroid)
  • 50. Endocrine 05May2009 DO NOT DISTRIBUTE - 50 - c) Testosterone (IM or transdermal patch) d) Somatotropin (Humatrope) e) Oxytocin (Pitocin) 8) What is the drug of choice for TSH deficiency? a) Glucocorticoid replacement (Hydrocortisone) b) Levothroxine sodium (Synthroid) c) Testosterone (IM or transdermal patch) d) Somatotropin (Humatrope) e) Oxytocin (Pitocin) 9) For women who do not desire fertility, what is the hormone replacement therapy used for gonadotropin deficiency? a) Estrogen therapy b) Progesterone therapy c) Gonadotropin therapy d) A & B e) B &C 10) What is the hormone replacement therapy used in males with gonadotropin deficiency? a) Estrogen therapy b) Progesterone therapy c) Testosterone therapy d) FSH therapy e) LH therapy Endocrine #17 – Clinical: Pituitary Tumors 1) What tumor size distinguishes a microadenoma from a macroadenoma? a) 1cm b) 2cm c) 3cm d) 4cm e) 5cm 2) Which of the following is NOT a key question to ask when evaluating a pituitary tumor? a) Is the tumor causing a local mass effect? b) Is hypopituitarism present? c) Is there evidence of hormone excess? d) Is the patient pregnant and at risk for Sheehan syndrome? 3) Which of the following is NOT a neurological defect that can occur with a pituitary tumor? a) Visual field and acuity defects (superior tumor extension) b) Hypothalamic syndrome (superior tumor extension) c) Diplopia (lateral tumor extension) d) Parkinson-like movements (lateral tumor extension) e) CSF rhinorrhea (inferior tumor extension) 4) Which of the following hyperfunctioning pituitary tumors is the most common? a) Prolactinomas